You are on page 1of 107

Хворому на гострий бронхіт призначено сульфадиметоксин.

Що потрібно рекомендувати хворому


для попередження можливої нефротоксичної дії? A. Запивати ЛЗ великою кількістю лужного
питва.
B. Приймати сульфаніламід разом з антибіотиком.
C. Приймати ЛЗ 4 рази на день.
D. Постійно контролювати кількість сечі.
E. Приймати ЛЗ після їжі.

При ревматизмі для етіотропної фармакотерапії не використовують:


A. тетрацикліни
B. цефалоспорини
C. макроліди
D. напівсинтетичні пеніциліни
E. природні пеніциліни

Які з перелічених засобів сприяють покращенню ниркового кровоплину і застосовуються в


комплексі лікування хронічного гломерулонефриту?
A. Абсиксимаб
B. Ацетил цистеїн
C. Дротаверин
D. Ацетилсаліцилова кислота
E. Дипіридамол

Хворому на гігантизм лікар призначив засіб етіологічної терапії, але він забув його назву. Про який
засіб йде мова?: A. Лізодрел
B. Парлодел
C. Перитол
D. Конвулекс
E. Цитадрел

У хворого, який тривало приймає інгаляційний ЛЗ з приводу лікування бронхіальної астми,


виникли прояви кандидозу ротової порожнини. ЛЗ якої групи міг призвести до такого ускладнення
фармакотерапії?
A. Неселективні холінолітики.
B. Селективні холіноблокатори.
C. β2-агоністи короткої дії.
D. Системні глюкокортикоїди.
E. Інгаляційні глюкокортикоїди.

У хворої виник епілептичний напад, однак при цьому свідомість була збереженою і вона могла
описати свої відчуття. Вкажіть, для якого виду нападів це характерно?:
A. міоклонічні
B. ккладні парціальні
C. прості парціальні
D. абсанси
E. генералізовані напади

Який з наведених антибактерійних асобів вважається "золотим стандартом" цефалоспоринів І


покоління?
A. Цефаклор.
B. Цефазолін.
C. Цефадроксил.
D. Цефуроксим. E. Цефіксим.
Дайте визначення поняттю «первинна профілактика ревматизму».
A. введення після перенесеної гострої ревматичної гарячки бензатину бензилпеніциліну
щомісячно впродовж індивідуально визначеного терміну
B. введення бензатину бензилпеніциліну впродовж 10 днів хворому на ревматичну ваду серця C.
введення після перенесеного гострого стрептококового тонзиліту бензатину бензилпеніциліну,
одноразово
D. введення після перенесеної гострої ревматичної гарячки бензилпеніциліну у терапевтичній дозі
впродовж 10 днів
E. введення після перенесеного гострого тонзиліту бензатину бензилпеніциліну щомісячно
впродовж індивідуально визначеного терміну

Хворий В., з артеріальною гіпертензією, хронічною серцевою недостатністю ІІA застосовував


еналаприл 5 мг двічі на добу. У хворого виник сухий кашель. Запропонуйте ЛЗ, на який доцільно
замінити еналаприл. A. Телмісартан.
B. Атенолол.
C. Метопролол.
D. Нікардипін.
E. Спіронолактон.

Вкажіть ЛЗ, який неможна застосовувати для лікування сифілісу у дітей до 8 років: A.
оксацилін
B. еритроміцин
C. ампіцилін
D. цефтріаксон
E. тетрациклін

Хворому В., 34 років, встановлено діагноз гострий пієлонефрит. Яка група лікарських засобів у
лікуванні є засобами вибору:
A. Урикозуричні засоби.
B. Спазмолітики.
C. Антибіотики.
D. Нефропротектори. E. Сечогінні.

Хворому з атеросклерозом призначено симвастатин. Який механізм дії даного лікарського засобу?
A. Інгібітори 3-гідрокси-3 метилглутаргіл коензиму А
B. Підвищення активності фосфодіестерази
C. Антиокидантка дія
D. Зниження в крові переважно тригліцеридів
E. Підвищення виведення з організму жовчних кислот і холестерину

Симптом – ознака хвороби може бути:


A. суб'єктивний, об'єктивний, специфічний
B. правильної відповіді немає
C. об’єктивний
D. тільки специфічний
E. специфічний

Лікочутливий туберкульоз – це:


A. Туберкульоз, викликаний штамом мікобактерій, чутливим до усіх антибактерійних ЛЗ.
B. Туберкульоз, викликаний штамом мікобактерій, чутливим до цефалоспоринів.
C. Туберкульоз, викликаний штамом мікобактерій, нечутливим до амікацину, стрептоміцину,
етамбутолу.
D. Туберкульоз, викликаний штамом мікобактерій, чутливим до аміноглікозидів.
E. Туберкульоз, викликаний штамом мікобактерій, чутливим до рифампіцину, стрептоміцину,
ізоніазиду, етамбутолу.

Незвичайна реакція на перше введення ЛЗ – це: A.


Потенціювання.
B. Залежність.
C. Ідіосинкразія.
D. Сенсибілізація. E. Тахіфілаксія.

Який із наведених блокаторів кальцієвих каналів належить до групи похідних фенілалкіламіну? A.


Лацидипін.
B. Фелодипін.
C. Дилтіазем.
D. Лерканідипін. E. Верапаміл.

Патогенетичною основою розвитку цукрового діабету 2 типу є:


A. Інсулінорезистентність
B. Гіперглікемія
C. Інсулінорезистентність, інсулінодефіцит
D. Інсулінодефіцит
E. Гіпоглікемія

Допоможіть молодому колезі обрати симптом не характерний для хвороби Паркінсона: A.


м’язова ригідність
B. гіпомімія
C. «шаркаюча» хода
D. тремор по типу «катання пілюль»
E. прискорення мови

«Синдром «відміни» - це:


A. Побічна дія, обумовлена раптовим припиненням приймання ЛЗ.
B. Відсутність комплаєнсу.
C. Побічна дія, обумовлена підвищеною чутливістю організму до ліків і розвитком тахіфілаксії.
D. Побічна дія, обумовлена специфічними фармакологічними властивостями ліків. E. Помилки у
дозуванні ліків.

Який із вказаних лікарських засобів відноситься до засобів етіотропної фармакотерапії


оперізуючого лишаю (захворювання, яке викликає вірус простого герпесу):
A. ципрофлоксацин
B. ацикловір
C. зидовудин
D. еритроміцин
E. кларитроміцин

Екскреція – це процес
A. незворотного виведення лікарської речовини у незміненому стані
B. незворотного виведення лікарської речовини
C. усі відповіді правильні
D. зворотного виведення лікарської речовини
E. введення лікарської речовини в організм
В аптеку звернувся хворий з рецептами на омепразол 20 мг х 2 р/д, кларитроміцин 0,5 х 2 р/д,
амоксицилін 1г – 2 р/д. Така схема терапії, найімовірніше, була призначена з приводу:

A. Хронічного гастриту В
B. Хронічного холециститу
C. Хронічного панкреатиту
D. Хронічного коліту
E. Хронічного гастриту А

Якими чинниками зумовлений розвиток мегалобластних анемій?


A. недостатністю заліза у крові
B. пригніченням синтезу в нирках еритропоетину
C. прискореним руйнуванням еритроцитів
D. недостатністю фолієвої кислоти або вітаміну В12
E. токсичним пригніченням кісткового мозку

Хворий В., з артеріальною гіпертензією, хронічною серцевою недостатністю ІІA застосовував


еналаприл 5 мг двічі на добу. У хворого виник сухий кашель. Запропонуйте ЛЗ, на який доцільно
замінити еналаприл. A. Телмісартан.
B. Нікардипін.
C. Атенолол.
D. Метопролол.
E. Спіронолактон.

Поліморбідність – це:
A. Поєднання фармакотерапії та інших методів лікування.
B. Раціональне застосування водночас кількох лікарських засобів.
C. Наявність водночас кількох захворювань у пацієнта.
D. Один із основних (обов’язкових) методів діагностики.
E. Нераціональне застосування водночас кількох лікарських засобів.

У хворого на пептичну виразку під час комбінованої фармакотерапії випорожнення набули чорного
забарвлення. Назвіть лікарський засіб, який міг спричинити таку зміну:
A. Вісмуту субцитрат
B. Метронідазол
C. Омепразол
D. Ранітидин
E. Актовегін

Застосування лікарських засобів з метою попередження розвитку захворювання чи виникнення


патологічних станів – це:
A. Базова фармакотерапія.
B. Відповідальна фармакотерапія.
C. Комплексна фармакотерапія.
D. Фармакотерапія окремих груп пацієнтів (дітей, вагітних, похилого віку). E. Фармакопрофілактика.

У хворого 40 р., який хворіє на артеріальну гіпертензію та отримує відповідне лікування, з’явились
ядуха та набряки нижніх кінцівок. Який з наведених гіпотензивних засобів необхідно додати у
лікувальну схему ? A. Індапамід.
B. Метилдопа.
C. Верапаміл.
D. Доксазозин.
E. Пропранолол.
Найпростішим, але найменш точним методом розрахунку дози для дитини є розрахунок за:

A. Юнгом
B. Ленартом
C. Номограмами
D. Харнаком
E. Кларком

Хворому з гіперацидним гастритом назначено ранітидин. Який механізм дії цього лікарського
засобу?
A. Блокує Н2-гістамінові рецептори шлунка
B. Блокує фермент Н+-К+-АТФ-азу, яка відповідає за продукцію соляної кислоти
C. Утворює плівку з колоїду, яка захищає чутливі нервові закінчення від дії соляної кислоти
D. Викликає дегідратацію або часткову коагуляцію тканних білків чи ранового ексудату і утворюють
альбумінати, які захищають слизову оболонку
E. Нейтралізує соляну кислоту в шлунку

Назвіть антибіотик з групи цефалоспоринів 1-го покоління, що вважається "золотим стандартом"


парентеральної терапії.
A. Цефуроксим.
B. Цефтріаксон.
C. Цефіксим.
D. Цефазолін.
E. Цефотаксим.

У жінки 50 р., біль у суглобах кистей рук, значна ранкова скутість; ШОЕ 24 мм/год. Який діагноз
можна припустити? A. ревматоїдний артрит
B. дерматоміозит
C. системний червоний вовчак
D. хронічний активний гепатит
E. системна склеродермія

Який протитуберкульозний засіб міг викликати у хворого червоне забарвлення сечі і харкотиння?
A. Рифампіцин
B. ПАСК
C. Офлоксацин
D. Етамбутол
E. Ізоніазид

Бета-гемолітичний стрептокок є головним етіологічним чинником:


A. Хронічного гастриту типу В
B. Гострого гломерулонефриту
C. Хронічного пієлонефриту
D. Гострого ентероколіту
E. Хронічного гастриту типу А

Які з перелічених нижче лікарських засобів не покращують нирковий кровоплин? A.


Курантил
B. Аспірин
C. Трентал
D. Гепарин
E. Еноксапарин

У хворої П., яка приймала петльовий діуретик з’явились м’язова слабкість, судоми м’язів нижніх
кінцівок, відчуття перебоїв у роботі серця. Назвіть ЛЗ для корекції даного ускладнення.
A. Піридоксин та магнію лактат.
B. Магнію сульфат.
C. Алюмінію фосфат.
D. Дефероксамін.
E. Калію та магнію аспарагінат.

Зниження рівня альбуміну в сироватці може спричинити:


A. прискорення вивільнення лікарського засобу
B. підвищення взаємодії лікарських засобів
C. зниження концентрації вільного лікарського засобу
D. збільшення ризику прояву побічних ефектів лікарського засобу
E. пригнічення терапевтичного ефекту

Ревматизм характеризується::
A. ізоімунізацією
B. порушенням імунологічної толерантності
C. активністю «заборонених клонів»
D. мутацією антитілоутворюючих клітин
E. руйнуванням тканинних структур серця

У хворого М., що лікується з приводу гострої ревматичної гарячки, виявлено гіперчутливість до


пеніциліну. Які з перерахованих груп протимікробних ЛЗ йому показані?
A. тетрацикліни
B. макроліди
C. сульфаніламіди
D. фторхінолони
E. цефалоспорини

Які з перелічених збудників найчастіше є причиною розвитку гострого гломерулонефриту? A.


Диплокок
B. Кишкова паличка
C. Ентеровірус
D. Стафілокок
E. Стрептокок

Яким найбільш близьким за дією ЛЗ можна замінити відсутній в аптеці бромгексин? A.


Лібексин.
B. Амброксол.
C. Кодеїну фосфат.
D. Натрію гідрокарбонат. E. Натрію хлорид.

Основний шлях елімінації більшості лікарських засобів з організму:


A. з жовчю
B. правильна відповідь відсутня
C. нирками
D. з видихуваним повітрям
E. 50% нирками, 50% з жовчю
При якій супутній патології застосування ксилометазоліну та інших симпатоміметиків не
рекомендовано?
A. Кишкова непрохідність.
B. Артеріальна гіпотензія.
C. Гломерулонефрит.
D. Епілепсія.
E. Глаукома.

Який протитуберкульозний засіб має найбільш виражену ототоксичну дію?


A. Ізоніазид
B. Стрептоміцину сульфат
C. Етіонамід
D. Рифампіцин
E. Циклосерин

У хворого 53 років, дрібновузловий цироз печінки. Приймає гепатопротектори і


глюкокортикостероїди. Протягом останніх 3 місяців скаржиться на наростаючі набряки нижніх
кінцівок, збільшення живота, ядуху. Яку комбінацію ЛЗ доцільно додати до лікування, що вже
здійснюється?
A. Ретаболіл + фуросемід
B. Верошпірон + фуросемід
C. Альбумін + аскорутин
D. Верошпірон + аскорутин
E. Лідокаїн + гіпотіазид

Як правильно вводити препарати інсуліну за базис-болюсною схемою?


A. Базисний інсулін зранку, інсулін ультракороткої дії після їжі
B. Базисний інсулін та інсулін ультракороткої дії зранку
C. Базисний інсулін перед сном, інсулін ультракороткої дії зранку і ввечері
D. Базисний інсулін 2 рази на день, інсулін ультракороткої з кожним прийомом їжі
E. Базисний інсулін ввечері, інсулін ультракороткої дії перед кожним прийомом їжі

Тривалість антибіотикотерапії при лікуванні неускладненої форми фарингіту становить:


A. 10 днів.
B. 14 днів.
C. 7 днів.
D. 5 днів.
E. Антибіотикотерапія не проводиться.

Блокатори бета-адренорецепторів можуть викликати наступні побічні реакції, окрім:


A. Брадикардія.
B. Прогресування серцевої недостатності.
C. Тахікардія.
D. Гіпотензія.
E. Бронхоспазм.

Який із вказаних лікарських засобів відноситься до засобів етіотропної фармакотерапії


оперізуючого лишаю (захворювання, яке викликає вірус простого герпесу):
A. ципрофлоксацин
B. еритроміцин
C. зидовудин
D. ацикловір
E. кларитроміцин
Оберіть ЛЗ, який не варто застосовувати для корекції артеріального тиску при ішемічному інсульті
A. лабеталол
B. урапідил
C. еналаприл
D. фуросемід
E. есмолол

За допомогою в аптеку звернувся хворий, у якого декілька годин тому після стресової ситуації
виник різкий біль у крижах з іррадіацією в живіт і пахвинну область; псевдопозиви на
сечовиділення. Сказав, що сеча має червонуватий колір. Яка найбільш вірогідна патологія
зумовлює такий стан? A. Напад ниркової кольки.
B. Гострий коліт
C. Гострий цистит
D. Радикуліт
E. Пієлонефрит

Який з перелічених засобів Ви оберете в першу чергу для ліквідації приступу ниркової кольки? A.
Метамізол
B. Ацетамінофен
C. Атропін
D. Нітроксолін
E. Дротаверин

На фармакокінетичні та фармакодинамічні процеси, пов’язані із лікарськими засобами в організмі


пацієнта впливають усі індивідуальні особливості хворого, окрім:
A. Особливі стани (вагітність, лактація)
B. Вікові особливості пацієнтів
C. Фізико-хімічні властивості лікарських засобів
D. Патологія внутрішніх органів (печінки, нирок, серця)
E. Генетична індивідуальність пацієнтів

Гострий інтенсивний біль в поперековій ділянці при наявності дизурії, піурії і лихоманки є ознакою:
A. Гострого гломерулонефриту
B. Ниркової кольки
C. Оперізуючого лишаю
D. Радикуліту
E. Гострого пієлонефриту

Зниження ефекту при повторному введенні ЛЗ – це


A. Лікова залежність.
B. Кумуляція.
C. Звикання.
D. Резистентність. E. Ідіосинкразія.

У жінки, яка тривалий час лікується з приводу інфільтративно-вогнищевого туберкульозу, різко


знизився слух. Який з перерахованих лікарських засобів міг спричинити таку побічну дію? A.
Рифампіцин.
B. Стрептоміцин.
C. Ізоніазид.
D. Етамбутол.
E. Етіонамід.
У хворої 36 років виявлено атипову пневмонію, яка викликана хламідійною інфекцією. Який
антибактеріальний засіб доцільно рекомендувати у даному випадку ?
A. Цефуроксим.
B. Рокситроміцин.
C. Ампіцилін.
D. Амоксицилін.
E. Ванкоміцин.

Серед вказаних чинників, оберіть ті, які впливають на біодоступність лікарського засобу:
A. стать
B. хімічна природа лікарської речовини, її розчинність і проникність
C. значення відносної біодоступності
D. час і кратність дозування
E. значення абсолютної біодоступності

Дистрибуція лікарських засобів може відбуватись в:


A. крові
B. м’язах
C. жировій тканині
D. крові та інших рідинах і тканинах організму
E. лімфі

Лікарським засобом вибору для проведення антиагрегантної терапії при стабільній стенокардії у
випадку протипоказань до ацетилсаліцилової кислоти є: A. Клопідогрель.
B. Мелоксикам.
C. Пентоксифілін.
D. Тиклопідин.
E. Дипіридамол.

Абсолютним протипоказанням до проведення тромболітичної терапії тканинним активатором


плазміногену при ішемічному інсульті є
A. судомні напади в дебюті інсульту
B. глікемія менше 2,75 ммоль/л чи більше 22 ммоль/л
C. рівень тромбоцитів в крові менше 100 000 в 1 мкл
D. тяжка ЧМТ чи інсульт у попередні 3 місяці
E. вагітність

Яка побічна реакція НЕ характерна для ЛЗ заліза:


A. метеоризм
B. потемніння зубів
C. алергічні реакції
D. забарвлення калу у чорний колір
E. поліурія

Тривалість антибіотикотерапії при лікуванні лакунарної ангіни становить:


A. 14 днів.
B. 7 днів.
C. Антибіотикотерапія не проводиться.
D. 5 днів.
E. 10 днів.
Хворий В., з артеріальною гіпертензією та хронічною серцевою недостатністю, приймав
периндоприл по 5 мг на добу. У хворого виник сухий кашель. Ваші поради пацієнту. Оберіть
правильну відповідь
A. Консультація провізора: продовжувати приймати периндоприл, додати амброксол.
B. Консультація лікаря: продовжувати приймати периндоприл, додати льодяники від кашлю .
C. Консультація провізора: замінити периндоприл на телмісартан.
D. Консультація лікаря: замінити периндоприл на телмісартан.
E. Консультація лікаря: замінити периндоприл на нікардипін.

Виберіть лікарський засіб, який не належить до протитуберкульозних засобів ІІ ряду:


A. Амікацин.
B. Капреоміцин.
C. Канаміцин.
D. Стрептоміцин.
E. Ципрофлоксацин.

Одним із етіологічних чинників, які спричинюють розвиток хвороби Аддісона є:


A. Туберкульоз
B. Аденома наднирників
C. Черепно-мозкова травма
D. Гіперфункція імунної системи
E. Пухлина гіпофізу

При непереносимості антибіотиків пеніцілінового ряду для профілактики і лікування ревматизму


показані ЛЗ із групи:
A. фторхінолони
B. цефалоспорини
C. сульфаніламіди
D. макроліди
E. тетрацикліни

Посилення ефекту при сумісній дії ЛЗ називається:


A. Антагонізмом.
B. Потенціюванням.
C. Сенсибілізацією.
D. Кумуляцією.
E. Ідіосинкразією.

Обріть шлях введення антибактерійних ЛЗ для лікування позашпитальної пневмонії у пацієнтів І


категорії.:
A. Внутрішньошкірний.
B. Пероральний.
C. Інгаляційний.
D. Внутрішньом’язовий.
E. Внутрішньовенний.

Якi ЛЗ мають найбільш виражену ульцирогенну дiю?


A. Глюкокортикоїднi
B. Антагонiсти кальцiю
C. Антигiстамiннi препарати
D. Ганглiоблокатори
E. Сульфанiламiднi
При гіпертонічному кризі хворому ввели магнію сульфат, в результаті чого настало різке зниження
артеріального тиску і розвинулась гіпотонія. Який з перелічених лікарських засобів можна
застосувати для запобігання побічних ефектів сульфату магнію?
A. Калію хлорид
B. Протаміну сульфат
C. Кальцію хлорид
D. Унітіол
E. Натрію хлорид

Розрахувати константу елімінації лікарського засобу, якщо відомо, що його об’єм дистрибуції
становить 63 л, кліренс – 40 мл/хв
A. 0,347 год-1
B. 0,446 год-1
C. 0,635 год-1
D. 0,111 год-1
E. 0,154 год-1

Для загострення хронiчного панкреатиту характерно


A. Субфебрилiтет, кахексiя, закрiпи
B. Болi в лiвiй половинi живота
C. Нудота, пронос, оперізуючий характер болю
D. Анорексiя, диспепсiя, кахексiя
E. Розлитого характеру болi в животi, диспепсiя

Серед вказаних чинників, оберіть ті, які впливають на біодоступність лікарського засобу:
A. значення абсолютної біодоступності
B. вік
C. значення відносної біодоступності
D. стать
E. активність ферментів, що беруть участь у метаболізмі

У приймальне відділення поступив хворий з гіпертонічним кризом і симптомами набряку легень.


Що необхідно ввести хворому для нормалізації артеріального тиску?
A. Каптоприл під язик
B. Магнію сульфат всередину
C. Атенолол всередину
D. Нітрогліцерин довенно краплинно
E. Атропін підшкірно

У хворого на пептичну виразку під час комбінованої фармакотерапії випорожнення набули чорного
забарвлення. Назвіть лікарський засіб, який міг спричинити таку зміну:
A. Ранітидин
B. Вісмуту субцитрат
C. Актовегін
D. Омепразол
E. Метронідазол

Назвіть інгаляційний глюкокортикоїд тривалої дії, який доцільно застосовувати для профілактики
нападів бронхіальної астми.
A. Теофілін.
B. Сальбутамол.
C. Фенспірид.
D. Орципреналін. E. Флутиказон.

Вкажіть середню тривалість хіміотерапії при туберкульозі легень, пацієнтів 1-ої категорії:
A. 3-6 міс.
B. Більше 12 міс.
C. 1-2 міс.
D. 10-12 міс. E. 7-9 міс.

Який протитуберкульозний засіб міг викликати у хворого червоне забарвлення сечі і харкотиння?
A. ПАСК
B. Ізоніазид
C. Етамбутол
D. Рифампіцин
E. Офлоксацин

До додаткових методів дослідження хворих НЕ належить: A.


Визначення біохімічних показників крові.
B. Рентгенографія органів грудної клітки.
C. Аналіз калу на яйця глистів.
D. Посів харкотиння на флору.
E. Огляд хворого.

Хворому з виразковою хворобою шлунка призначили лікарський засіб, який має одночасно
антацидну, цитопротекторну та бактерицидну на Неlicobacter pilori дію. Назвіть цей лікарський
засіб.
A. Фамотидин
B. Де-нол
C. Альмагель
D. Гастроцепин
E. Маалокс

Яке захворювання можна запідозрити у пацієнта, якщо після зняття нальоту із мигдаликів
з’явилися кровоточиві виразки?
A. Виразково-некротична ангіна.
B. Дифтерія.
C. Флегмонозна ангіна.
D. Лакунарна ангіна.
E. Дизентерія.

Цукровий діабет 1 типу відрізняється від цукрового діабету 2 типу:


A. Хворіють особи із ожирінням
B. Хворіють переважно діти
C. Хворіють переважно чоловіки
D. Хворіють особи старші 40 р.
E. Хворіють особи із обтяженою спадковістю

Синдром – закономірне поєднання симптомів,


A. спричинених єдиним патогенезом.
B. спричинених об’єктивними чинниками
C. спричинених різним патогенезом
D. спричинених єдиним психогенним чинником
E. правильної відповіді немає

До симптомів цукрового діабету належать всі перелічені, окрім:


A. Спрага
B. Поліурія
C. Тремор повік
D. Відчуття затерпання кінцівок
E. Погіршення зору

Діагностичними критеріями акромегалії є:


A. Антропометричний контроль, підвищена концентрація соматостатину
B. Характерна зовнішність, підвищена концентрація соматотропіну
C. Результати рентгенологічного обстеження, підвищена концентрація соматостатину
D. Результати дослідження очного дна, знижена концентрація соматотропіну
E. Результати комп’ютерної томографії, знижена концентрація соматотропіну

Підберіть середньотерапевтичне дозування клопідогрелу для профілактики атеротромбозів:

A. 5 тис. од 4 рази на добу.


B. 75 мг на добу.
C. 100 мг на добу.
D. 1 мг/кг 2 рази на день. E. 250 мг 2 рази на добу.

Оберіть коректний шлях введення вакцини БЦЖ: A.


Внутрішньовенний.
B. Внутрішньом’язовий.
C. Підшкірний.
D. Внутрішньошкірний. E. Трансдермальний.

При якій супутній патології застосування нафазоліну та інших симпатоміметиків не


рекомендовано?
A. Артеріальна гіпертензія.
B. Гастрит.
C. Кишкова непрохідність.
D. Метаболічний алкалоз.
E. Артеріальна гіпотензія.

Безпека фармакотерапії сифілісу у вагітних еритроміцином полягає у тому, що:


A. ЛЗ швидко елімінує з організму
B. ЛЗ не проникає через гематоенцефалічний бар’єр
C. ЛЗ діє поверхнево, шляхом контакту з ураженою поверхнею
D. правильної відповіді немає
E. ЛЗ не проникає через плаценту

Назвіть характерні симптоми гострих синуситів.


A. Біль в навколовушній ділянці, посилюється при натисканні на козелок, ринорея, біль при жуванні.
B. Біль в навколоносовій ділянці, який згодом переходить у біль голови, нежить (з виділенням
слизу або гною), температура фебрильного типу.
C. Біль в навколоносовій ділянці, який посилюється вранці, субфебрильна температура, сухий кашель.
D. Біль горла при ковтанні, задушливий кашель, який посилюється вночі, підвищена пітливіть,
субфебрильна температура.
E. Біль голови, який посилюється вранці, ринорея (виділення слизу з домішками крові), температура
фебрильного типу.

Хвора 52 років перебуває на стаціонарному лікуванні з діагнозом гострого панкреатиту. Який із


перелічених ЛЗ протипоказаний у даному випадку?
A. Папаверин
B. Апротинін
C. Морфін
D. Промедол
E. Метамізол натрію

У хворої з жовчнокам'яною хворобою виник приступ інтенсивного болю в правому підребер’ї,


нудота, блювота. В роті відчуття гіркоти. Який засіб найбiльш ефективний для лікування цього
стану? A. Папаверин
B. Дротаверин
C. Платифiлiн
D. Баралгiн
E. Омепразол

Графічне зображення залежності площі поверхні тіла від маси та зросту дітей називається: A.
Номограма
B. Гістограма
C. Ідеограма
D. Піктограма
E. Скатерограма

Гайморит – це:
A. Запалення верхньощелепної пазухи носа.
B. Запалення клиновидної пазухи носа.
C. Запалення євстахієвої труби.
D. Запалення решітчастого лабіринта носа. E. Запалення лобової пазухи носа.

Характерною клінічною ознакою ревматоїдного артриту є A.


ранкова скутість колінних суглобів тривалістю 5-10 хв.
B. симетричний остеопороз дрібних кісток кистей рук
C. остеоліз кінцевих фаланг пальців рук та стоп
D. деформуюче ураження великих суглобів
E. ранкова скутість понад 3 суглобових зон тривалістю понад 1 год.

Пацієнт Т., 78 р., страждає на хворобу Паркінсона. Приймає леводопу у комбінації з карбідопою 5
разів на добу. Надайте коректну фармацевтичну опіку стосовно дієти пацієнта
A. обмеження білку і рівномірний його розподіл в раціоні; збагачення кальцієм та вітаміном D
B. обмеження жирної їжі, дієта збагачена білком та клітковиною
C. обмеження легкозасвоюваних вуглеводів та цитрусових соків
D. кетогенна дієта із підвищеним вмістом жирів, зниженим вмістом вуглеводів та білку E.
особливостей дієти при застосуванні леводопи у комбінації з карбідопою немає

До аптеки звернувся хворий, у якого після курсу антибіотикотерапії розвинувся дизбактеріоз. Який
препарат доцільно йому призначити?
A. Церукал
B. Бісакоділ
C. Хілак форте
D. Лоперамід
E. Солізім

До Вас в аптеку звернувся хворий на виразкову хворобу шлунку зі скаргами на нудоту, у якого при
обстеженні виявлено підвищену кислотоутворюючу функцію шлунка, Helіcobacter рylorі. Яка група
ЛЗ має бути основою лікування ?
A. Антациди
B. Седативні ЛЗ
C. Коректори моторної функції
D. Антимікробні ЛЗ
E. Антисекреторні ЛЗ

Процес абсорбції описується наступними фармакокінетичними параметрами, окрім:


A. t1/2a
B. tmax
C. Cmax
D. Vd
E. ka

Хворий на артеріальну гіпертензію тривало приймав ЛЗ, який раптово припинив вживати. Після
цього стан хворого погіршився, виник гіпертензивний криз (синдром відміни). Який з ЛЗ володіє
такою побічною дією?
A. Пропранолол.
B. Фуросемід.
C. Моксонідин.
D. Ніфедипін.
E. Еналаприлу малеат.

Хворому з пневмонією призначили антибіотик четвертого покоління цефалоспоринів, який добре


проникає крізь мембрани грамнегативних бактерій, забезпечує стійкість відносно лактамаз і має
широкий спектр антибактеріальної дії. Який засіб лікар призначив хворому?
A. Цефпіром
B. Цефтриаксон
C. Цефалексин
D. Цефазолін
E. Цефатоксим

Комбіноване застосування β2-агоністів (фенотерол) та холінолітиків (іпратропію бромід) у хворих


на бронхіальну астму доцільне:
A. Для зменшення біодоступності холінолітиків.
B. Для профілактики тахіфілаксії.
C. Для зниження кардіотоксичності β2-агоністів.
D. Для посилення антиалергічної дії.
E. Для посилення бронхолітичного ефекту та його подовження.

До «медикаментозного» анамнезу, що збирають у хворого при проведенні моніторингу


фармакотерапії, не належить з’ясування наступних даних: A. Час і шлях введення ЛЗ.
B. Доза і лікарська форма ЛЗ.
C. Комплаєнс і шкідливі звички.
D. Період напіввиведення ЛЗ.
E. Назва лікарського засобу.

Застосування лікарських засобів з метою лікування хворого – це: A.


Фармакокінетика.
B. Фармакодинаміка.
C. Фітотерапія.
D. Фармакотерапія.
E. Фармацевтична опіка хворого.

У хворого Д., з підтвердженою УЗД картиною діагнозу жовчокам’яної хвороби, з’явились біль у
правому підребер’ї, жовтушність шкіри та слизових оболонок. Який із наркотичних анальгетиків
протипоказаний для усунення больового синдрому?
A. Морфін
B. Трамадол
C. Фентаніл
D. Омнопон
E. Промедол

Визначте період напіввиведення гепарину, якщо відомо, що його константа елімінації становить
0,525 год.-1?
A. 3,32 год.
B. 1,32 год.
C. 3,05 год.
D. 2,8 год.
E. 1,5 год.

До антисекреторних ЛЗ належать всі, окрім:


A. Ранітидину
B. Фамотидину
C. Комбінації алюмінію гідроксид+магнію гідроксид
D. Омепразолу
E. Пірензепіну

Назвіть основну ланку патогенезу гломерулонефриту.


A. Інфаркт нирки
B. Токсичний некроз гломерул та канальців
C. Автоімунне ураження клубочків
D. Запально-інфекційне ураження канальців
E. Склероз гломерул (гломерулосклероз)

Вказати до якого синдрому при туберкульозі відносяться симптоми: підвищення температури,


слабкість, пітливість, похудіння, поганий сон:
A. Анемічного
B. Інтоксикаційного
C. Абстинентного
D. Бронхо-легенево-плеврального
E. Обструктивного

Хворий Л., 56 років, ВІЛ-інфікований був госпіталізований з приводу генералізованого


кандидомікозу. Який з нижченаведених протигрибкових препаратів застосовується для лікування
системних мікозів?
A. ундецилова кислота
B. кетоконазол
C. ністатин
D. леворин
E. амфотерицин В
Лікарськими засобами першого вибору для лікування позашпитальних пневмоній у пацієнтів 1
категорії є:
A. Ампіцилін або тетрациклін в/м.
B. Ампіцилін або офлоксацин перорально.
C. Амоксицилін або кларитроміцин перорально.
D. Бензилпеніцилін в/м або еритроміцин перорально.
E. Амоксицилін або норфлоксацин в/в.

Пацієнт пропустив час введення лікарського засобу. Вкажіть, що буде відбуватися із концентрацією
лікарського засобу в плазмі крові:
A. пропорційно збільшується
B. збільшується у двічі
C. хаотично зменшується
D. пропорційно зменшується
E. залишається без змін

Характерними клінічними ознаками акромегалії є:


A. Підвищення артеріального тиску
B. Особливий психічний статус
C. Зниження пам’яті
D. Збільшення кистей, стоп, „грубі” риси обличчя, зміна голосу, порушення обміну речовин
E. Гіпогонадизм

У осiб якої групи кровi найчастiше зустрiчається виразкова хвороба шлунка i 12-палої кишки?
A. А/II/
B. О/I/
C. Не залежить вiд групи кровi
D. АВ/IV/
E. В/III/

У пацієнтів похилого та старечого віку при в/м та п/ш введенні всмоктування ЛЗ, як правило:
A. Прискорене
B. Сповільнене
C. Не прогнозоване
D. Пролонговане
E. Без змін

Для покращення відходження мокроти, дренажної функції бронхів, хворій 52 років, з


бронхіальною астмою призначено лікарський засіб. Через деякий час у хворої з’явились
сльозотеча, ринорея, лоскіт в горлі. Який засіб отримувала хвора?
A. Карбоцистеїн.
B. Ацетилцистеїн.
C. Амброксол.
D. Калію йодид.
E. Бромгексин.

Оберіть метод розрахунку необхідної дози ЛЗ для застосування у геріатрії:


A. 1/8 - 1/10 від середньотерапевтичної дози для пацієнтів середнього віку
B. 2/3 - 3/5 від середньотерапевтичної дози для пацієнтів середнього віку
C. 1/6 - 1/10 від середньотерапевтичної дози для пацієнтів середнього віку
D. немає правильної відповіді
E. ½ - 2/3 від середньотерапевтичної дози для пацієнтів середнього віку
Розрахувати константу елімінації пефлоксацину, якщо відомо, що його період напіввиведення
становить 8 год. A. 0,087 год-1
B. 0,24 год-1
C. 0,031 год-1
D. 0,02 год-1
E. 0,014 год-1

Вкажіть «ідеальну» для фармакотерапії (з точик зору фармакокінетики) частоту застосування


лікарських засобів при багаторазовому їх застосуванні:
A. 1 раз на добу
B. 2 рази на 1 період напіввиведення
C. частота не залежить від фармакокінетичних параметрів
D. 1 раз на 1 період напіввиведення
E. 1 раз на 4-5 періодів напіввиведення

Хворий 65 років, з нападом стенокардії поступив у терапевтичне відділення. Через 48 годин у нього
розвинулась госпітальна пневмонія, була призначена антибактерійна. фармакотерапія. Через 3
доби хворий почав втрачати слух. Назвіть ЛЗ, який міг спричинити глухоту:
A. Мідекаміцин.
B. Левофлоксацин.
C. Амікацин.
D. Цефтріаксон.
E. Фузафунгін.

Який метод найбiльш надiйний для дiагностики пептичної виразки?


A. Ендоскопiчний
B. Рентгенологiчний
C. Дослiдження шлункової секрецiї
D. Дослiдження калу на скриту кров
E. Фiзикальний /пальпацiя, перкусiя/

Гостре бактеріальне захворювання ниркових мисок, мисочок і канальців у вагітних жінок та


породіль - це:
A. Гострий гестаційний пієлонефрит
B. Гострий цистит вагітних
C. Сечокислий діатез
D. Гостра нефропатія вагітних
E. Гострий гломерулонефрит вагітних

Хворий Б., 35 років хворіє на виразкову хворобу асоційовану з Helіcobacter pylorі. Лікар призначив
амоксицилін, омепразол, вісмуту субцитрат, алюмінію гідроксид + магнію гідроксид, фамотидин.
Вкажіть який із перерахованих лікарських засобів володіє антихелікобактерною та
цитопротекторною дією одночасно?
A. Амоксицилін
B. Алюмінію гідроксид+магнію гідроксид
C. Фамотидин
D. Вісмуту субцитрат
E. Омепразол

Антибіотики якого ряду найдоцільніше призначити хворому на сифіліс?


A. аміноглікозиди
B. поліміксин
C. цефалоспорини
D. тетрацикліни
E. пеніциліни

До лікаря звернувся хворий зі скаргами на оперізуючий біль в епігастрії, слабість. Зловживає


алкоголем. В анамнезі – хронічний панкреатит. Який найбiльш цiнний лабораторний показник в
дiагностицi загострення хронiчного панкреатиту?
A. Рiвень лужної фосфатази
B. Лейкоцитоз
C. Рiвень трансамiназ кровi
D. Рiвень амiлази кровi
E. Гiперглiкемiя

Дайте визначення поняттю «вторинна профілактика ревматизму».


A. введення бензатину бензилпеніциліну впродовж 10 днів хворому на ревматичну ваду серця
B. введення після перенесеного гострого стрептококового тонзиліту бензатину бензилпеніциліну,
одноразово
C. введення після перенесеного гострого тонзиліту бензатину бензилпеніциліну щомісячно впродовж
індивідуально визначеного терміну
D. введення після перенесеної гострої ревматичної гарячки бензилпеніциліну у терапевтичній дозі
впродовж 10 днів
E. введення після перенесеної гострої ревматичної гарячки бензатину бензилпеніциліну щомісячно
впродовж індивідуально визначеного терміну

До особливостей елімінації лікарських засобів у дитячому віці не належить:


A. Знижена швидкість клубочкової фільтрації
B. Видільна функція нирок знижена
C. Правильна відповідь відсутня
D. Усі відповіді вірні
E. Видільна функція нирок збільшена

Виберіть антибактеріальний засіб для лікування хламідійної інфекції:


A. пеніцилін
B. лінкоміцин. C. цефазолін
D. гентаміцин
E. роваміцин

У хворого на позашпитальну пневмонію в анамнезі – алергія на ампіцилін у вигляді набряку


Квінке. Який з перелічених антибактеріальних ЛЗ інших груп не слід призначати?
A. Лінкоміцин.
B. Моексифлоксацин.
C. Цефтріаксон.
D. Азитроміцин. E. Спіраміцин.

Найчастішим симптомом серцевої недостатності є:


A. Гепатомегалія.
B. Тахікардія.
C. Спленомегалія.
D. Задишка. E. Олігурія.
Який із блокаторів β-адренорецепторів належить до кардіоселективних:
A. Надолол.
B. Тимолол.
C. Пропранолол.
D. Соталол.
E. Бісопролол

Хворому з атеросклерозом призначено симвастатин. Який механізм дії даного лікарського засобу?
A. Антиокидантка дія
B. Підвищення виведення з організму жовчних кислот і холестерину
C. Інгібітори 3-гідрокси-3 метилглутаргіл коензиму А
D. Підвищення активності фосфодіестерази
E. Зниження в крові переважно тригліцеридів

Оберіть коректний шлях введення туберкуліну для проведення проби Манту:


A. Підшкірний.
B. Внутрішньовенний.
C. Трансдермальний.
D. Внутрішньошкірний. E. Внутрішньом’язовий.

Що не характерно для ПР ЛЗ алергійного ґенезу:


A. Виникають при будь-якому шляху введення ЛЗ.
B. Опосередковані IgЕ.
C. Виникають при введенні ЛЗ в будь-якій дозі.
D. Виникають при першому введенні лікарської речовини. E. Виникають при повторному введенні
ЛЗ.

При якій супутній патології застосування нафазоліну та інших симпатоміметиків не


рекомендовано?
A. Кишкова непрохідність.
B. Артеріальна гіпертензія.
C. Гастрит.
D. Метаболічний алкалоз.
E. Артеріальна гіпотензія.

Назвіть основний критерій компенсації цукрового діабету?


A. Постпрандіальна глікемія
B. Концентрація глюкози перед сном
C. Концентрація глюкози натще
D. Добовий профіль глікемії
E. Рівень глікозильованого гемоглобіну

Які з перелічених збудників найчастіше є причиною розвитку гострого гломерулонефриту? A.


Кишкова паличка
B. Стафілокок
C. Диплокок
D. Стрептокок
E. Ентеровірус

До аптеки звернувся хворий, у якого після курсу антибіотикотерапії розвинувся дизбактеріоз. Який
лікарський засіб доцільно йому призначити?
A. ЛЗ, що містить лакто- та біфідобактерії
B. ципрофлоксацин
C. метоклопрамід
D. бісакоділ
E. лоперамід

У хворого К., 80 р., з’явився тремору в спокої, більш виражений у правій руці, що зникає під час
сну, збіднення рухів та ригідність. Допоможіть обрати лікарський засіб для лікування
захворювання.
A. дулоксетин
B. рилузол
C. кеторолак
D. бетаметазон
E. леводопа+карбідопа

У хворого з артеріальною гіпертензією та хронічним бронхітом посилився сухий кашель.


Температура тіла, аускультативна картина в легенях, лабораторні показники - не змінились.
Відомо, що хворий приймає каптоприл. Поясніть причину цього явища. A.
Збільшене утворення реніну.
B. Зменшене утворення альдостерону.
C. Збільшене утворення брадикініну.
D. Алергічна реакція на каптоприл.
E. Збільшене утворення ангіотензину

У хворого на пептичну виразку під час комбінованої фармакотерапії випорожнення набули


чорного забарвлення. Назвіть лікарський засіб, який міг спричинити таку зміну:
A. Вісмуту субцитрат
B. Омепразол
C. Метронідазол
D. Ранітидин
E. Актовегін

Лікування, що призначається хворому з метою впливу на причину захворювання A.


Етіотропне.
B. Паліативне.
C. Комплексне.
D. Патогенетичне.
E. Симптоматичне

Оберіть ЛЗ, який може застосовуватися для фармакотерапії діареї при колітах шляхом блокади
опіодних рецепторів кишківника:
A. Лактулоза
B. Симетикон
C. Ніфуроксазид
D. Активоване вугілля
E. Лоперамід

Процес підвищення біологічної доступності пов’язаний із:


A. усі відповіді правильні
B. сповільнення процесу біотрансформації
C. модифікацією процесу абсорбції
D. модифікацією процесу розподілу
E. сповільнення процесу елімінації
У хворого на цукровий діабет 2 типу ниркова недостатність, кліренс креатиніну менше 60 мл/хв,
який пероральний цукрознижуючих засіб доцільно обирати?
A. Метформін
B. Піоглітазон
C. Гліквідон
D. Натеглінд
E. Глімепірид

Оберіть невірне твердження стосовно лікарського засобу тригексафенідилу (циклодол):


A. призначають при переважанні у клініці ригідності та гіпокінезії
B. практично не діє на ригідність та гіпокінезію
C. антихолінергічний засіб центральної дії, порушує звʹязки між допаміном та ацетилхоліном у
центральній нервовій системі
D. усі відповіді вірні
E. призначають при переважанні у клініці тремору

Блокатори бета-адренорецепторів можуть викликати наступні побічні реакції, окрім: A.


Бронхоспазм.
B. Тахікардія.
C. Гіпотензія.
D. Брадикардія.
E. Прогресування серцевої недостатності.

Який із наведених ЛЗ належить до неінгаляційних анестетиків для проведення загальної анестезії?


A. диетиловий ефір
B. тіопентал натрію
C. закис азоту
D. ксенон
E. севофлуран

Чоловік 40 р., ріст 130 см, пропорційної тілобудови, інтелект збережений. Яке захворювання
можна припускати?
A. Акромегалію
B. Синдром Іценко-Кушинга
C. Гіперпродукцію соматостатину
D. Гіпофізарний нанізм
E. Гіперпродукцію соматотропіну

У хворого з пневмонією з’явився виснажливий сухий кашель, що спричиняє різкий біль у грудній
клітці. Який протикашлевий лікарський засіб слід порекомендувати хворому у цьому випадку? A.
Еуфілін
B. Фалімінт
C. Настій трави термрпсису
D. Тусупрекс
E. Кодеїну фосфат

Які лікарські засоби у першу чергу застосовуються для усунення ниркової кольки?
A. Антибіотики
B. Антигістамінні ІІІ покоління
C. Наркотичні анальгетики
D. Нестероїдні протизапальні ЦОГ-2
E. Спазмолітики
Ураження шкіри у вигляді вузлуватої та кільцеподібної еритеми, що може локалізуватися на
обличчі, грудях, животі, спині, кінцівках, є проявом:
A. системного червоного вовчака
B. системної склеродермії
C. подагричного артриту
D. гострої ревматичної лихоманки
E. псоріазу

Фронтит – це:
A. Запалення решітчастого лабіринта носа.
B. Запалення лобової пазухи носа.
C. Запалення євстахієвої труби.
D. Запалення гортані.
E. Запалення глотки

Посилення ефекту при сумісній дії лз називається


A. Ідіосинкразією.
B. Кумуляцією.
C. Сенсибілізацією.
D. Потенціюванням. E. Антагонізмом.

Під час оперативного втручання, яке здійснювалось під місцевою анестезією 0,25 % розчином
новокаїну, пацієнт зненацька покрився червоними плямами, виникли інтенсивне потовиділення,
тахікардія, набряк слизової носа, бронхоспазм. Яка причина цього ускладнення ?
A. Синдром віддачі.
B. Тахіфілаксія.
C. Синдром відміни.
D. Алергічна реакція сповільненого типу.
E. Алергічна реакція негайного типу.

Допоможіть обрати антибіотик місцевої дії, який можна застосовувати шляхом інгаляцій після
перенесеної тонзилектомії:
A. Макропен
B. Цефтріаксон
C. Ломефлоксацин
D. Фузафунгін
E. Амоксицилін

Вкажіть, який фармакокінетичний процес може змінюватись в осіб похилого віку внаслідок
зменшення кількості альбумінів плазми:
A. Елімінація
B. Розподіл
C. Всмоктування
D. Метаболізм
E. Екскреція

При застосуванні якого з нижчеперелічених гіпотензивних лікарських засобів може виникнути


виражена брадикардія?
A. Ніфедипін
B. Еналаприл
C. Віскен
D. Доксазозин
E. Метопролол
Хворому в комплексній терапії стенокардії призначено аторвастатин. Вкажіть, з якої групи
лікарських засобів цей препарат.
A. Блокатори протонної помпи
B. Інгібітори 3-гідрокси-3 метилглутаргіл коензиму А
C. Антагоністи кальцію
D. Блокатори рецепторів ангіотензину ІІ
E. Бета-адреноблокатори

Тривалість антибіотикотерапії при лікуванні неускладненої форми фарингіту становить: A.


14 днів.
B. Антибіотикотерапія не проводиться.
C. 7 днів.
D. 5 днів.
E. 10 днів.

Які антагоністи кальцію належать до групи бензотіазепінів: A.


Фелодипін.
B. Ніфедипін.
C. Дилтіазем.
D. Амлодипін.
E. Верапаміл.

Який із наведених блокаторів β-адренорецепторів належать до кардіоселективних: A.


Надолол.
B. Бісопролол.
C. Пропранолол.
D. Соталол.
E. Тимолом.

Швидкість досягнення терапевтичного ефекту визначається наступними фармакокінетичними


параметрами, окрім:
A. tmax
B. Cmax
C. t1/2a
D. AUC
E. період напіввиведення

ЛЗ з групи М-холіноблокаторів, що відзначається селективною дією на дихальні шляхи це: A.


Іпратронію бромід
B. Атропіну сульфат
C. Сальбутамол
D. Ефедрину хлорид
E. Ізадрин

Етмоїдит – це:
A. Запалення середнього вуха.
B. Запалення глотки.
C. Запалення клиновидної пазухи носа.
D. Запалення лобової пазухи носа.
E. Запалення решітчастого лабіринта носа.
При лікування пієлонефриту вагітних в ІІ-ІІІ триместрі при непереносимості антибіотиків
пеніцілінового ряду можливе застосування:
A. Аміноглікозидів
B. Макролідів
C. Стрептоміцину
D. Тетрацикліну
E. Левоміцетину

До блокаторів рецепторів ангіотензину ІІ належить:


A. Телмісартан.
B. Амлодипін.
C. Моексиприл.
D. Флуоксетин.
E. Діазепам.

Оцінка ефективності антибактеріальної терапії ЛЗ І ряду для лікування негоспітальної пневмонії


проводиться:
A. Лише в ІІ групі пацієнтів.
B. Лише в ІV групі пацієнтів.
C. Оцінка ефективності не є обов’язковою.
D. Обов’язково через 5 днів після початку антибіотикотерапії.
E. Обов’язково через 48 год після початку антибіотикотерапії.

Аускультація – це метод обстеження хворого, що ґрунтується на


A. Дотику та промацуванні поверхневих тканин і органів, що знаходяться глибше.
B. Просвічуванні окремих частин тіла рентгенівськими променями.
C. Дослідженні слизової оболонки порожнистих органів за допомогою ендоскопу.
D. Оцінюванні звуку отриманого при вистукуванні поверхні тіла.
E. Вислуховуванні звукових явищ внутрішніх органів.

Жінка 48 років, госпіталізована у кардіологічне відділення з діагнозом ІХС: стенокардія, напади


виникають 1-2 рази на день. Який лікарський засіб найдоцільніше рекомендувати?
A. Ізосорбіду динітрат.
B. Дипіридамол.
C. Теофілін.
D. Дротаверин.
E. Папаверин.

Надайте пацієнту фармацевтичну опіку при відпуску інгаляційного ЛЗ для лікування бронхіальної
астми.
A. Роблячи глибокий вдих через рот, слід натиснути на верхівку інгалятора, затримати дихання на 5
секунд.
B. Роблячи видих через рот, слід натиснути на верхівку інгалятора, затримати дихання на 5 секунд.
C. ЛЗ застосовують на вдиху, після чого негайно слід зробити видих.
D. Роблячи видих через рот, слід натиснути на верхівку інгалятора, запити великою кількістю води. E.
ЛЗ застосовують при затримці дихання, після чого слід прополокати ротову порожнину.

У хворої 45 років, яка скаржилась на схуднення, болі в животі, стілець з домішкою крові, після
обстеження діагностовано неспецифiчний виразковий коліт. В основі комплексу лікування цього
захворювання:
A. Сульфосалазин або салазопiридазин
B. Цитостатики та симптоматична терапiя
C. Вiтамiни та мiсцева репаративна терапiя
D. Гемотрансфузiї та бiлковi препарати
E. Глюкокортикостероїди та салазопiридазин

Оберіть лікарський засіб групи неглікозидних кардіотонічних засобів, що застосовується для


короткотривалої парентеральної фармакотерапії гострої серцевої недостатності: A.
Клопідогрель.
B. Мілринон.
C. Дигоксин.
D. Епінефрин. E. Корглікон.

Під час вагітності при лікуванні гострого гестаційного пієлонефриту абсолютно протипоказані: A.
Кларитроміцин, пеніцилін
B. Цефазолін, ципринол
C. Норфлоксацин, ципрофлоксацин
D. Стрептоміцин, тетрациклін
E. Бісептол, нітроксолін

У хворого 49 років, загострення пептичної виразки 12-палої кишки з супутнім антральним


гастритом. Доведена наявність хелікобактерної інфекції. Під час комплексної фармакотерапії
хворий поскаржився на появу калу чорного кольору. Який із призначених ЛЗ змінив колір калових
мас?
A. Тетрациклін
B. Вісмуту субцитрат
C. Фуразолідон
D. Амоксицилін
E. Метронідазол

Коли слiд вживати холiнолiтичнi лікарські засоби?


A. Через 1 - 2 години пiсля приймання їжi
B. За 30 хвилин до приймання їжi
C. В часи приймання їжi
D. Через 30 хвилин пiсля приймання їжi
E. Тiльки на нiч

Етіологічними причинами синдрому Іценко-Кушинга є всі перелічені, окрім:


A. АКТГ-продукуюча пухлина
B. Порушення периферичного метаболізму стероїдів
C. Тривале лікування глюкокортикоїдами
D. Аденома наднирників
E. Надлишкова секреція АКТГ в аденогіпофізі

Про який вид фармакотерапії йдеться, коли хворому з туберкульозом легень лікар призначає
лібексин?
A. Симптоматичне лікування.
B. Етіотропне лікування.
C. Базове лікування.
D. Патогенетичне лікування.
E. Радикальне лікування.

До лікарських засобів, що виявляють короткотривалий позитивний інотропний ефект належать: A.


Добутамін; мілринон.
B. Фуросемід; допамін.
C. Дилтіазем; допамін.
D. Торасемід; мілринон.
E. Ніфедипін; фуросемід.

Резистентність – це
A. Незвичайна реакція на введення речовини.
B. Накопичення речовини в організмі.
C. Підвищення чутливості до ЛЗ при повторних введеннях.
D. Стійкість організму до дії певних ЛЗ.
E. Швидке звикання.

Який зі способів введення лікарських засобів не належить до парентерального?


A. Внутріартеріальний
B. Дом’язовий
C. Довенний
D. Інтратекальний
E. Підшкірний
Визначте період напіввиведення оксациліну, якщо відомо, що його константа елімінації становить
1,386 год.-1?
A. 0,8 год.
B. 30 хв.
C. 0,6 год.
D. 15 хв.
E. 3 хв.

У пацієнта Л. 3 р. діагностовано сфеноїдит. Із запропонованих антибактерійних ЛЗ оберіть


найбезпечніший у цій клінічній ситуації.
A. Спарфлоксацин.
B. Хлорамфенікол.
C. Цефотаксим.
D. Геміфлоксацин.
E. Тетрациклін.

Яка з наведених анемій спричинена прискореним руйнуванням еритроцитів?


A. гемолітична анемія
B. В12-дефіцитна анемія
C. залізодефіцитна анемія
D. фолієводефіцитна анемія
E. гостра постгеморагічна анемія

Хвора 32 років перебуває на стаціонарному лікуванні з приводу гострого калькульозного


холециститу. Які з перелічених ЛЗ протипоказані в даному випадку?
A. ЛЗ, до складу якого входить жовч
B. Ципрофлоксацин
C. Ранітидин
D. Метамізол натрію
E. Папаверин

У якого із перелічених нестероїдних протизапальних засобів найслабше проявляється


ульцерогенна дія?
A. мелоксикам
B. диклофенак
C. ацетилсаліцилова кислота
D. піроксикам
E. індометацин

Якими чинниками зумовлений розвиток мегалобластних анемій?


A. прискореним руйнуванням еритроцитів
B. недостатністю заліза у крові
C. пригніченням синтезу в нирках еритропоетину
D. недостатністю фолієвої кислоти або вітаміну В12
E. токсичним пригніченням кісткового мозку

Назвіть "золотий стандарт" цефалоспоринів І покоління.


A. Цефіксим.
B. Цефазолін.
C. Цефадроксил.
D. Цефуроксим.
E. Цефалексин.

Вкажіть, які фармакокінетичні процеси характерні для довенного введення лікарських засобів:
A. абсорбції, розподілу, елімінації
B. абсорбції та елімінації
C. абсорбції та метаболізму
D. абсорбції та розподілу
E. розподілу, метаболізму та елімінації

Яка із перелічених ознак не стосується ревматоїдного поліартриту?


A. навколосуглобовий остеопороз
B. ранкова скутість (не менше 1 год.)
C. наявність підшкірних вузликів
D. наявність симетричного ураження суглобів
E. ушкодження сонячного сплетіння

Який із зазначених факторів не впливає на рівень концентрації лікарського засобу в організмі


хворого?
A. взаємодія з іншими ліками
B. шлях введення
C. період доби
D. лікарська форма
E. кратність дозування

Найбільш ефективний метод усунення азотемії при гострій нирковій недостатності:


A. Вуглеводно-жирова дієта
B. Застосування фуросеміду
C. Гемодіаліз
D. Обмеження білків у раціоні
E. Прийом фітолізину

Для обструктивного бронхіту не характерно:


A. Затрудення видиху. B. Задишка
C. Кашель.
D. Гостра дихальна недостатність. E. Кров’янисте харкотиння.
Почервоніння та набряк слизової оболонки мигдаликів без утворення нальотів, незначне
збільшення та болісність регіонарних лімфовузлів є клінічними симптомами такого захворювання
як:
A. Виразково-некротична ангіна.
B. Катаральна ангіна.
C. Флегмонозна ангіна.
D. Лакунарна ангіна.
E. Фолікулярна ангіна.

Який із наведених ЛЗ належить до неінгаляційних анестетиків для проведення загальної анестезії?


A. тіопентал натрію
B. ксенон
C. закис азоту
D. диетиловий ефір
E. севофлуран

Які з наведених засобів можуть послаблювати терапевтичну дію ЛЗ заліза? A.


глюкоза
B. метіонін
C. алкоголь
D. аскорбінова кислота
E. Тетрациклін

До пробіотиків, які застосовуються при лікуванні і профілактиці дисбактеріозу НЕ належить:


A. Лінекс
B. Бактрим
C. Бактисуптил
D. Хілак форте
E. Біфілонг

У хворої Г., 54 р. на фоні ревматоїдного артриту розвинулась виразка 12-ти палої кишки. Який НПЗЗ
із перелічених рекомендовано призначити для лікування основного захворювання? A. піроксикам
B. мелоксикам
C. Індометацин
D. диклофенак
E. ацетамінофен

ЛЗ якої фармакотерапевтичної групу будуть застосовані в якості «швидкої допомоги» для зняття
нападу гострого бронхоспазму?
A. Системні глюкокортикостероїди.
B. β2-агоністів короткої дії.
C. Блокатори лейкотрієнових рецепторів.
D. Інгаляційні глюкокортикостероїди.
E. М-холінолітики.

Неврологічна симптоматика характерна для ревматизму:


A. поліневрит
B. тромбоішемічний інсульт
C. хорея
D. психоз
E. енцефалопатія
Серед факторів, що призводять до виникнення анемії оберіть нехарактерний:
A. крововтрата
B. недостатнє продукування еритроцитів
C. надмірний вміст заліза в дієті
D. менструація
E. деструкція форменних елементів крові

Гострий тонзиліт – це
A. Запалення слизової оболонки гортані.
B. Запалення м'яких тканин зовнішнього слухового проходу.
C. Інфекційно-запальне ураження лімфоїдної тканини горла, переважно мигдаликів.
D. Запалення слизової оболонки гортані та трахеї.
E. Запалення слизової оболонки глотки.

Хророму Т., для лікування крупозної пневмонії призначено цефтріаксон, через 7 днів виникли
загальна слабість, здуття живота, періодичні проноси, які змінюються закрепами. Який з наведених
ЛЗ слід рекомендувати для корекції даного стану?

A. Пікосульфат натрію.
B. Біфідо- та лактобактерії.
C. Домперидон.
D. Лоперамід.
E. Сироп крушини.

Хворому з пептичною виразкою шлунка призначено фамотидин. Кислотність шлункового соку


значно знизилась. Який механізм лежить в основі дії даного лікарського засобу?
A. Пригнічення активності Н+К+АТФ-ази
B. Блокада М-холінорецепторів
C. Блокада Н1-гістамінових рецепторів
D. Блокада N-холінорецепторів симпатичних гангліїв
E. Блокада Н2-гістамінових рецепторів

Специфічні шкірні прояви ревматизму це:


A. петехіальні висипи
B. еритема кільцеподібна
C. пустульозна висипка
D. вузлики Гебердена
E. телеангіектазії

У юнака 16 років після перенесеної ангіни виникли набряки на обличчі, тупий біль у попереку,
кров’яниста сеча. В аналізі сечі – виражена протеїнурія, еритроцити на все поле зору, циліндри. Яка
найбільш вірогідна патологія, що зумовлює таку клініку?
A. Туберкульоз нирок
B. Гострий гломерулонефрит
C. Радикуліт
D. Цистит
E. Гострий пієлонефрит

Дайте визначення поняттю «вторинна профілактика ревматизму».


A. введення після перенесеної гострої ревматичної гарячки бензатину бензилпеніциліну щомісячно
впродовж індивідуально визначеного терміну
B. введення після перенесеного гострого стрептококового тонзиліту бензатину бензилпеніциліну,
одноразово
C. введення бензатину бензилпеніциліну впродовж 10 днів хворому на ревматичну ваду серця
D. введення після перенесеного гострого тонзиліту бензатину бензилпеніциліну щомісячно впродовж
індивідуально визначеного терміну
E. введення після перенесеної гострої ревматичної гарячки бензилпеніциліну у терапевтичній дозі
впродовж 10 днів

Назвіть лікарський засіб, який крім антацидного ефекту, проявляє ще адсорбуючу та в'яжучу дію.
A. Плантаглюцид
B. Магнію окис
C. Натрію гідрокарбонат
D. Магнію карбонат
E. Алюмінію гідроокис

Молода жiнка часто прикидається вночi через виникнення болю в епiгастрiї. Має печію, розлади
стільця. Біль зменшується після вживання їжі. Дратівлива. Який найбiльш вiрний дiагноз? A.
Езофагіт
B. Рак шлунку
C. Пептична виразка 12-палої кишки
D. Хвороба Крона
E. Холангiт

Діагноз (розпізнавання хвороби) –


A. Результати обстеження хворого.
B. Коротке заключення лікаря про суть захворювання та стан пацієнта.
C. Правильної відповіді немає.
D. Коротке заключення пацієнта про стан хвороби.
E. Заключення лікаря та медсестри про суть захворювання та стан пацієнта.

Тривалість антибактерійної терапії неускладненої позашпитальної пневмонії складає в середньому:


A. 28 днів.
B. 14 днів.
C. 21 день.
D. 3-5 днів. E. 7 днів.

Хворому 43 років з артеріальною гіпертензією призначено комбінований лікарський засіб ліпразид


по 1табл. 2 рази на день. Однак хворий вживає його лише при виникненні болю голови.
Відмічаються часті гіпертензивні кризи. Яка причина неефективності фармакотерапії?
A. Відсутність комплаєнсу.
B. Недостатня дія призначеного ЛЗ.
C. Синдром відміни.
D. Виникнення побічної дії ЛЗ.
E. Необхідність призначення етіотропної фармакотерапії.

Причиною хвороби Іценко-Кушинга є:


A. Пухлина кори наднирників
B. Гіпопродукція АКТГ в гіпофізі
C. Дефіцит продукції гормонів кори наднирників
D. Гіпокортезолемія
E. Гіперпродукція АКТГ в аденогіпофізі
Оберіть групу ЛЗ, що не належить до засобів першого ряду для фармакотерапії хронічної серцевої
недостатності?
A. Серцеві глікозиди.
B. Блокатори рецепторів ангіотензину ІІ.
C. Тіазидні діуретики.
D. Інгібітори АПФ.
E. Блокатори β-адренорецепторів.

Дайте визначення поняттю «Мікрогематурія».


A. Кров в сечовому міхурі і сечоводі
B. Наявність згустків крові в сечі
C. Видима кров в сечі
D. Червоне забарвлення сечі
E. Еритроцити в сечі при її мікроскопії

Який із симптомів найбільш характерний для анемії?


A. діарея
B. задишка
C. почервоніння обличчя
D. підвищений апетит
E. слабкість та блідість шкіри

Для знеболення при гострому панкреатиті рекомендовано застосовувати:


A. Урсодезоксихолеву кислоту
B. Індометацин
C. Морфін
D. Тримеперидин (промедол)
E. Панкреатин

У хворого, який тривало приймає інгаляційний ЛЗ з приводу лікування бронхіальної астми,


виникли прояви кандидозу ротової порожнини. ЛЗ якої групи міг призвести до такого ускладнення
фармакотерапії?
A. Інгаляційні глюкокортикоїди.
B. β2-агоністи короткої дії.
C. Системні глюкокортикоїди.
D. Неселективні холінолітики.
E. Селективні холіноблокатори.

Порекомендуйте пацієнту з ангіною жарознижуючий ЛЗ, який проявляє імуностимулюючу


активність.
A. Ібупрофен.
B. Мелоксикам.
C. Індометацин.
D. Мефенамінову кислоту.
E. Метамізол натрію.

Які з наведених продуктів можуть послаблювати терапевтичну дію ЛЗ заліза?


A. аскорбінова кислота
B. сорбіт
C. апельсиновий сік
D. фрукти
E. кава, чай
До аптеки звернувся хворий, у якого після курсу антибіотикотерапії розвинувся дизбактеріоз. Який
лікарський засіб доцільно йому призначити?
A. метоклопрамід
B. ципрофлоксацин
C. лоперамід
D. бісакоділ
E. ЛЗ, що містить лакто- та біфідобактерії

Характерними клінічними ознаками хвороби Іценко-Кушинга є:


A. Остеопороз у жінок, зменшення кісткової маси у чоловіків
B. Гіперхолестеринемія
C. Ожиріння тулуба, стрії на шкірі, артеріальна гіпертензія
D. Гіперпігментація шкіри та слизових оболонок
E. Гіпертрихоз, гінекомастія

Серед вказаних чинників, оберіть ті, які впливають на біодоступність лікарського засобу:
A. супутні захворювання
B. функціональний стан нирок
C. значення абсолютної біодоступності
D. швидкість локального кровоплину в окремого хворого E. стать

Назвіть клінічні ознаки системного червоного вовчака.


A. кісетоподібний рот, амімічне обличчя, мікростомія, остеоліз кінцевих фаланг пальців кистей рук
B. ексудативна еритема на щоках, вісцерити, люпус-нефрит, люпус-кардит
C. ксеростомія, ксерофтальмія, хронічний поліартрит
D. деструкція міжсуглобового хряща, крепітація при рухах, коксартроз, суглобові «миші» E. кіфоз
грудного відділу хребта, самовільні переломи кісток, зменшення росту людини

Для розрідження в’язкого мокротиння лікар призначив ЛЗ з групи прямих муколітиків. Оберіть цей
ЛЗ.
A. Амброксол.
B. Карбоцистеїн.
C. Теофілін.
D. Ацетилцистеїн. E. Окселадин.

Хворий Д., 35 р., який нещодавно переніс ангіну, відмітив серцебиття, припухлість і біль колінних
та ліктьових суглобів. При обстеженні у пацієнта виявлено ваду мітрального клапана. Визначте
попередній діагноз.
A. системна склеродермія
B. ревматизм
C. системний червоний вовчак
D. вузликовий периартериїт
E. ревматоїдний артрит
Курс хіміотерапії для лікування туберкульозу передбачає:
A. Застосування 3-х протитуберкульозних ЛЗ протягом не менше 6-ти місяців.
B. Застосування 3-х протитуберкульозних ЛЗ протягом 3-х місяців.
C. Застосування 1-го протитуберкульозного ЛЗ протягом не менше 1-го року.
D. Застосування 1-го протитуберкульозного ЛЗ протягом не менше 6-ти місяців. E. Застосування 2-х
протитуберкульозних ЛЗ протягом не менше 6-ти місяців.
Після курсу антибактерійної терапії у дитини 2 р. з’явились сіре забарвлення шкіри, диспептичні
явища, гіпотермія. Який ЛЗ міг викликати такі побічні явища?
A. кларитроміцин
B. тетрациклін
C. хлорамфенікол
D. рифампіцин E. доксициклін

Вагітна жінка захворіла на флегмонозну форму ангіни. Який з нижченазваних антибактеріальних


засобів доцільно їй призначити:
A. Гентаміцину сульфат.
B. Тетрацикліну гідрохлорид.
C. Офлоксацин.
D. Левоміцетин.
E. Цефотаксим.

Розрахувати константу елімінації триметазидину, якщо відомо, що його період напіввиведення


становить 4,5 год.
A. 0,237 год-1
B. 0,248 год-1
C. 0,102 год-1
D. 0,214 год-1
E. 0,154 год-1

Фармакотерапію ранньої стадії хвороби Паркінсона у пацієнтів старше 75 років починають із


призначення наступного лікарського засобу:
A. пірибедил
B. леводопа
C. бромокриптин
D. селегілін
E. праміпексол

До лікаря звернувся хворий зі скаргами на оперізуючий біль в епігастрії, слабість. Зловживає


алкоголем. В анамнезі – хронічний панкреатит. Який найбiльш цiнний лабораторний показник в
дiагностицi загострення хронiчного панкреатиту?
A. Рiвень трансамiназ кровi
B. Гiперглiкемiя
C. Рiвень амiлази кровi
D. Лейкоцитоз
E. Рiвень лужної фосфатази
При лікуванні гострого пієлонефриту серед лікарських засобів на першому місці:
A. Нефропротектори
B. Сечогінні
C. Антибіотики
D. Урикозуричні
E. Спазмолітики

Бета-гемолітичний стрептокок є етіологічним чинником всіх перелічених захворювань, окрім:


A. гострий гломерулонефрит
B. хронічний пієлонефрит
C. ревматизм
D. скарлатина
E. гострий тонзиліт
Чоловіка 62-х років після фізичного навантаження почав турбувати біль за грудиною з іррадіацією в
ліву руку. Який лікарський засіб слід рекомендувати для його усунення?
A. Дилтіазем.
B. Нітрогліцерин.
C. Ізосорбіду динітрат.
D. Ізосорбіду мононітрат.
E. Дротаверин.

Який із перерахованих ЛЗ може маскувати ознаки кровотечі у верхніх відділах


шлунковокишкового тракту (забарвлювати стілець в чорний колір)?
A. Пірензепін
B. Сукральфат
C. Алюмінію гідроксид+магнію гідроксид
D. Вісмуту субцитрат
E. Фамотидин

Процес підвищення біологічної доступності пов’язаний із:


A. модифікацією процесу розподілу
B. сповільнення процесу елімінації
C. усі відповіді правильні ???
D. модифікацією процесу абсорбції
E. сповільнення процесу біотрансформації

Який рівень артеріального тиску є цільовим при лікуванні АГ у хворих похилого віку? A.
не вище 220/150.
B. < 120/80.
C. на 20% нижче від початкового рівня.
D. не нижче 220/160. E. < 140/90.

Ураження шкіри у вигляді еритематозних висипань на спинці носа і щоках є проявом:


A. системного червоного вовчака
B. гострої ревматичної гарячки
C. подагричного артриту,
D. хронічної ревматичної хвороби
E. системної склеродермії

До лікаря звернулась жінка зі скаргами на відчуття важкості в правому підребер’ї, нудоту. В


анамнезі – холецистит, гастрит, вада серця. При огляді – істеричність склер, на шкірі – судинні
«зірочки». Що є вирішальним у встановлені діагнозу хронічного гепатиту?
A. Перенесений вiрусний гепатит
B. Виявлення в сироватцi кровi австралiйського антигену
C. Данi гiстологiчного дослiдження печiнки
D. Виявлення в сироватцi кровi альфа-фетопротеїну
E. Перiодичний субфебрилiтет, iктеричнiсть, болi в правому пiдребер'ї

Яка з наведених анемій спричинена прискореним руйнуванням еритроцитів?


A. гостра постгеморагічна анемія
B. В12-дефіцитна анемія
C. фолієводефіцитна анемія
D. гемолітична анемія ?
E. залізодефіцитна анемія
Виберіть лікарський засіб, який не обов’язково включати у схему фармакотерапії гострої фази
нешпитальної пневмонії:
A. Амоксиклав
B. Ністатин
C. Цефтріаксон
D. Аугментин
E. Дуовіт

До симптомів цукрового діабету належать всі перелічені, окрім:


A. Поліурія
B. Погіршення зору
C. Відчуття затерпання кінцівок
D. Спрага
E. Тремор повік

Антидотом при передозуванні гепарину є: A.


Протаміну сульфат.
B. Пентоксифілін.
C. Апротинін.
D. Менадіон (вікасол).
E. Унітіол.

Який з наведених ЛЗ використовують для гігієни носових ходів? A.


Ксилометазолін.
B. Засоби морської води.
C. Розчин магнію сульфату.
D. Оксиметазолін.
E. Цефазолін.

Біотрансформація ЛЗ в основному відбувається в:


A. Крові.
B. Кишківнику.
C. Печінці.
D. Легенях. E. Нирках.

В комплексі лікування гломерулонефриту не застосовують:


A. Імунодепресанти
B. Інгібітори АПФ
C. Антикоагулянти
D. Антигістамінні
E. Антибіотики

До ускладнень стрептококової ангіни належать усі перелічені, крім: A.


Отит.
B. Гіпертиреоз.
C. Ревматизм.
D. Ревматоїдний артрит. E. Гайморит.

Дитина проковтнула невідому кількість таблеток заліза сульфату. Який засіб слід застосувати при
розвитку симптомів отруєння цим ЛЗ:
A. дефероксамін
B. протаміну сульфат
C. метиленовий синій
D. налоксон
E. унітіол

При відпуску з аптеки нітрогліцерину провізор попередив хворого про найбільш часту побічну
реакцію при застосуванні цього лікарського засобу:
A. Гіпертензія.
B. Біль голови.
C. Гіперглікемія.
D. Дизурія.
E. Сухість у роті.

Для клінічних проявів гастриту типу В характерно все, окрім:


A. Підвищеної втомлюваності, дратівливості
B. «Голодного» болю в епігастрії
C. Інфікування Helіcobacter рylorі
D. Підвищення секреторної функції шлунка
E. Зниження секреторної функції шлунка

Найефективнішим методом діагностики епілепсії є


A. електроенцефалографія
B. комп’ютерна томографія
C. рентгенографія
D. МРТ
E. ангіографія

Хворому В., 34 років, встановлено діагноз гострий пієлонефрит. Яка група лікарських засобів у
лікуванні є засобами вибору: A. Антибіотики.
B. Сечогінні.
C. Спазмолітики.
D. Урикозуричні засоби. E. Нефропротектори.

Для гострого панкреатиту характерно:


A. Нудота, блювання, пронос, інтенсивний біль «оперізуючого» характеру
B. Схуднення, біль ниючого характеру, закрепи, метеоризм
C. Біль в лівій половині живота, зниження маси тіла
D. Анорексія, диспепсія, кахексія
E. Біль ниючого характеру, диспепсія

Виберіть симптом характерний для ревматоїдного артриту.


A. постійна гарячка
B. ранкова скутість суглобів
C. гепатомегалія
D. набряк та болючість гомілок
E. підвищення рівня креатиніну в сечі

У хворого з підвищеною бронхіальною секрецією та значним відходженням мокроти протикашлеві


засоби:
A. Призначаються лише на ніч.
B. Не призначають.
C. Протипоказані особам із супутньою патологією печінки.
D. Є обов’язковим компонентом фармакотерапії. E. Не призначають дітям до 3 років.

Хворому призначили антибіотик четвертого покоління цефалоспоринів, який добре проникає крізь
мембрани грамнегативних бактерій, забезпечує стійкість відносно лактамаз і має широкий спектр
антибактеріальної дії. Який засіб лікар призначив хворому.
A. цефтріаксон
B. цефепім
C. ципрофлоксацин
D. цефуроксим
E. цефазолін

При застосуванні якого з нижчеперелічених гіпотензивних лікарських засобів може виникнути


виражена брадикардія?
A. Еналаприл
B. Віскен
C. Доксазозин
D. Ніфедипін
E. Метопролол

Молода жінка знепритомніла в тісному приміщенні. Щоб привести її до тями лікар дав понюхати
розчин аміаку. Яка точка прикладання фармакотерапії цим лікарським засобом?
A. Етіотропна дія.
B. Симптоматична дія.
C. Місцева дія.
D. Рефлекторна дія.
E. Патогенетична дія.

В аптеку звернувся чоловік 30 років з захворюванням печінки і алергією на ЛЗ виготовлені з


росторопші плямистої. Який ЛЗ з групи гепатопротекторів йому доцільно призначити? A.
Дарсил
B. Силібор
C. Ессенціале
D. Карсил
E. Гепабене

Хворий 54 років хворіє на ішемічну хворобу серця. Який з перелічених лікарських засобів володіє
одночасно коронаророзширюючою і антиагрегантною діями?
A. Аспірин
B. Дипіридамол ???
C. Нітросорбід
D. Лізиноприл
E. Ніфедипін

Перкусія – це метод обстеження хворого, що ґрунтується на:


A. Просвічуванні окремих частин тіла рентгенівськими променями.
B. Оцінюванні звуку отриманого при вистукуванні поверхні тіла.
C. Дослідженні слизової оболонки порожнистих органів за допомогою ендоскопу.
D. Вислуховуванні звукових явищ внутрішніх органів.
E. Дотику та промацуванні поверхневих тканин і органів, що знаходяться глибше.
До Вас звернулася хвора на хронічний панкреатит з вираженими проявами недостатності
зовнішньо-секреторної функції підшлункової залози: метеоризмом, важкістю в животі після їжі,
нудотою тощо. Який з наведених ЛЗ найдоцільніше призначати в даному випадку? A.
Панкреатин
B. Домперидон
C. Вісмуту субцитрат колоїдний
D. Омепразол
E. Дротаверин

Поясніть молодому колезі, що є патогенетичною ланкою розвитку хвороби Паркінсона?


A. руйнування допаміну в корі головного мозку
B. нечутливість рецепторів до допаміну
C. прогресуюча дегенерація допамінергічних нейронів
D. відсутність синтезу допаміну
E. надмірне продукування допаміну в корі головного мозку

Діагностичними критеріями акромегалії є:


A. Антропометричний контроль, підвищена концентрація соматостатину
B. Характерна зовнішність, підвищена концентрація соматотропіну
C. Результати комп’ютерної томографії, знижена концентрація соматотропіну
D. Результати рентгенологічного обстеження, підвищена концентрація соматостатину E. Результати
дослідження очного дна, знижена концентрація соматотропіну

Що таке епілептичний статус?


A. епілептичний напад, який триває більше 30 хвилин або повторні епілептичні напади, між якими
у хворого свідомість не відновлюється
B. епілептичний напад, що супроводжується внутрішньомозковим крововиливом
C. епілептичний напад без втрати свідомості
D. неврологічний синдром, що характеризується сповільненням рухів, ригідністю м'язів, тремором
спокою і порушенням пізніх рефлексів
E. гостра втрата кровопостачання в судинних басейнах головного мозку, яка призводить до ішемії та
випадіння неврологічних функцій в уражених ділянках

У хворої 62 р. на фоні гіпертензивного кризу (АТ – 200/110 мм рт. ст., Ps – 100 ударів за хвилину)
виникли судоми. Який гіпотензивний засіб з перелічених є ЛЗ вибору? A. Ніфедипін
сублінгвально.
B. Нітрогліцерин судлінгвально або довенно.
C. Каптоприл перорально.
D. Неофілін довенно.
E. Магнію сульфат довенно. ???

Вказати до якого синдрому при туберкульозі відносяться симптоми: підвищення температури,


слабкість, пітливість, похудіння, поганий сон:
A. Абстинентного
B. Обструктивного
C. Анемічного
D. Бронхо-легенево-плеврального
E. Інтоксикаційного

Які зміни в сечі дозволяють відрізнити гломерулонефрит від пієлонефриту?


A. Колір сечі
B. Вираженість протеїнурії
C. Еритроцитурія
D. Глюкозурія
E.Оксалатурія

Який з типів побічних дій лікарських засобів зустрічається найчастіше?


A. Побічна дія, обумовлена раптовим припиненням приймання ЛЗ.
B. Побічна дія при ненавмисному введенні ЛЗ.
C. Побічна дія, обумовлена абсолютним чи відносним передозуванням ліків.
D. Побічна дія, обумовлена специфічними фармакологічними властивостями ліків.
E. Побічна дія, обумовлена підвищеною чутливістю організму до ліків.

Хворому 46 років із загостренням пептичної виразки 12-палої кишки лікарем в комплексі терапії
призначено алмагель. Визначте раціональну схему його застосування?
A. Через 1 і 3 години після кожного приймання їжі і перед сном
B. Безпосередньо після кожного прийому їжі і перед сном
C. В час їди і перед сном
D. Безпосередньо перед кожним вживанням їжі і перед сном
E. На протязі дня при болю и перед сном

В комплекс лікування неускладненого туберкульозу легень включають: A.


Муколітики.
B. Гастропротектори.
C. Антисептики.
D. Протикашлеві.
E. Вітаміни групи В.

В І триместрі вагітності при розвитку пієлонефриту показана лише одна група антибактеріальних
засобів: A. Фторхінолони
B. Пеніциліни
C. Аміноглікозиди
D. Сульфаніламіди
E. Цефалоспорини

Пацієнтка 33 р. скаржиться на значну м'язову слабкість, ранкову скутість в суглобах, яка триває
декілька год. При огляді – деформація кистей («плавники моржа»). Які процеси лежать в основі
розвитку даного захворювання?
A. аутоімунне, імунокомплексне ураження сполучної тканини
B. вимивання кальцію з кісток і зменшення їх маси
C. інфікування стафілококом і розвиток запалення
D. Порушення обміну уратів і відкладання їх у наколосуглобові тканини E. дегенерація хряща і
структурні зміни субхондральної кістки

Що в першу чергу необхідно зменшити в дієті у хворого з хронічною нирковою недостатністю? A.


Вуглеводи
B. Жири
C. Білки
D. Кількість рідини
E. Кухонну сіль

При непереносимості пацієнтом бета-лактамних антибіотиків, для лікування лакунарної ангіни


альтернативним є застосування:
A. Мідекаміцину. ???
B. Цефуроксиму.
C. Амоксициліну.
D. Бензилпеніцилін.
E. Цефазоліну.

Апластична анемія – це захворювання, що характеризується


A. підвищеною деструкцією форменних елементів крові
B. втратою здатності кісткового мозку виробляти достатню кількість еритроцитів
C. недостатнім поступленням заліза із харчовими продуктами
D. втратою форменних елементів внаслідок кровотечі
E. неможливістю засвоювати вітамін В12

До лікарських засобів, що виявляють короткотривалий позитивний інотропний ефект належать:


A. Ніфедипін; фуросемід.
B. Фуросемід; допамін.
C. Добутамін; мілринон.
D. Дилтіазем; допамін.
E. Торасемід; мілринон.

Хворій А., 72 р., призначено серцеві глікозиди з приводу прогресування серцевої недостатності.
Зміна яких показників клінічного статусу хворої свідчить про терапевтичну ефективність? A.
Збільшення розмірів печінки.
B. Зменшення ЧСС, набряків, задишки.
C. Підвищення артеріального тиску.
D. Поява брадикардії, аритмії. E. Зменшення діурезу.

До найчастіших причин розвитку хронічного гепатиту належить:


A. Вживання ліків
B. Радіація
C. Алкоголь
D. Перенесена вірусна інфекція
E. Контакт з токсичними речовинами

Лікування, що призначається хворому з метою впливу на причину захворювання A.


Симптоматичне.
B. Паліативне.
C. Етіотропне.
D. Комплексне.
E. Патогенетичне.

Оберіть протитуберкульозний ЛЗ першого вибору при лікочутливому туберкульозі. A.


Канаміцин.
B. Рифабутин.
C. ПАСК.
D. Циклосерин.
E. Рифампіцин.

У хворого на цукровий діабет 2 типу ниркова недостатність, кліренс креатиніну менше 60 мл/хв,
який пероральний цукрознижуючих засіб доцільно обирати?
A. Піоглітазон
B. Глімепірид
C. Гліквідон
D. Метформін
E. Натеглінд
У хворого діагностовано хронічний гепатит з мінімальним ступенем активності. Які лікарські засоби
не слід застосовувати при цьому захворюванні?
A. Антиоксиданти
B. Жовчегiнні
C. Вiтамiни ??
D. Спазмолiтики
E. Глюкокортикоїди

Вкажіть, який шлях введення дозволяє найшвидше досягнути терапевтичного ефекту? A.


ректальний
B. довенний
C. дом’язовий
D. пероральний
E. підшкірний

Хвора А., протягом 7 днів отримувала антибіотикотерапію, яка ускладнилась розвитком


дисбактеріозу кишечника. Яку із груп лікарських засобів слід використати для корекції цього
ускладнення?
A. Адаптогени.
B. Спазмолітики.
C. Антациди.
D. Жовчогінні.
E. Пробіотики

Після переохолодження у хворої Д, 60 років виникли часте і болісне сечовиділення, позиви на


сечовипускання та інші прояви дизурії, що супроводжуються помутнінням сечі. Температура тіла –
37,0 С. При УЗД нирок - патології не виявлено. Яке захворювання можна запідозрити у хворої?
A. Сечокам’яна хвороба. Ниркова коліка.
B. Гострий пієлонефрит
C. Гострий гломерулонефрит
D. Гострий цистит
E. Радикуліт

Назвіть симптом, не характерний для гіпертензивного кризу.


A. Блювання.
B. Порушення зору.
C. Миготіння «мушок» перед очима.
D. Часте сечовиділення.
E. Тенезми.

Діагностичним критерієм при захворюваннях щитоподібної залози вважають:


А. Рівень ТТГ і Т4
В. Рівень Т3
С. Рівень ТТГ та Т3 і Т4
D. Рівень Т3 і Т4
E. Рівень ТТГ

Через який проміжок часу після перенесеної інфекції може розвинутись гострий
гломерулонефрит?
А. Через 2-3 дні
В. Через 2-3 тижні
С. Через півроку
D. Через 1 рік
E. Через 2-3 місяці

Для бронхіальної астми не характерні наступні симптоми:


А. Еозинофілія
В. В’язке харкотиння
С. Сухі хрипи
D. Напади задухи
E. Гарячка

Хворий С. 47 років, ентерально приймає преднізолон у дозі 30 мг. на добу для лікування
бронхіальної астми. Раптово виник біль в епігастральній ділянці. Яке ускладнення гормонотерапії
найбільш ймовірно виникло?
А. Гострий коліт
В. Гострий холецистит
С. Інфаркт міокарда
D. Пептична виразка шлунка
E. Гострий панкреатит

Сфеноїдит-це…
А. Запалення лобвої пазухи носа
В. Запалення середнього вуха
С. Запалення клиновидної пазухи носа
D. Запалення євстахієвої труби
E. Запалення верхньощелепної пазухи носа

У хворої на цукровий діабет після перенесеного гострого бронхіту з’явилася темпертура 39* С,
озноб, болюче сечовипускання, тупий біль в правій поперековій ділянці. Добовий діурез 600 мл.,
лейкоцитоз 12x109/л, ШОЕ 44 мм/год., в сечі-лейкоцити на все поле зору, креатинін в крові, цукор
крові- 4,5 ммоль/л. Яке захворювання розвинулось у хворої? A. Діабетична кома
B. Ниркова недостатність
C. Гострий пієлонефрит
D. Гострий гломерулонефрит
Е. Хронічний бронхіт

Виберіть правильне твердження:


А. Урсодезоксихолеву кислоту приймають вранці натще ?
В. Холелітолітичні ЛЗ ефективні при наявності білірубінових та кальцифікованих конкрементів
С. Пероральний літоліз здійснюють при наявності конкрементів більше 30 мм.
D. Лікування засобами жовчних кислот триває 1-2 місяці
E. Фармакотерапія холелітичними ЛЗ здійснюється від 6 до 24 місяців під контролем УЗД

Абсолютним протипоказанням до проведення тромболітичної терапії тканинним активатором


плазміногену при ішемічному інсульті є
А. рівень тромбоцитів в крові менше 100 000 в 1 мкл
В. вагітність
С. тяжка ЧМТ чи інсульт у попередні 3 місяці
D. глікемія менше 2,75 ммоль/л чи більше 22 ммоль/л
E. судомні напади в дебюті інсульту
Виберіть лікарський засіб, який не належить до протитуберкульозних засобів 2 ряду: А.
Капреоміцин
В. Канаміцин
С. Стрептоміцин
D. Ципрофлоксацин
E. Амікацин

Зниження рівня альбуміну в сироватці може спричинити:


А. зниження концетрації вільного лікарського засобу
В. зміни часу запізнення
С. наростання терапевтичного ефекту
D. подовження періоду напіввиведення
E. зростання концетрації вільного лікарського засобу

Хворий 65 років, з нападом стенокардії поступив у терапевтичне відділення. Через 48 годин у


нього розвинулось госпітальна пневмонія, була призначена антибактерійна фармакотерапія. Через
3 доби хворий почав втрачати слухи. Назвіть ЛЗ, який міг спричинити глухоту:
А. Фузафангін
В. Мідекаміцин
С. Левофлоксацин
D. Амікацин
E. Цефтріаксон

Хворий на артеріальну гіпертензію тривалий час приймав пропранолол, який раптово припинив
вживати. Стан хворого погіршився,розвинувся гіпертензивний криз. До якої категорії належить ця
побічна дія? А. Cиндром відміни
В. Кумуляція
С. Залежність
D. Толерантність
E. Сенсибілізація

Оберіть правильний принцип фармакотерапії протиепілептичними лікарськими засобами:


А. симптоматичне лікування частих нападів
В. комбінована терапія двома і більше ЛЗ
С. монотерапія
D. етіотропна терапія
Е. пульс-терапія високими дозами

В аптеку звернувся хворий 28 років, який страждає на пептичну виразку 12-палої кишки;
секреторна функція різко підвищена. Який з наведених ЛЗ бажано призначити у цьому випадку
А. Діосмектит
В. Алюмінію гідроксид + Магнію гідроксид
С. Омепразол
D. Атропіну сульфат
E. Метоклопрамід

У хворого на гіпертонічну хворобу при систематичному лікуванні антигіпертинзивним препаратом


виник сухий кашель. Який з названих лікарських засобів може бути причиною даного небажаного
ефекту?
А. Лозартан
В. Амлодипін
С. Хлорталідон
D. Еналаприл
E. Карведилол

Про який вид фармакотерапії йдеться, коли хворому з ревматизмом лікар призначає пеніцілін?
А. Етіотропне лікування
В. Симптоматичне лікування
С. Базове лікування
D. Патогенетичне лікування
Е. Замісне лікування

Оберіть лікарський засіб антидот при передозуванні варфарину?


А. тіамін
В. вітамін К
С. гепарин
D. етамзилат натрію
E. протаміну сульфат

Хворій на ангіну дитині призначено комбінований засіб,який містить в собі сульфаніламід та


триметоприм. Який ЛЗ призначено?
А. Cульфален
В. Сульфаметаксазол
С. Бісептол
D. Сульфадиметоксин
E. Ентеросептол

Вкажіть нітрат-лікарський засіб тривалої дії, що застосовується для попередження приступів


стенокардії:
А. Нітрогліцерин
В. Нітро-спрей
С. Ериніт
D. Нітронг-міте
E. Ізосорбіду динітрат

Хворому потрібно призначити синтетичний аналог соматостатину. Який із нижче перелічених


засобів ви можете йому запропонувати?
А. Окреотид
В. Бромокриптин
С. Перитол
D. Меланотропін
E. Пітуїтрин

Назвіть додаткові методи обстеження хворого, необхідні для встановлення діагнозу (оберіть
найбільш повну відповідь)
А. Лабораторне дослідження крові і сечі
В. Інструментальне дослідження
С. Збір скарг і анамнезу, лабораторна діагностика
D. Збір скарг і анамнезу, фізикальне обстеження (пальпація, перкусія, аускультація) E.
Лабораторне і інструментальне дослідження

Лікування залізодефіцитної анемії може передбачати?


А. приймання глюкокортикостероїдів
В. включення в раціон залізовмісних продуктів
С. застосування дефероксаміну
D. зниження споживання залізовмісних продуктів
E. застосування нестероїдних протизапальних засобів

Який з перелічених ЛЗ належить до неселективних агоністів β-адренорецепторів?


A. Сальбутамол
B. Епінефрин
C. Формотерол
D. Орципреналін
E. Іпратропію бромід

Які хратерні ознаки гострого пієлонефриту дозволяють відрізнити його від гострого
гломерулонефриту за аналізом сечі?
A. Глюкозурія
B. Оксалати, урати
C. Бактеріурія, піурія
D. Протеїнурія
E. Слиз, епітелій в сечі

У хворого 31 р. напад ядухи на фоні застосування ЛЗ з групи НПЗЗ. ЛЗ якої групи є засобом вибору у
цьому випадку?
A. Агоніст β2-адренореепторів.
B. Стабілізатор мембран опасистих клітин.
C. Блокатор лейкотрієнових рецепторів
D. Метилксантини.
E. М-холінолітики.

Хронічний гастрит, асоційований з перніціозною анемією, - це:


A. Гастрит типу А
B. Еозинофільний
C. Хімічний
D. Гастрит типу В
E. Гастрит типу С

У хворого М., що лікується з приводу гострої ревматичної гарячки, виявлено гіперчутливість до


пеніциліну. Які з перерахованих груп протимікробних ЛЗ йому показані?

A. сульфаніламіди
B. макроліди
C. тетрацикліни
D. цефалоспорини
E. фторхінолони

Тахіфілаксія – це
A. Накопичення ЛЗ в організмі.
B. Швидке звикання до певних ЛЗ.
C. Незвичайна реакція на введення ЛЗ.
D. Алергійна реакція.
E. Підвищення чутливості до ЛЗ при повторних введеннях.

Які з наведених засобів можуть послаблювати терапевтичну дію ЛЗ заліза?


A. алкоголь
B. глюкоза
C. тетрациклін
D. метіонін
E. аскорбінова кислота

Який шлях введення лікарських засобів є найбільш раціональним та ефективним при лікуванні
хворих на хронічні обструктивні хвороби легень (ХОЗЛ)? A. Ендотрахеальний.
B. Інгаляційний.
C. Парентеральний.
D. Інтраназальний. E. Пероральний.

Пеніциліни, захищені клавулановою кислотою:


A. Застосовуються лише для фармакотерапії захворювань ЛОР-органів.
B. Застосовуються лише як засіб симптоматичної фармакотерапії.
C. Стійкі до дії бета-лактамаз мікроорганізмів.
D. Використовуються для відповідального самолікування. E. Застосовуються не більше 3-х днів.

Ефективність фармакотерапії при ревматизмі визначають усі з перелічених критерії окрім:


A. відсутність прояву побічних реакцій при застосуванні ЛЗ
B. нормалізація показників активності запального процесу
C. стабільність морфофункціональних показників за ЕхоКГ з боку клапанів та камер серця
D. нормалізація титрів стрептококових антитіл
E. відсутність кардіальних та артралгічних синдромів

Пацієнту, в якого з’явилися перші клінічні прояви вітряної віспи, призначили противірусний
лікарський засіб ацикловір. Оберіть вид фармакотерапії, що проводиться:
A. Замісна.
B. Симптоматична.
C. Етіотропна.
D. Профілактична.
E. Патогенетична.
Вопрос 16

Хворий звернувся до лікаря зі скаргами на пронос, переймоподібний біль в животі, нудоту. Після
обстеження діагностовано амебну 1дизентерію. Лікарем призначено тетрациклін. Який вид
фармакотерапії застосовано лікарем у даного хворого?

A. Етіотропна.
B. Базова.
C. Симптоматична.
D. Патогенетична. E. Замісна.

Біологічною доступністю називають:


A. швидкість і ступінь, з якою активна субстанція або її активна частина всмоктується з готової
лікарської форми і піддається ефекту першого проходження
B. швидкість і ступінь, з якою активна субстанція або її активна частина всмоктується з готової
лікарської форми і появляється у малому колі кровообігу
C. швидкість і ступінь, з якою активна субстанція або її активна частина всмоктується з готової
лікарської форми і появляється у великому колі кровообігу
D. швидкість і ступінь, з якою активна субстанція або її активна частина всмоктується з готової
лікарської форми і елімінує з організму
E. правильна відповідь відсутня
Вопрос 18

Який лікарський засіб має одночасно здатність збільшувати діурез і знижувати артеріальний тиск?
A. Клофелін
B. Раунатин
C. Еуфілін
D. Папаверин
E. Платифілін

Оберіть ЛЗ, що надлежить до прямих антикоагулянтів? A.


Ацетилсаліцилова кислота.
B. Дипіридамол.
C. Варфарин.
D. Вікасол.
E. Еноксипарин.

Яка група сечогінних лікарських засобів має найбільш виражений гіпотензивний ефект і
застосовується для постійного контролю артеріального тиску?
A. Осмотичні
B. Всі відповіді правильні
C. Тіазидні
D. Антагоністи альдостерону
E. Петльові

Який лікарський засіб є препаратом вибору у хворих на цукровий діабет 2 типу із ожирінням?
A. Акарбоза
B. Інсулін
C. Розіглітазон
D. Гліклазид
E. Метформін

Хворому з артеріальною гіпертензією було призначено один із антигіпертензивних засобів.


Артеріальний тиск нормалізувався, однак хворого почав турбувати постійний сухий кашель. Який з
перерахованих ЛЗ має таку побічну дію.

A. Периндоприл.
B. Доксазозин.
C. Індапамід.
D. Ніфедипін.
E. Пропранолол.

Для фармакотерапії хронічного бронхіту слід застосовувати антибактерійні ЛЗ: A.


В осінньо-зимовий період.
B. Якщо харкотиння слизисте.
C. Тривало.
D. Не слід застосовувати взагалі.
E. В період виділення гнійного харкотиння.
У хворої П., яка приймала петльовий діуретик з’явились м’язова слабкість, судоми м’язів нижніх
кінцівок, відчуття перебоїв у роботі серця. Назвіть ЛЗ для корекції даного ускладнення. A.
Магнію сульфат.
B. Дефероксамін.
C. Піридоксин та магнію лактат.
D. Калію та магнію аспарагінат.
E. Алюмінію фосфат.

У хворого Д., з підтвердженою УЗД картиною діагнозу жовчокам’яної хвороби, з’явились біль у
правому підребер’ї, жовтушність шкіри та слизових оболонок. Який із наркотичних анальгетиків
протипоказаний для усунення больового синдрому?
A. Фентаніл
B. Промедол
C. Морфін
D. Омнопон
E. Трамадол

Лікарським засобом, що НЕ належить до інгібіторів ГМГ-КоА-редуктази є: A.


Правастатин.
B. Симвастатин.
C. Розувастатин.
D. Ловастатин.
E. Холестирамін.

У хворого К, 53 років, бронхіальна астма та стенокардія напруги. Який лікарський засіб із


перелічених нижче протипоказаний у цьому випадку?
A. Сальбутамол.
B. Преднізолон.
C. Беклометазон.
D. Орципреналін.
E. Амлодипін.

Хворий 52 років, з метою купування нападу стенокардії прийняв таблетку нітрогліцерину, після
чого відчув сильний головний біль. Який лікарський засіб слід рекомендувати приймати
одночасно з нітратами для зменшення інтенсивності болю голови? A. Ацетилсаліцилова
кислота.
B. Кеторолак.
C. Кофеїн.
D. Суматриптан.
E. Валідол.

Вкажіть ЛЗ для етіотропної фармакотерапії ревматизму.


A. хлорохін
B. мелоксикам
C. бензилпеніцилін
D. левофлоксацин
E. доксициклін

Хворий на гострий бронхіт скаржиться на кашель з виділенням помірної кількості харкотиння.


Лікар призначив бромгексин, ацетилцистеїн та преноксдіазин. Яка корекція фармакотерапії
необхідна у цьому випадку? A. Додати глауцин.
B. Додати теофілін пролонгованої дії.
C. Відмінити преноксдіазин.
D. Корекції проводити не потрібно.
E. Відмінити ацетилцистеїн.

До симптомів передозування нітрогліцерином не належить: A.


Біль голови, запаморочення.
B. Артеріальна гіпотензія, тахікардія.
C. Артеріальна гіпертензія.
D. Метгемоглобінемія.
E. Підвищення внутрішньочерепного тиску.

Оберіть неправильне твердження щодо особливостей організму людей похилого та старечого віку:
A. Кількість води зменшується на 15-20%
B. Зменшується вміст білків (альбумінів)
C. Кількість жирової тканини зростає на 20-25%
D. Кількість жирової тканини зменшується на 20-25%
E. Зменшується функціональна активність мікросомальних ферментів печінки

Порадьте, який агоніст β2-адренорецепторів застосовують для усунення нападів бронхіальної


астми:
A. Орципреналін.
B. іпратропію бромід.
C. Сальбутамол.
D. Беклометазон.
E. Сальметерол.

Жінці, хворій на артеріальну гіпертензію, був призначений лізиноприл. Який механізм дії цього ЛЗ?
A. Блокада ангіотензин перетворюючого ферменту
B. Блокада рецепторів до ангіотензину ІІ
C. Блокада альфа1-адренорецептори судин
D. Зменшує вміст Na+ в організмі
E. Блокада бета-адренорецепторів

Назвіть симптом, не характерний для гіпертензивного кризу.


A. Блювання.
B. Часте сечовиділення.
C. Тенезми.
D. Порушення зору.
E. Миготіння «мушок» перед очима.

Який вид фармакотерапії використовується для лікування хвороби Аддісона


A. Патогенетична
B. Етіологічна
C. Правильна відповідь відсутня
D. Симптоматична
E. Замісна

Визначте, який з названих лікарських засобів не належить до антибіотиків групи цефалоспоринів:


A. ципрофлоксацин
B. цефалоридин
C. цефпіром
D. цефалексин
E. цефокситин
В аптеку звернувся пацієнт, який втрату голосу, сухий подразливий кашель і, підвищення
температури тіла до 37,4˚С. Яке захворювання Ви можете запідозрити? A. Катаральна
ангіна.
B. Тонзиліт.
C. Ларингіт.
D. Лакунарна ангіна. E. Фарингіт.

Хворому з набряками серцевого ґенезу ввели фуросемід. Який механізм дії цього ЛЗ? A.
Інгібує фермент карбоангідразу.
B. Покращує кровоплин у нирках.
C. Сприяє нагромадженню цАМФ у тканинах організму.
D. Пригнічує реабсорбцію натрію і хлору.
E. Блокує SH-групи ферментів клітин ниркових канальців.

Оберіть неінгаляційний анестетик похідний бензодіазепіну, який застосовується для премедикації,


у тому числі при проведенні короткотермінових маніпуляцій та під час хірургічних втручань
A. тіопентал натрію
B. натрію оксибутират
C. кетамін
D. мідазолам
E. ксенон

Назвіть номер дієти для лікування хворого з артеріальною гіпертензією та ішемічною хворобою
серця?
A. 13
B. 5
C. 10
D. 7
E. 1

До лікаря звернувся хворий 70 р., з приводу підвищеного артеріального тиску. З анамнезу відомо
про наявність доброякісної гіперплазії передміхурової залози. Який ЛЗ бажано призначити цьому
хворому?
A. Пропранолол.
B. Доксазозин.
C. Лозартан.
D. Дилтіазем.
E. Еналаприл.

Хворий П., 67 р., знаходиться у терапевтичному відділенні з приводу прогресування серцевої


недостатності. Через 3 тижні лікування стан пацієнта погіршився, що проявилось зниженням
апетиту, нудотою, при погляді на джерело світла з’явились жовті кола. Який із ЛЗ міг спричинити
вказаний стан?
A. Ізосорбіду динітрат.
B. Еналаприл.
C. Гідрохлортіазид.
D. Дигоксин.
E. Ацетилсаліцилова кислота.

У хворого на артеріальну гіпертензію при систематичному лікуванні антигіпертензивним засобом


з’явився кашель. Який з нижчезазначених засобів може бути причиною такої побічної дії? A.
Клонідин.
B. Каптоприл.
C. Бендазол.
D. Верапаміл. E. Празозин.

У хворої з жовчнокам'яною хворобою виник приступ інтенсивного болю в правому підребер’ї,


нудота, блювота. В анамнезі – пептична виразка шлунка. В роті відчуття гіркоти. Який найбільш
вірогідний діагноз?
A. Ниркова коліка
B. Гострий панкреатит
C. Перфоративна виразка
D. Печінкова коліка
E. Пенетраця виразки шлунка в підшлункову залозу

Назвіть антифіброзний ЛЗ, що застосовується для лікування ревматоїдного запалення суглобів


A. гідроксихлорохін
B. метотрексат
C. пеніциламін
D. целекоксиб
E. диклофенак

Поняття комплаєнс означає:


A. Готовність пацієнта проводити відповідальне самолікування.
B. Готовність пацієнта виконувати всі рекомендації лікаря.
C. Готовність пацієнта проводити само профілактику.
D. Всі відповіді правильні.
E. Готовність пацієнта до самодопомоги.

Причинами розвитку гострого токсичного гепатиту у першу чергу є:


A. Вірусний гепатит
B. Сальмонельоз
C. Застосування антибiотикiв широкого спектру дiї
D. Отруєння солями металiв, фосфорорганiчними речовинами
E. Застосування цитостатикiв

Хворому з пептичною виразкою дванадцятипалої кишки для ерадикації хелікобактерної інфекції


було призначено лансопразол, кларитроміцин, амоксицилін. Яка побічна реакція може виникнути
у хворого?
A. Діарея
B. Пенетрація
C. Дисбіоз
D. Закреп
E. Кровотеча

При якій супутній патології застосування ксилометазоліну та інших симпатоміметиків не


рекомендовано?
A. Артеріальна гіпотензія.
B. Гломерулонефрит.
C. Кишкова непрохідність.
D. Епілепсія.
E. Глаукома.
У хворого виник інтенсивний біль в попереку, часті позиви до сечовипускання. Температура тіла
39°C, дизуричні явища. АТ – 150/90 мм рт ст.. В сечі – лейкоцитурія, помірна протеїнурія. Про яке
захворювання можна думати у першу чергу.?
A. Інтерстиціальний нефрит
B. Гіпертонічна хвороба
C. Гломерулонефрит
D. Приступ ниркової кольки
E. Пієлонефрит

Комбінована фармакотерапія – це
A. Поєднання декількох лікарських засобів з різними фармакотерапевтичними ефектами.
B. Правильна відповідь відсутня.
C. Поєднання декількох лікарських форм лікарського засобу.
D. Усі відповіді правильні.
E. Поєднання декількох шляхів введення лікарського засобу.

Оберіть серед наведених гіполіпідемічних ЛЗ інгібітор ГМГ-КоА-редуктази A.


Холестирамін.
B. Аторвастатин.
C. Кислота нікотинова.
D. Клофібрат.
E. Фенофібрат.

Максимальна (100%) біологічна доступність завжди можлива при:


A. інгаляційному введенні
B. дом’язовому введенні
C. ректальному введенні
D. підшкірному введенні
E. внутрішньосудинному введенні

Оберіть ЛЗ, що надлежить до непрямих антикоагулянтів? A.


Дипіридамол.
B. Гепарин.
C. Варфарин.
D. Ацетилсаліцилова кислота.
E. Еноксипарин.
У хворого В., з пневмонією на тлі лікування цефазоліном, бісептолом, димедролом, вітамінами,
бромгексином протягом 3 тижнів з’явилися біль в горлі, при огляді на слизовій оболонці
порожнини рота спостерігається білий наліт. Яку помилку в роботі допустив лікар? A. Не
призначав дезінтоксикаційну терапію.
B. Не призначив протигрибковий ЛЗ.
C. Не призначав імунокоректори.
D. Не проводив лабораторний контроль крові (лейкоцити, еритроцити).
E. Не проводив лабораторний контроль за мокротинням.

Схема введення препаратів інсуліну, яка імітує фізіологічну секрецію називається:


A. Базисна
B. Інтермітуюча
C. Пульсаційна
D. Базис-болюсна
E. Болюсна
Термін «систолічна гіпертензія» означає:
A. Ізольоване підвищення діастолічного АТ.
B. Гіпертензію, причину якої можна встановити.
C. Ізольоване підвищення систолічного АТ.
D. Артеріальну гіпертензію у осіб молодого віку.
E. Гіпертензію, причину якої неможливо встановити.

Для хворого на пептичну виразку шлунка оберіть засіб, який одночасно знижує кислотоутворення в
шлунку і має здатність пригнічувати Helicobacter pylori
A. Лансопразол
B. Альмагель
C. Гастроцепин
D. Маалокс
E. Ренні

У хворого з високою температурою, кашлем, задишкою після проведених обмежень встановлено


діагноз: стрептококова пневмонія. Який ЛЗ залишається засобом вибору при цьому захворюванні?
A. Ампіцилін
B. Тетрациклін
C. Бензилпеніцилін
D. Еритроміцин
E. Гентаміцин

До лікаря звернувся хворий зі скаргами на пронос, спастичні болі внизу живота, слабість, які не
залежать від прийому їжі. Хворий втратив вагу. В анамнезі – нещодавнє тривале самолікування
антибіотиками з приводу ангіни. Про яке захворювання слід думати в першу чергу?
A. Хронічний коліт
B. Гастродуоденіт
C. Гострий ентероколіт
D. Дисбактеріоз кишківника
E. Хронічний панкреатит

У хворої 56 років - жовчнокам'яна хвороба. На УЗД виявлено конкремент 30 мм. Яке з ускладнень
може розвинутись при несприятливому перебігу цього захворювання:
A. Перфорацiя жовчного мiхура
B. Гепатолiєнальний синдром
C. Нутрiтивна алергiя
D. Тромбоз печiнкових артерiй
E. Кахексiя

Ускладненням гострого гломерулонефриту не є:


A. Ниркові конкременти
B. Гостра ниркова недостатність
C. Артеріальна гіпертензія
D. Нефротичний синдром
E. Гостра лівошлуночкова недостатність

Який лікарський засіб отримують із трави горицвіту весняного, що виявляє позитивний ізотропний
ефект?
A. Дигоксин.
B. Целанід.
C. Строфантин.
D. Адонізид.
E. Корглікон.

Після 2-х днів застосовування присипки, до складу якої входить борна кислота, у немовляти
з’явилися симптоми інтоксикації – блювання, нудота, падіння артеріального тиску, судоми. Як
пояснити виникнення інтоксикації?
A. Правильна відповідь відсутня
B. Добре кровопостачання шкіри, тонкий роговий шар, відсутня підшкірна жирова клітковина C.
Сповільнений метаболізм ЛЗ у печінці, недостатня швидкість елімінації ЛЗ нирками
D. Усі відповіді вірні
E. Пришвидшений метаболізм ЛЗ у печінці, більша швидкість елімінації ЛЗ нирками

Посилення симптоматики хвороби Паркінсона до кінця дії кожної дози леводопи називається
A. біфазна дискінезія
B. дистонія включення чи піку дози
C. феномен «зношування»
D. дистонія виключення чи кінця дози
E. синдром «включення-виключення»

Хвора 17 років, страждає на бронхіальну астму персистуючого перебігу легкого ступеня. Останнім
часом напади задухи почастішали до 8 разів на добу. Для купування нападу використовує інгаляції
сальбутамолу. Яку групу ліків слід призначити в якості засобів базисної терапії A.
Інгаляційні глюкокортикоїди.
B. Кромони.
C. Холінолітики.
D. Антибіотики.
E. Системні глюкокортикостероїди.

Які з перелічених ЛЗ є селективними інгібіторами ЦОГ–2


A. мефенамінова кислота
B. ацетамінофен
C. індометацин
D. мелоксикам
E. диклофенак натрію

В аптеку звернувся хворий із сильним сухим кашлем. Який лікарський засіб Ви порекомендуєте
вжити хворому з метою пригнічення непродуктивного кашльового рефлексу?
A. Калію йодид.
B. Натрію гідрокарбонат.
C. Ацетилцистеїн.
D. Трипсин.
E. Лібексин.

Хвора 52 років перебуває на стаціонарному лікуванні з діагнозом гострого панкреатиту. Який із


перелічених ЛЗ протипоказаний у даному випадку?
A. Промедол
B. Метамізол натрію
C. Морфін
D. Апротинін
E. Папаверин
Хворій на ангіну дитині призначено комбінований засіб, який мистить в собі сульфаніламід та
триметоприм. Який ЛЗ призначено?
A. Ентеросептол
B. Бісептол
C. Сульфаметаксозол
D. Сульфален
E. Сульфадиметоксин

Яка з наведених анемій є мікроцитарною?


A. залізодефіцитна
B. постгеморагічна
C. гемолітична
D. В12-дефіцитна
E. фолієводефіцитна

Для гострої пневмонії не характерні наступні симптоми: A.


Прискорення ШОЕ і лейкоцитоз.
B. Температура тіла понад 38˚.
C. Постійний сухий кашель.
D. Кашель з харкотиннім, іноді з “іржавим” (прожилки крові).
E. Вологі хрипи над легенями.

Для лікування хронічного холециститу хворій рекомендовано холеретик рослинного походження.


Назвіть його.
A. Холосас
B. Циквалон
C. Нікотин
D. Дротаверин
E. Сорбіт

Накопичення ЛЗ при його повторних введеннях – це


A. Анафілаксія.
B. Ідіосинкразія.
C. Потенціювання.
D. Кумуляція.
E. Тахіфілаксія.

До тіазидоподібних діуретиків належить:

A. Індапамід.
B. Етакринова кислота.
C. Фуросемід.
D. Торасемід.
E. Спіронолактон.

Яку групу лікарських засобів необхідно застосовувати у фармакотерапії пацієнтів з хронічним


гломерулонефритом з високою активністю процесу та резистентністю до глюкокортикоїдів?
A. Антибіотики.
B. Бета-адреноблокатори.
C. Антиагреганти.
D. Імунодепресанти.
E. Нестероїдні протизапальні засоби.
У хворого діагностували інфаркт міокарда з тромбозом лівої коронарної артерії. Яка з означених
фармакологічних груп ЛЗ використовується для відновлення кровотоку в коронарних артеріях у
перші 6-12 годин захворювання?
A. Курантил
B. Активатори фібринолізу
C. Гепарин
D. Інгібітори АПФ
E. Бета-адреноблокатори

Інфікування організму мікобактерією туберкульозу визначають за допомогою:


A. Реакції Манту.
B. Флюорографії легень.
C. Вакцинації БЦЖ.
D. Аускультації легень.
E. Проби Вассермана.

У хворого К, 53 років, бронхіальна астма та стенокардія напруги. Який лікарський засіб із


перелічених нижче протипоказаний у цьому випадку? A. Амлодипін.
B. Преднізолон.
C. Орципреналін.
D. Беклометазон. E. Сальбутамол.

Які ускладнення найчастіше виникають при лікуванні протитуберкульозними засобами? A.


Кровоточивість
B. Диспепсія
C. Кашель
D. Тахікардія
E. Резистентність

Який з ЛЗ не має антиагрегантних властивостей?


A. Абсіксімаб
B. Аспірин
C. Тиклопідин
D. Клопідогрель
E. Надропарин

Розрив ахілового сухожиння є характерною побічною реакцією при застосуванні антибактерійних


засобів групи:
A. Тетрацикліни.
B. Фторхінолони.
C. Пеніциліни.
D. Аміноглікозиди.
E. Цефалоспорини.

Хворому на гострий пієлонефрит, викликаний коковою інфекцією, лікар збирається призначити


антибіотик. Антибіотики пеніцилінового ряду викликають у хворого алергічну реакцію. Порадьте л
ікарю, який антибіотик можна рекомендувати для лікування хворого?
A. Оксацилін
B. Цефтазидим
C. Феноксиметилпеніцилін
D. Цефуроксим
E. Азитроміцин
При відпуску нестероїдних протизапальних засобів для ревматологічного відділення провізор
міжлікарняної аптеки акцентував увагу лікарів, що найменшою ушкоджувальною дією на
шлунково-кишковий тракт володіє лікарський засіб:
A. целекоксиб
B. піроксикам
C. індометацин
D. диклофенак
E. вольтерен

При інформуванні хворого про наявність в аптеці жовчогінних засобів, відмітьте, якому з ЛЗ
властива холеретична дія?
A. Холецистокінін
B. Сорбіт
C. Еуфілін
D. Холензим
E. Магнію сульфат

В аптеку звернувся хворий на бронхіальну астму, якому призначено ЛЗ з групи М-холінолітик. Який
це ЛЗ?
A. Беклометазон.
B. Іпратропію бромід.
C. Сальметерол.
D. Теофілін.
E. Будесонід.

Час та кратність дозування лікарських засобів визначається наступними фармакокінетичними


параметрами:
A. t1/2
B. Vd
C. F
D. Cl
E. t1/2a

Який механізм дії антацидних засобів?


A. Викликають дегідратацію або часткову коагуляцію тканних білків чи ранового ексудату і утворюють
альбумінати, які захищають слизову оболонку
B. Утворюють плівку з колоїду, яка захищає чутливі нервові закінчення від дії соляної кислоти
C. Блокують Н2-гістамінові рецептори шлунка
D. Нейтралізують соляну кислоту в шлунку
E. Блокують фермент Н+-К+-АТФ-азу, яка відповідає за продукцію соляної кислоти

Хворому з артеріальною гіпертензією призначено раміприл. Який механізм дії цього ЛЗ?
A. Блокада α-адренорецепторів.
B. Блокада рецепторів ангіотензину ІІ.
C. Периферична вазодилатуюча дія.
D. Блокада β-адренорецепторів.
E. Пригнічення ангіотензинперетворюючого ферменту.

Дитині 10 років для усунення сухого кашлю сусідка порадила застосовувати бромгексин. Однак,
кашель не припинявся. Порадьте, який протикашлевий лікарський засіб доцільніше застосовувати.
A. Настій кореня алтея.
B. Амброксол.
C. Окселадин.
D. Кодеїну сульфат.
E. Ацетилцистеїн.

Хвора скаржиться на задишку, серцебиття, набряки ступнів, які турбують її більше трьох місяців. В
анамнезі – перенесений інфаркт міокарда, гіпертонічна хвороба. Оберіть препарати для
гемодинамічного розвантаження серця у цієї хворої:
A. Каптоприл, дігоксин
B. Дігітоксин, вазапростан
C. Метопролол, ірбесартан
D. Верапаміл; молсидомін
E. Фуросемід, олікард

Для бронхіальної астми не характерне: A.


Важке відкашлювання мокротиння.
B. Напади ядухи.
C. Сухі хрипи.
D. Підвищення температури. E. Еозинофілія.

В аптеку звернувся хворий 75 років, який скаржиться на напади болю за грудиною і в ділянці
серця, задишку, біль голови. Артеріальний тиск у хворого 110/60 мм рт.ст. Має глаукому. Який з
лікарських засобів йому протипоказаний? A. Бісопролол.
B. Молсидомін.
C. Ніфедипін.
D. Нітрогліцерин. E. Атенолол.

Хворий, 32 років, надійшов на лікування з діагнозом виразкова хвороба 12-палої кишки. Лікар
призначив відсутній в аптеці ранітидин. Яким ЛЗ із дозволу лікаря можна його замінити?
A. Лансопразол
B. Вісмуту субцитрат
C. Панкреатин
D. Кларитроміцин
E. Метронідазол

Виберіть правильне твердження:


A. Холелітоличні ЛЗ ефективні при наявності білірубінових та кальцифікованих конкрементів
B. Пероральний літоліз здійснюють при наявності конкрементів більше 30 мм
C. Лікування засобами жочних кислот триває 1-2 місяці
D. Фармакотерапія холелітичними ЛЗ здійснюється від 6 до 24 місяців під контролем УЗД E.
Урсодезоксихолеву кислоту приймають вранці натще

Оберіть неінгаляційний анестетик – барбітурат, який застосовується при нетривалих оперативних


втручаннях; вступний та базисний наркоз з подальшим використанням інших засобів анестезії?
A. мідазолам
B. ксенон
C. пропофол
D. тіопентал натрію
E. ізофлуран

Основним засобом фармакотерапії цукрового діабету 1 типу є:


A. Пероральні цукрознижуючі засоби
B. Бігуаніди
C. Фізичні навантаження
D. Дієта
E. Препарати інсуліну

Хворій Н. з неускладненою позашпитальної пневмонії з метою полегшення відходження в’язкого


мокротиння був призначений муколітик прямої дії, а саме: A. Бромгексин.
B. Карбоцистеїн.
C. Гвайфенезин.
D. Амброксол.
E. Ацетилцистеїн.

Скільки білка може бути в сечі здорової людини?


A. Менше 1,0 г/л
B. Більше 0,01 г/л
C. Від 1,0 до 10,1 г/л
D. Не повинно бути
E. Менше 0,01 г/л

Назвіть лікарські засоби першого ряду для лікування туберкульозу легень.


A. Офлоксацин, стрептоміцин
B. Ізоніазид, ПАСК
C. Ізоніазид, рифампіцин
D. Офлоксацин, амоксицилін з клавулановою кислотою
E. ПАСК, протіонамід

Допоможіть молодому колезі обрати симптом не характерний для хвороби Паркінсона:


A. прискорення мови
B. м’язова ригідність
C. «шаркаюча» хода
D. тремор по типу «катання пілюль»
E. гіпомімія

Який з перелічених лікарських засобів не виявляє ульцерогенної дії?


A. Індометацин
B. Преднізолон
C. Ацетилсаліцилова кислота
D. Пропранолол
E. Резерпін

Пацієнту з закладенням носа для відновлення носового дихання призначили деконгестант


оксиметазолін, що є прикладом:
A. Профілактичної фармакотерапії.
B. Патогенетичної фармакотерапії.
C. Етіотропної фармакотерапії.
D. Замісної фармакотерапії.
E. Симптоматичної фармакотерапії.

При лікуванні еналаприлом можливий наступний побічний ефект:


A. Слюнотеча.
B. Кашель.
C. Закреп.
D. Набряки кінцівок.
E. Гіпокаліемія.
Характерною ознакою хвороби Аддісона є:
A. Нудота
B. Прогресуюче потемніння шкіри та слизових оболонок
C. Підвищена втомлюваність
D. Гіпоглікемія
E. Артеріальна гіпотензія

Який гормон гіпофізу впливає на білковий, вуглеводний і ліпідний обмін, контролює швидкість
скелетного та вісцерального росту?
A. Адренокортикотропін
B. Інтермедин
C. Соматропін
D. Тиреотропін
E. Вазопресин

Хворий на аутоіммунний гепатит, ШОЕ 60 мм/год. Який з наявних ЛЗ підходить для лікування в
даній ситуації?
A. Нічого з перерахованого
B. Гептрал
C. Урсодезоксихольова кислота
D. Преднізолон
E. Альфа-Інтерферон

Підшкірні вузлики є діагностичним критерієм:


A. остеоартрозу X
B. системної склеродермії
C. подагри
D. остеопорозу
E. гострої ревматичної лихоманки

Пацієнт скаржиться на раптову появу нудоти, блювоти “кавовою гущею”, запаморочення, раптову
слабість. В анамнезі – пептична виразка шлунка. Хворий блідий, покритий холодним липким
потом. Що могло стати причиною описаного стану ?
A. Гостре порушення мозкового кровообігу
B. Гострий панкреатит
C. Гострий пієлонефрит
D. Гострий інфаркт міокарда
E. Шлунково-кишкова кровотеча

Найчастіше етіологічним чинником гострого бронхіту є: A.


Гемофільна паличка.
B. Гонококи.
C. Віруси.
D. Хламідії.
E. Пневмококи.

Оберіть схему фармакотерапії пептичної виразки 12-палої кишки першої лінії:


A. Омепразол, Амоксицилін, Кларитроміцин
B. Пантопразол, Вісмуту субцитрат, Метронідазол, Тетрациклін
C. Метронідазол, Алюмінію гідроксид+магнію гідроксид, Тетрациклін, Мізопростол
D. Фамотидин, Орнідазол, Ампіцилін, Диклофенак
E. Рабепразол, Метронідазол, Алюмінію гідроксид+магнію гідроксид, домперидон

Хворому з генералізованою формою сальмонельозу була призначена антибактеріальна терапія.


Через деякий час при проведенні загального аналізу крові була виявлена ретикуло- та
грунулоцитопенія, відмічалось деяке зниження кількості еритроцитів. При призначенні якого
антибіотику найбільш вірогідна подібна зміна картини крові?
A. гентаміцину сульфат х
B. тетрациклін х
C. амоксицилін х
D. цефтріаксон
E. хлорамфенікол

У хворого скарги на інтенсивні приступоподібний інтенсивний біль внизу живота з іррадіацією в


спину, нудоту. Загальний аналіз сечі: білок - 0,066 г/л; в полі зору: епітелій перехідний і нирковий
по 2-3, лейкоцити 8-10 місцями групами по 20-30, еритроцити не змінені 80-120, кристали сечової
кислоти окремо і зростки-багато. Яке захворювання можна запідозрити?
A. Гострий апендицит х
B. Гострий гломерулонефрит
C. Ниркова коліка
D. Гострий цистит
E. Гострий пієлонефрит

Пацієнтка 32 р. звернулася до лікаря зі скаргами на ранкову скутість в суглобах, яка триває кілька
год. При огляді – симетрична деформація кистей у вигляді «плавників моржа», коліна збільшені,
деформовані. Який діагноз можна припустити?
A. остеопороз
B. остеоартроз первинний
C. подагра
D. ревматизм
E. ревматоїдний артрит

Туберкулінодіагностика – це:
A. Проба Манту.
B. Мікроскопія змивних вод бронхів.
C. Пробна протитуберкульозна хіміотерапія.
D. Культуральне виявлення мікобактерій туберкульозу.
E. Імунологічне дослідження із серологічними пробами на туберкульоз.

Лікар-початківець для лікування гіпертонічної хвороби призначив хворому 56 років комбіновану


фармакотерапію. Схема лікування складається з трьох різних лікарських засобів протягом доби, з
яких два належать до однієї групи гіпотензивних засобів, але мають деякі відмінності
фармакодинамічних показників. Відмічається повільне покращення самопочуття і стабілізація
артеріального тиску. Дайте оцінку фармакотерапії.
A. Існує підвищений ризик побічної дії від взаємодії ЛЗ.
B. Раціональна фармакотерапія.
C. Нераціональна фармакотерапія.
D. Поліпрагмазія. ?
E. Існує ризик розвитку лікової залежності.

Які з наведених засобів можуть послаблювати терапевтичну дію ЛЗ заліза?


A. глюкоза
B. тетрациклін
C. алкоголь
D. метіонін
E. аскорбінова кислота

Антибактерійними засобами вибору для лікування ангіни є: A.


Фторхінолони.
B. Тетрацикліни.
C. Аміноглікозиди.
D. Пеніциліни.
E. Сульфаніламіди.

Оберіть ЛЗ, що надлежить до прямих антикоагулянтів?


A. Дипіридамол.
B. Ацетилсаліцилова кислота.
C. Вікасол.
D. Варфарин.
E. Еноксипарин.

Хворий з хронічними гастроентероколітом та холециститом, що супроводжуються важкістю в


шлунку після їжі та здуттям живота, звернувся до провізора з проханням порекомендувати йому
препарат для покращання процесів травлення. Який з названих засобів найбільш ефективний? A.
Панкреатин
B. Симетикон
C. Натуральний шлунковий сік
D. Панкреатин + хеміцелюлази + порошок жовчі бичачої
E. Пепсин

Хворий на вірусний гепатит, в крові HbsAg (Поверхневий антиген вірусу гепатиту В). Який з наявних
ЛЗ підходить для лікування в даній ситуації? A.
Нічого з перерахованого
B. Гептрал ?
C. Урсодезоксихольова кислота
D. Преднізолон
E. Альфа-Інтерферон

Для профілактики загострення бронхіальної астми ЛЗ вибору є: A.


Метилксантини.
B. Седативні лікарські засоби.
C. Холінолітики.
D. Інгаляційні глюкокортикоїди. E. Симпатоміметики.

У жінки 50 р., біль у суглобах кистей рук, значна ранкова скутість; ШОЕ 24 мм/год. Який діагноз
можна припустити?
A. дерматоміозит
B. системна склеродермія
C. ревматоїдний артрит
D. системний червоний вовчак
E. хронічний активний гепатит

Зниження метаболізму в печінці вимагає наступних дій:


A. Підвищення дози лікарського засобу
B. Зміни шляху введення лікарського засобу
C. Зниження дози лікарського засобу та моніторингу побічних ефектів
D. Моніторингу дії лікарського засобу
E. Збільшення дози лікарського засобу

Піурія – це:
A. Наявність солей в сечі
B. Наявність епітелію в сечі
C. Наявність гною в сечі
D. Наявність крові в сечі E. Наявність слизу в сечі

При відпуску з аптеки вушних крапель пацієнта слід повідомити, що:


A. Після розкриття флакона краплі можна застосовувати до закінчення їх терміну придатності.
B. Краплі слід нагріти до температури тіла.
C. З метою попередження виникнення побічних реакцій вушні краплі застосовують лише вранці.
D. При виникненні алергії не слід припиняти застосування крапель. E. Усі вушні краплі придатні й для
застосування в офтальмології.

До клінічних ознак системного червоного вовчака належать всі перелічені, окрім наступних: A.
вісцерити – плеврит, перикардит
B. кардит
C. еритематозні висипання на спинці носа і щоках
D. нефрит
E. остеоліз нігтьових фаланг, рот у вигляді кисету

Яке захворювання можна запідозрити у пацієнта, якщо після зняття нальоту із мигдаликів
з’явилися кровоточиві виразки?
A. Виразково-некротична ангіна.
B. Дифтерія.
C. Лакунарна ангіна.
D. Дизентерія.
E. Флегмонозна ангіна.

Хворий з явищами бронхіту звернувся до провізора з проханням порекомендувати йому


відхаркувальний засіб рослинного походження. Який з ЛЗ можна запропонувати пацієнту?
A. Натрію хлорид.
B. Натрію гідрокарбонат.
C. Амброксол.
D. Мукалтин.
E. Ацетилцистеїн.

Вагітна жінка захворіла на негоспітальну пневмонію. Який з перелічених антибактерійних засобів


доцільно призначити: A. Геміфлоксацин.
B. Тетрациклін.
C. Офлоксацин.
D. Амоксицилін.
E. Гентаміцин.

Механізм дії ловастатину полягає у:


A. Підвищенні фібринолітичної активності.
B. Посиленні синтезу простагландинів.
C. Інгібуванні ферменту ГМГ-КоА-редуктаза.
D. Зменшенні мобілізації вільних жирних кислот.
E. Активації фермента протеїдліпаза.

Оберіть неінгаляційний анестетик – барбітурат, який застосовується при нетривалих оперативних


втручаннях; вступний та базисний наркоз з подальшим використанням інших засобів анестезії?
A. тіопентал натрію
B. ксенон
C. мідазолам
D. ізофлуран
E. пропофол

Який з наведених ЛЗ є засобом вибору для лікування артеріальної гіпертензії у вагітної жінки в ІІ
триместрі?
A. Метилдопа.
B. Еналаприл.
C. Верапаміл.
D. Лозартан.
E. Фуросемід.

Назвіть лікарський засіб, який належить до групи інгаляційних глюкокортикоїдів: A.


Тербуталін.
B. Окситропіум.
C. Флутиказон.
D. Зафірлукаст.
E. натрію кромоглікат.

Які з перелічених фармакотерапевтичних груп лікарських засобів є засобами першого ряду для
фармакотерапії стабільної стенокардії?
A. Блокатори β-адренорецепторів.
B. Тіазидні діуретики.
C. Блокатори α-адренорецепторів.
D. Калійзберігаючі діуретики.
E. Агоністи імідазолінових рецепторів.

Після неодноразових нападів задухи та кашлю з виділенням в’язкого харкотиння під час фізичного
навантаження, що супроводжувались свистячим диханням, у хворого Е., 27 років, діагностовано
бронхіальну астму. Лікар розпочав лікування з призначення натрію кромоглікату. До якої групи ЛЗ
належить кромоглікат натрію?
A. Стабілізатори мембран опасистих клітин.
B. Метилксантини тривалої дії.
C. Інгаляційні кортикостероїди.
D. Агоністи β2-адренорецепторів тривалої дії. E. М-холінолітики.

У хворого на перитоніт висіяна синьо-гнійна паличка. Який лікарський засіб з групи цефалоспоринів
ІІІ покоління слід призначити?
A. цефтріаксон
B. цефепім
C. цефаклор
D. цефалексин
E. цефазолін
Хворому на гострий бронхіт призначили відхаркувальний засіб, після приймання якого виникло
блювання. Який з перерахованих ЛЗ міг спричинити подібну дію?
A. Терпінгідрат.
B. Натрію гідрокарбонат.
C. Калію йодид.
D. Настій трави термопсису.
E. Бромгексин.

Летальна доза заліза:


A. 150-160 мг/кг маси тіла
B. 70 мг на добу
C. 1 мг/кг маси тіла
D. 18-30 мг/кг маси тіла
E. 180-300 мг/кг маси тіла

Антибактерійним ЛЗ вибору для лікування синуситів є:


A. Ципрофлоксацин.
B. Тетрациклін.
C. Гентаміцин.
D. Амоксицилін.
E. Ванкоміцин.

Вкажіть чинник, який впливає на процес абсорбції


A. значення рН в місці всмоктування лікарської речовини
B. константа абсорбції
C. правильної відповіді немає
D. терапевтичний моніторинг
E. вид упакування лікарського засобу

До основних груп лікарських засобів, які справляють пряму бронхолітичну дію належать: A.
Стабілізатори мембран опасистих клітин, системні кортикостероїди, теофіліни.
B. М-холінолітики, агоністи β2-адренорецепторів, метилксантини.
C. Блокатори лейкотрієнових рецепторів, стабілізатори мембран опасистих клітин,
глюкокортикостероїди.
D. Стабілізатори мембран опасистих клітин, метилксантини, холінолітики.
E. Теофіліни, М-холіноблокатори, інгаляційні кортикостероїди.

Хворий Б., 35 років, хворіє на пептичну виразку асоційовану з Helicobacter pylori. Лікар призначив
амоксицилін, омепразол, сукральфат, алюмінію гідроксид+магнію гідроксид, фамотидин. Вкажіть
який із перерахованих лікарських засобів володіє антимікробною дією?
А. Амоксицилін
B. Омепразол
C. Фамотидин
D. Сукральфат
E. Алюмінію гідроксид+магнію гідроксид

Оберіть міорелаксант із центральним механізмом дії, похідний хіноліну, що застосовується при


інкубації трахеї, ендоскопічних процедурах (бронхо-, езофаго-, ларинго-, цистоскопія) тощо.
A. векуроній
B. гігроній
C. суксаметоній
D. рокуроній
E. атракуріум
До ускладнень, пов’язаних з перебігом неспецифічного виразкового коліту, не належать:
A. Анальнi тріщини
B. Шлункова кровотеча
C. Злоякiснi переродження
D. Перфорацiя, гостра токсична дилятацiя кишкiвника
E. Кишкова кровотеча

Хворий 54 років хворіє на ішемічну хворобу серця. Який з перелічених лікарських засобів володіє
одночасно коронаророзширюючою і антиагрегантною діями?
A. Аспірин
B. Лізиноприл
C. Ніфедипін
D. Дипіридамол
E. Нітросорбід

Назвіть вид фармакотерапії, який використовується при лікуванні гіпофізарного нанізму: A.


Етіологічна
B. Симптоматична
C. Профілактична
D. Патогенетична
E. Замісна ?

Етіологічною причиною виникнення акромегалії вважають:


A. Гіперпродукція соматостатину
B. Дефіцит соматотропіну
C. Ураження гіпоталамуса і/або гіпофіза
D. Гіпопродукція ліберину внаслідок черепно-мозкової травми
E. Гіперчутливість рецепторів епіфізарних хрящів до соматотропіну

Як називається ендокринне захворювання, спричинене надлишком соматотропіну, яке виникає у


дітей та підлітків? A. Гіперплазія гіпофізу
B. Хвороба Іценко-Кушинга
C. Синдром Іценко-Кушинга
D. Акромегалія
E. Гігантизм

Хворий 522 років, переніс інфаркт міокарда. Для попередження тромбоутворення призначена
ацетилсаліцилова кислота. Визначте раціональну добову дозу:
A. 1000 мг.
B. 100 мг.
C. 2000 мг.
D. 3000 мг.
E. 500 мг.

Розрахувати константу елімінації цефамандолу, якщо відомо, що його період напіввиведення


становить 45 хв. A. 0,731 год-1
B. 0,248 год-1
C. 0,924 год-1
D. 0,214 год-1
E. 0,102 год-1
Аускультація – це метод обстеження хворого, що ґрунтується на:
A. Дотику та промацуванні поверхневих тканин і органів, що знаходяться глибше.
B. Оцінюванні звуку отриманого при вистукуванні поверхні тіла.
C. Просвічуванні окремих частин тіла рентгенівськими променями.
D. Дослідженні слизової оболонки порожнистих органів за допомогою ендоскопу. E. Вислуховуванні
звукових явищ внутрішніх органів.

Хворому з гіпертензивним кризом внутрішньовенно ввели клонідин. Який механізм лежить в


основі антигіпертензивної дії клонідину?
A. Блокада α-адренорецепторів.
B. Пряма міотропна дія.
C. Блокада β-адренорецепторів.
D. Пригнічення активності ангіотензинперетворюючого ферменту.
E. Стимуляція центральних α2-адренорецепторів.

Визначте період напіввиведення фуросеміду, якщо відомо, що його константа елімінації становить
0,346 год.-1?
A. 2 год.
B. 2,5 год.
C. 2,05 год.
D. 1,8 год. E. 3 год.

Визначте період напіввиведення німодипіну, якщо відомо, що його константа елімінації становить
0,753 год.-1? A.
1,5 год.
B. 0,99 год.
C. 0,92 год.
D. 0,8 год.
E. 0,03 год.

Який з наведених антибіотиків має нефротоксичну дію:


A. гентаміцин
B. еритроміцин
C. роваміцин
D. ампіцілін
E. пеніцилін

Хворому в післяопераційний період тривалий час вводили промедол з метою знеболення. Після
відміни ЛЗ у хворого виникли важкі психічні і соматичні розлади. Яка причина цього побічного
явища фармакотерапії?
A. Синдром обкрадання.
B. Тахіфілаксія.
C. Абстинентний синдром.
D. Ідіосинкразія.
E. Синдром відміни.

Вкажіть, яка із наведених побічних реакцій є характерною при застосуванні інгібіторів ГМГ-
КоАредуктази:
A. Гіперурикемія.
B. Рабдоміоліз.
C. Гіперліпідемія.
D. Гіпертрофія м’язів нижніх кінцівок. E. Поліурія.
Причинами розвитку гострого токсичного гепатиту у першу чергу є:
A. Сальмонельоз
B. Вірусний гепатит
C. Застосування антибiотикiв широкого спектру дiї
D. Отруєння солями металiв, фосфорорганiчними речовинами
E. Застосування цитостатикiв

Хворий З., 34 роки страждає на гіпертонічну форму хронічного гломерулонефриту. Які гіпотензивні
засоби найбільш показані при даній формі гломерулонефриту?
A. Інгібітори АПФ
B. Гіпотіазид
C. Резерпин
D. Клофелін
E. Сульфат магнію

У хворого 40 р., який хворіє на артеріальну гіпертензію та отримує відповідне лікування, з’явились
ядуха та набряки нижніх кінцівок. Який з наведених гіпотензивних засобів необхідно додати у
лікувальну схему ? A. Індапамід.
B. Пропранолол.
C. Метилдопа.
D. Доксазозин.
E. Верапаміл.

У хворого, який тривалий час лікується з приводу ревматоїдного артриту, виявлено гіперглікемію.
Який із ЛЗ міг її спричинити?
A. метотрексат
B. тріамцінолон
C. пеніциламін
D. хлорохін
E. диклофенак натрію

Відпускаючи ЛЗ для лікування болю у горлі, що містять анестетики, провізору необхідно


попередити пацієнта про можливість:
A. Забарвлення язика у малиновий колір.
B. Забарвлення зубної емалі.
C. Кислий присмак.
D. Відчуття заніміння у роті.
E. Металевий присмак в роті.

В аптеку звернувся хворий 28 років, який страждає на пептичну виразку 12-палої кишки;
секреторна функція шлунку різко підвищена. Який з наведених ЛЗ бажано призначати у цьому
випадку?
A. Алюмінію гідроксид+магнію гідроксид
B. Атропіну сульфат
C. Метоклопрамід
D. Омепразол
E. Діосмектит

Оберіть лікарський засіб, що застосовується при ІХС для нормалізації холестеринемії шляхом
інгібування активності ГМГ-КоА-редуктази?
A. Стрептокіназа.
B. Натрію аденозинтрифосфат.
C. Інозин.
D. Триметазидин. E. Розувастатин.

Який з перелічених ЛЗ не застосовуються для лікування туберкульозу?


A. Стрептоміцин.
B. Піразинамід.
C. Еритроміцин.
D. Рифампіцин. E. Ізоніазид.

Найчастішими ускладненнями фармакотерапії при застосуванні антибактерійних засобів


пеніцилінового ряду є: A. Імуносупресія.
B. Еритродермія (синдром червоної людини).
C. Гіпертрихоз.
D. Алергійні реакції.
E. Гіпоплазія зубної емалі.

При якому захворюванні ниркова коліка не характерна:


A. Гломерулонефрит
B. Туберкульоз сечовивідних шліхів
C. Нирковокам’яна хвороба
D. Травми сечовивідних шляхів
E. Пухлина сечовивідних шляхів

У хворого хронічний гастрит типу А зі зниженою секреторною функцією, перніціозна анемія. Який
із ЛЗ показаний хворому?
A. епінефрин
B. дротаверин (но-шпа)
C. ціанокобаламін
D. менадіон (вікасол)
E. дифенгідрамін (димедрол)

При збільшенні дози лікарського засобу та при збереженні частоти його застосування час
досягнення рівноважної концентрації:
A. зменшується у двічі
B. залишається без змін
C. зменшується пропорційно до дози
D. зростає у двічі
E. зростає пропорційно дозі х

Лікування, спрямоване на призупинення або сповільнення прогресування захворювання A.


Симптоматичне.
B. Паліативне.
C. Етіотропне.
D. Патогенетичне. E. Комплексне.

Назвіть найбільш характерні симптоми коліту:


A. Тенезми, диспептичний синдром
B. Біль у лівому підребер’ї з іррадіацією в ліве плече
C. Блювота неперетравленою їжею
D. Оперізуючий та абдомінальний біль, нудота
E. Печія, ниючий біль внизу живота

Для патогенетичної терапії хронічного гломерулонефриту використовують усі засоби, окрім:


A. Цитостатиків.
B. Нестероїдних протизапальних засобів.
C. Антимікробних.
D. Глюкокортикоїдів.
E. Гіпотензивних засобів. ?

Назвіть найважливішу ознаку хронічної ниркової недостатності.


A. Гіперкаліємія
B. Артеріальна гіпертензія
C. Підвищення креатиніну в крові
D. Олігурія
E. Протеїнурія

Хворий з діагнозом “гострий панкреатит” госпіталізований у стаціонар. Вибір якого ЛЗ є


патогенетично обґрунтованим ?
A. Метилпреднізолон
B. Панкреатин
C. Кислота амінокапронова
D. Домперидон
E. Апротинін

Гостре інфекційно-запальне ураження лімфоїдної тканини горла, переважно мигдаликів – це: A.


Фарингіт.
B. Ангіна.
C. Ларингіт.
D. Отит.
E. Гайморит.

Яка група лікарських засобів повинна бути використана для лікування хворих на хронічний
гломерулонефрит з високою активністю запального процесу ?
A. Антиагреганти
B. Антібиотики
C. Імунодепресанти
D. Похідні нітрофурану
E. Нестероїдні протизапальні ЛЗ

Хворий Д., 35 р., який нещодавно переніс ангіну, відмітив серцебиття, припухлість і біль колінних
та ліктьових суглобів. При обстеженні у пацієнта виявлено ваду мітрального клапана. Визначте
попередній діагноз.
A. ревматизм
B. ревматоїдний артрит
C. системний червоний вовчак
D. системна склеродермія
E. вузликовий периартериїт

Основним етіологічним чинником пептичної виразки є:


A. Зловживання алкоголем
B. Порушення режиму харчування («сухом’ятка»)
C. Тривале переохолодження
D. Тривалі психоемоційні стреси
E. Інфекція

Після перенесеного інфаркту міокарда хворому рекомендовано застосовувати антиагрегант. В


анамнезі пептична виразка шлунку. Якому засобу Ви надасте перевагу? A. Стрептокіназа.
B. Феніндіон.
C. Ацетилсаліцилова кислота.
D. Гепарин.
E. Дипіридамол.

Хворому на гігантизм лікар призначив засіб етіологічної терапії, але він забув його назву. Про який
засіб йде мова? Выберите один ответ:
A. Цитадрел
B. Перитол
C. Лізодрел
D. Парлодел
E. Конвулекс

Алергічна реакція, при якій антигени надходять ззовні, система комплементу не активується, а в
патогенезі реакції суттєвою є реакція взаємодії антигену з ІgE та ІgG називається:
A. реакція імунних комплексів
B. реакція сповільненої гіперчутливості
C. правильної відповіді немає
D. цитотоксична реакція
E. анафілактична реакція

Пацієнту, в якого з’явилися перші клінічні прояви вітряної віспи, призначили противірусний
лікарський засіб ацикловір. Оберіть вид фармакотерапії, що проводиться:
A. Симптоматична.
B. Замісна.
C. Етіотропна.
D. Патогенетична. E. Профілактична.

Назвіть клінічні ознаки системного червоного вовчака.


A. кісетоподібний рот, амімічне обличчя, мікростомія, остеоліз кінцевих фаланг пальців кистей рук
B. деструкція міжсуглобового хряща, крепітація при рухах, коксартроз, суглобові «миші»
C. ексудативна еритема на щоках, вісцерити, люпус-нефрит, люпус-кардит
D. ксеростомія, ксерофтальмія, хронічний поліартрит
E. кіфоз грудного відділу хребта, самовільні переломи кісток, зменшення росту людини

До симптоматичного лікування діареї належать всі заходи, окрім:


A. Застосування ЛЗ для пероральної регідратації
B. Застосування пребіотиків
C. Застосування антибактерійних засобів
D. Застосування пробіотиків
E. Застосування ентеросорбентів

Назвіть критерії неефективності фармакотерапії:


A. Покращення клінічних симптомів хвороби.
B. Покращення даних додаткових методів обстеження.
C. Покращення якості життя.
D. Позитивна динаміка даних фізикального обстеження. E. Прогресування хвороби і розвиток її
ускладнень.

Допоможіть лікарю вибрати інгаляційний глюкокортикоїд, який застосовують у лікуванні


бронхіальної астми: A. Будесонід.
B. Метилпреднізолон.
C. Гідрокортизон.
D. Преднізолон.
E. Бетаметазон.

Ефективність фармакотерапії при ревматизмі визначають усі з перелічених критерії окрім:


A. відсутність прояву побічних реакцій при застосуванні ЛЗ
B. нормалізація титрів стрептококових антитіл
C. відсутність кардіальних та артралгічних синдромів
D. стабільність морфофункціональних показників за ЕхоКГ з боку клапанів та камер серця E.
нормалізація показників активності запального процесу

Хворий А., 45 років, відчув гострий напад болю в епігастральній ділянці живота, який через деякий
час набула оперізуючого характеру. Після обстеження лікар призначив хворому апротинін довенно
краплинно. Поясніть механізм дії препарату в даній клінічній ситуації.
A. Активація фібринолізу
B. Інгібування активності тромбіну
C. Пригнічення активності протеїназ
D. Зменшення проникливості судин
E. Потенціювання активності простацикліну

Хворий 54 років хворіє на ішемічну хворобу серця. Який з перелічених лікарських засобів володіє
одночасно коронаророзширюючою і антиагрегантною діями?
A. Нітросорбід
B. Аспірин
C. Ніфедипін
D. Лізиноприл
E. Дипіридамол

Хворий на аутоіммунний гепатит, ШОЕ 60 мм/год. Який з наявних ЛЗ підходить для лікування в
даній ситуації? A. Альфа-Інтерферон
B. Преднізолон
C. Нічого з перерахованого
D. Урсодезоксихольова кислота
E. Гептрал

Про який вид фармакотерапії йдеться, коли хворому зі стенокардією лікар призначає симвастатин?
A. Основне лікування.
B. Етіотропне лікування.
C. Замісна терапія.
D. Патогенетичне лікування.
E. Симптоматичне лікування.
В аптеку звернувся хворий з “голодними” болями у верхній половині живота. Про яке
захворювання можна думати?
A. Пептичну виразку
B. Гострий холецистит
C. Хронічний панкреатит
D. Голодування
E. Гепатит

Хворому потрібно призначити синтетичний аналог соматостатину. Який із нижче перелічених


засобів Ви можете запропонувати?
A. Октреотид
B. Меланотропін
C. Перитол
D. Пітуїтрин
E. Бромокриптин

Яка група сечогінних лікарських засобів має найбільш виражений гіпотензивний ефект і
застосовується для постійного контролю артеріального тиску?
A. Петльові
B. Тіазидні
C. Осмотичні
D. Всі відповіді правильні
E. Антагоністи альдостерону

У хворого на цукровий діабет 2 типу ниркова недостатність, кліренс креатиніну менше 60 мл/хв,
який пероральний цукрознижуючих засіб доцільно обирати?
A. Гліквідон
B. Натеглінд
C. Піоглітазон
D. Метформін
E. Глімепірид

У жінки в ІІІ триместрі вагітності в крові гемоглобін 104 г/л, еритроцитопенія, кольоровий показник
- 0,7, залізо сироватки - 22,8 ммоль/л. Назвіть найчастішу причину розвитку цього стану.
A. ендемічний залізодефіцит
B. пригнічення червоного паростка в кістковому мозку
C. дефіцит вітаміну В12
D. переважання витрат заліза над його надходженням
E. крововтрата

При лікуванні артеріальної гіпертензії медикаментозну терапію при стабілізації артеріального тиску
слід:
A. Відмінити через 1 рік.
B. Відмінити через 6 місяців.
C. Відмінити через 1 тиждень.
D. Відмінити через 1 місяць. E. Продовжувати постійно.

При застосуванні якого з нижчеперелічених гіпотензивних лікарських засобів може виникнути


виражена брадикардія?
A. Метопролол
B. Еналаприл
C. Віскен
D. Ніфедипін
E. Доксазозин

Тривалість антибактерійної терапії неускладненої позашпитальної пневмонії складає в середньому:


A. 14 днів.
B. 3-5 днів.
C. 28 днів.
D. 7 днів.
E. 21 день.

У пацієнта К., 58 років, у минулому був набряк Квінке на амоксицилін. Застосування яких ЛЗ
протипоказане через ризик перехресної алергії?
A. еритроміцин
B. ванкоміцин
C. офлоксацин
D. тетрациклін
E. цефазолін

До критеріїв ефективності антибактерійної терапії при лікуванні негоспітальної пневмонії не


належить:
A. Нормалізація частоти дихання.
B. Знижння систолічного АТ нижче 90 мм рт. ст.
C. Зменшення вираженості інтоксикації.
D. Відстність дихальної недостатності.
E. Зниження температури тіла.

Підберіть середньотерапевтичне дозування тиклопідину


A. 100 мг на добу.
B. 75 мг на добу.
C. 250 мг 2 рази на добу.
D. 5 тис. од 4 рази на добу.
E. 1 мг/кг 2 рази на день.

Хворому з артеріальною гіпертензією було призначено один із антигіпертензивних засобів.


Артеріальний тиск нормалізувався, однак хворого почав турбувати постійний сухий кашель. Який з
перерахованих ЛЗ має таку побічну дію.
A. Індапамід.
B. Доксазозин.
C. Пропранолол.
D. Ніфедипін.
E. Периндоприл.

Хворого турбують періодичні напади болю за грудиною, з приводу чого він звернувся в аптеку з
проханням порекомендувати ЛЗ. Після розмови з хворим провізор зробив висновок, що у нього
стенокардія. Яка тривалість больового синдрому при стенокардії?
A. 2-4 години.
B. Від кількох секунд до 20-30 хвилин.
C. 10 годин.
D. 6 годин.
E. 1 година.

Ототоксичність є побічною реакцією антибіотиків групи:


A. Пеніциліни.
B. Цефалоспорини.
C. Макроліди.
D. Карбапенеми.
E. Аміноглікозиди.
Хворому 57 років для усунення приступів стенокардії лікарем в комплексі з аспірином призначено
нітросорбід по 10 мг двічі на добу. Перші 3 тижні ефективність лікування була доброю. Однак,
пізніше для попередження приступів стенокардії (отримання попереднього ефекту) він змушений
приймати нітросорбід по 10 мг 4 рази на добу. Чим пояснити зміну ефективності фармакотерапії у
цьому випадку?
A. Лікова залежність.
B. Розвиток толерантності до ЛЗ.
C. Результат небажаної взаємодії ЛЗ.
D. Розвиток побічної діїі ЛЗ.
E. Відсутність комплаєнсу.

Оберіть групу ЛЗ, що не належить до засобів першого ряду для фармакотерапії хронічної серцевої
недостатності?
A. Блокатори β-адренорецепторів.
B. Тіазидні діуретики.
C. Інгібітори АПФ.
D. Блокатори рецепторів ангіотензину ІІ.
E. E. Серцеві глікозиди.

Який із перелічених ЛЗ застосовують при інфаркті міокарда для проведення тромболітичної


терапії?
A. Ацетилсаліцилова кислота.
B. Феніндіон.
C. Тиклопідин.
D. Гепарин.
E. Урокіназа.

Оберіть мікроорганізм, що є збудником сифілісу:


A. бліда трепонема
B. гонокок
C. шигелла зонне
D. бета-гемолітичний стрептокок
E. паличка Перфрінгенс

Для посилення антиагрегантного ефекту при лікуванні дрібно вогнищевого інфаркту міокарда
хворому призначено «Плавікс». Назвіть діючу речовину цього лікарського засобу.
A. Тиклопідин
B. Дипіридамол
C. Клопідогрель
D. Абсіксимаб
E. Грудин

Хворому М., який місяць тому переніс гострий інфаркт міокарда, для профілактики
тромбоутворення призначено ацетилсаліцилову кислоту. В якій дозі слід рекомендувати цей ЛЗ у
такій клінічній ситуації?
A. 3000 мг.
B. 100-300 мг.
C. 1000-1500 мг.
D. 750 мг.
E. 500 мг. Вопрос 40 У якого із перелічених нестероїдних протизапальних засобів найслабше
проявляється ульцерогенна дія? A. піроксикам
B. мелоксикам
C. диклофенак
D. ацетилсаліцилова кислота
E. індометацин

Лікарськими засобами для лікування генералізованої епілепсії з абсансами є


A. ламотріджин, фенобарбітал
B. вальпроєва кислота, клоназепам
C. карбамазепін, вальпроєва кислота
D. вальпроєва кислота, етосуксимід
E. вігабатрин, топірамат

На прийом до лікаря звернулась пацієнтка 45 р. зі скаргами на біль голови в потиличній ділянці,


періодичне підвищення АТ, зниження працездатності. Хворіє декілька тижнів після
психоемоційного навантаження. На момент огляду АТ становить 145/95 мм. рт. ст., інших
об’єктивних ознак захворювання не виявлено. Який попередній діагноз можна поставити у цьому
випадку?
A. Вторинна артеріальна гіпертензія.
B. АГ І стадії.
C. Жодний з перелічених варіантів.
D. Синдром «білого халата».
E. АГ ІІ стадії.

Вагітна жінка захворіла на флегмонозну форму ангіни. Який з нижченазваних антибактеріальних


засобів доцільно їй призначити: A. Цефотаксим.
B. Офлоксацин.
C. Левоміцетин.
D. Тетрацикліну гідрохлорид.
E. Гентаміцину сульфат.

Хворий, 32 років, госпіталізований з діагнозом виразкова хвороба 12-палої кишки. У комплексній


терапії лікар призначив омепразол, якого в аптеці не виявилось. Яким лікарським засобом можна
його замінити? A. Метронідазол
B. Вісмуту субцитрат
C. Фосфалюгель
D. Мізопростол
E. Лансопразол

Лікарським засобом вибору для проведення антиагрегантної терапії при стабільній стенокардії у
випадку протипоказань до ацетилсаліцилової кислоти є:
A. Пентоксифілін.
B. Дипіридамол.
C. Тиклопідин.
D. Клопідогрель.
E. Мелоксикам.

До симптомів передозування нітрогліцерином не належить:


A. Підвищення внутрішньочерепного тиску.
B. Метгемоглобінемія.
C. Артеріальна гіпертензія.
D. Біль голови, запаморочення.
E. Артеріальна гіпотензія, тахікардія.
Виберіть найбільш оптимальні лікарські засоби, для полікомпонентної фармакотерапії
туберкульозу при застосуванні ізоніазиду як ЛЗ першого вибору:
A. Парацетамол, гентаміцин
B. Кетоконазол, дуовіт
C. Норфлоксацин, ацетилсаліцилова кислота
D. Альмагель, мефенамінова кислота
E. Глутамінова кислота, піридоксин

Вкажіть, який фармакокінетичний параметр визначає частоту застосування лікарського засобу: A.


маса пацієнта
B. період напіввиведення
C. швидкість метаболізму
D. вік пацієнта
E. період напівабсорбції

Для стенокардії не характерно:


A. Знеболюючий ефект нітрогліцерину.
B. Загрудинна локалізація болю.
C. Тривалість болю 5-10 хв.
D. Колючий характер болю.
E. Зв'язок болю з фізичним навантаженням .

До критеріїв ефективності фармакотерапії з метою ліковування туберкульозу легень не належить:


A. Відновлення функціональних можливостей організму та працездатності.
B. Відсутність побічних ефектів від застосування протитуберкульозних ЛЗ.
C. Загоєння каверн в легенях.
D. Стійке припинення бактеріовиділення.
E. Зникнення клінічних та лабораторних ознак туберкульозного запалення.

В групу непрямих антикоагулянтів не входить:


A. Фепромарон
B. Дикумарин
C. Гірудин
D. Варфарин
E. Фенілін

Розрахувати константу елімінації пефлоксацину, якщо відомо, що його період напіввиведення


становить 8 год.
A. 0,031 год-1
B. 0,014 год-1
C. 0,02 год-1
D. 0,24 год-1
E. 0,087 год-1

До блокаторів кальцієвих каналів належать всі з перелічених лікарських засобів, окрім:


A. Верапаміл
B. Лозартан
C. Дилтіазем
D. Амлодипін
E. Лекарнідипін

Найчастішим симптомом серцевої недостатності є:


A. Олігурія.
B. Тахікардія.
C. Гепатомегалія.
D. Задишка.
E. Спленомегалія.

Характерною побічною реацією при застосуванні рифампіцину є:


A. Підвищення в’язкості харкотиння.
B. Ульцерогенна дія.
C. Ототоксична дія.
D. Ретробульбарний неврит.
E. Забарвлення сечі та харкотиння в червоний колір.

Як правильно вводити засоби інсуліну за базис-болюсною схемою?


A. базисний інсулін середньої тривалості дії 2 рази на день, інсулін короткої дії перед кожним
прийманням їжі
B. базисний інсулін середньої тривалості дії 1 раз на день, інсулін тривалої дії перед кожним
прийманням їжі
C. базисний інсулін середньої тривалості дії перед сном, інсулін ультракороткої дії зранку і ввечері
D. базисний інсулін середньої тривалості дії та інсулін короткої дії зранку
E. базисний інсулін тривалої дії 2 рази на день, інсулін короткої дії перед кожним прийманням їжі

Який вид фармакотерапії використовується для лікування хвороби Аддісона


A. Замісна
B. Патогенетична
C. Симптоматична
D. Правильна відповідь відсутня
E. Етіологічна

У хворої 36 років виявлено атипову пневмонію, яка викликана хламідійною інфекцією. Який
антибактеріальний засіб доцільно рекомендувати у даному випадку ?
A. Цефуроксим.
B. Ванкоміцин.
C. Амоксицилін.
D. Ампіцилін.
E. Рокситроміцин.

Хворому з гіперацидним гастритом назначено ранітидин. Який механізм дії цього лікарського
засобу?
A. Блокує фермент Н+-К+-АТФ-азу, яка відповідає за продукцію соляної кислоти
B. Нейтралізує соляну кислоту в шлунку
C. Блокує Н2-гістамінові рецептори шлунка
D. Викликає дегідратацію або часткову коагуляцію тканних білків чи ранового ексудату і утворюють
альбумінати, які захищають слизову оболонку
E. Утворює плівку з колоїду, яка захищає чутливі нервові закінчення від дії соляної кислоти

Назвіть ваду серця, яка найчастіше розвивається при ревматизмі.


А. недостатність аортального клапану
В. стеноз гирла аорти
С. недостатність трикуспідального клапану
D. недостатність мітрального клапану
E. стеноз трикуспідального клапану
Найчастіша локалізація ураження суглобів при ревматоїдному артриті?
А. кульшові суглоби
В. суглоби гомілково- стопові
С. колінні суглоби та суглоби стопи
D. п’ясткофалангові суглоби кистей рук
E. плечові суглоби та хребет

При нападі стенокардії біль найчастіше локалізується:


А. Під правою лопаткою
В. У правій половині
С. У епігастральній ділянці
D. В правій підпахвинній ділянці
E. За грудиною

У хворого діагностували інфаркт міокарда з тромбозом лівої коронарної артерії. Яка з означених
фармакологічних груп ЛЗ використосувається для відновлення кровотоку в коронарних артеріях у
перші 6-12 год. захворювання?
А. Курантил
В. Інгібітори АПФ
С. Бета-адреноблокатори
D. Гепарин
E. Активатори фібринолізу

До критеріїв ефективності фармакотерапії з метою лікування туберкульозу легень не належить?


А. Стійке припинення бактеріовиділення
В. Відсутність побічних дій від застосування протитуберкульозних ЛЗ
С. Зникнення клінічних та лабораторних ознак туберкульозного запалення
D. Відновлення функціональних можливостей організму та працездатності E.
Загоєння каверн в легенях

При закапуванні в ніс хворому з гострим ринітом розчину ефедрин гідрохлориду розвинулося
явище тахіфілаксії. Чим характеризується дана реакція:
А. Фізичною залежністю
В. Швидким звиканням
С. Психічною залежністю
D. Повільним звиканням
E. Послабленням процесу всмоктування

Хворому з генералізованою формою сальмонельозу була призначена антибактеріальна терапія.


Через деякий час при проведенні загального аналізу крові була виявлена ретикуло- та
гранулоцитопенія, відмічалось деяке зниження кількості еритроцитів. При призначенні якого
антибіотику найбільш вірогідна подібна зміна картини крові?
А. цефтріаксон
В. хлорамфенікол
С. гентаміцину сульфат
D. тетрациклін
E. амоксицилін
Який з наведених ЛЗ найдоцільніше застосувати для лікування гострого бронхіту, що
супроводжується продуктивним кашлем? A. Кодеїну сульфат.
B. Окселадин.
C. Бутамірат.
D. Преноксдіазин.
E. Ердостеїн.

Пацієнт Т., 78 р., страждає на хворобу Паркінсона. Приймає леводопу у комбінації з карбідопою 5
разів на добу. Надайте коректну фармацевтичну опіку стосовно дієти пацієнта
A. обмеження жирної їжі, дієта збагачена білком та клітковиною
B. обмеження білку і. рівномірний його розподіл в раціоні; збагачення кальцієм та вітаміном D
C. обмеження легкозасвоюваних вуглеводів та цитрусових соків
D. особливостей дієти при застосуванні леводопи у комбінації з карбідопою немає
E. кетогенна дієта із підвищеним вмістом жирів, зниженим вмістом вуглеводів та білку

Люпус-кардит (перикардит, міокардит та ендокардит із розвитком вад серця) є проявом:


A. вірусного міокардиту
B. гострої ревматичної гарячки
C. ревматоїдного артриту
D. дерматоміозиту
E. системного червоного вовчака

Який з перелічених лікарських засобів не виявляє ульцерогенної дії?


A. Резерпін
B. Індометацин
C. Преднізолон
D. Ацетилсаліцилова кислота
E. Пропранолол

Гіперсекреторний хронічний гастрит – це:


A. Гастрит типу В
B. Еозинофільний
C. Гастрит типу С
D. Гастрит типу А
E. Хімічний

Лікарським засобом, що НЕ належить до інгібіторів ГМГ-КоА-редуктази є:


A. Правастатин.
B. Розувастатин.
C. Симвастатин.
D. Холестирамін.
E. Ловастатин.

Ускладненням гострого гломерулонефриту не є:


A. Ниркові конкременти
B. Гостра лівошлуночкова недостатність
C. Артеріальна гіпертензія
D. Гостра ниркова недостатність
E. Нефротичний синдром

Назвіть вид фармакотерапії,який використовується при лікуванні гіпофізарного нанізму?


A. Патогенетична
B. Симптоматична
C. Замісна
D. Етіологічна
E. Профілактична

В процесі діагностичного пошуку при верифікації діагнозу гепатиту, цирозу, найбiльшу цінність
має:
A. Бiопсiя печiнки
B. Ангiографiя печiнки
C. Скенування печiнки
D. Гепатографiя
E. Ультразвукове дослiдження

Фронтит – це:
A. Запалення глотки.
B. Запалення євстахієвої труби.
C. Запалення лобової пазухи носа.
D. Запалення решітчастого лабіринта носа. E. Запалення гортані.

Допоможіть обрати антибіотик місцевої дії, який можна застосовувати шляхом інгаляцій після
перенесеної тонзилектомії:
A. Макропен
B. Амоксицилін
C. Фузафунгін
D. Ломефлоксацин
E. Цефтріаксон

Пацієнту з закладенням носа для відновлення носового дихання призначили деконгестант


оксиметазолін, що є прикладом:: A. Етіотропної фармакотерапії.
B. Патогенетичної фармакотерапії.
C. Симптоматичної фармакотерапії.
D. Профілактичної фармакотерапії.
E. Замісної фармакотерапії.

Коли слiд вживати холiнолiтичнi лікарські засоби?


A. Тiльки на нiч
B. В часи приймання їжi
C. Через 1 - 2 години пiсля приймання їжi
D. Через 30 хвилин пiсля приймання їжi
E. За 30 хвилин до приймання їжi

Який із перелічених лікарських засобів може маскувати ознаки кровотечі у верхніх відділах ШКТ
(чорне забарвлення стільця)?
A. Сукральфат
B. Алюмінію гідроксид+магнію гідроксид
C. Фамотидин
D. Пірензепін
E. Вісмуту субцитрат колоїдний

Противірусна терапія вірусного бронхіту ефективна, якщо лікування розпочинають


A. Противірусна фармакотерапія неефективна.
B. У перші 2-3 дні захворювання.
C. Противірусна фармакотерапія ефективна лише в 1 день захворювання.
D. Через 5 днів після початку захворювання.
E. Через 7 днів від початку захворювання.
Розрахувати константу елімінації пефлоксацину, якщо відомо, що його період напіввиведення
становить 8 год. A. 0,02 год-1
B. 0,031 год-1
C. 0,014 год-1
D. 0,24 год-1
E. 0,087 год-1

Яка група лікарських засобів із перелічених не викликає «синдрому відміни» при раптовому
припиненні їх вживання?
A. Клофелін.
B. Бета-адреноблокатори.
C. Прямі антикоагулянти.
D. Глюкокортикоїди.
E. Блокатори протонної помпи.

ЛЗ з групи М-холіноблокаторів, що відзначається селективною дією на дихальні шляхи це:


A. Ізадрин
B. Сальбутамол
C. Атропіну сульфат
D. Іпратронію бромід
E. Ефедрину хлорид

Які з перелічених побічних реакцій не характерні для глюкортикостероїдів?


A. гіперглікемія
B. синдром Іценко-Кушинга
C. порушення серцевого ритму
D. остеопороз
E. гіпертензія

Лікар-початківець для лікування гіпертонічної хвороби призначив хворому 56 років комбіновану


фармакотерапію. Схема лікування складається з трьох різних лікарських засобів протягом доби, з
яких два належать до однієї групи гіпотензивних засобів, але мають деякі відмінності
фармакодинамічних показників. Відмічається повільне покращення самопочуття і стабілізація
артеріального тиску. Дайте оцінку фармакотерапії. A. Існує ризик розвитку лікової залежності.
B. Раціональна фармакотерапія.
C. Нераціональна фармакотерапія.
D. Існує підвищений ризик побічної дії від взаємодії ЛЗ.
E. Поліпрагмазія???

Термін «вторинна артеріальна гіпертензія» означає: A.


Гіпертензію, причину якої можна встановити.
B. Артеріальна гіпертензія після стресу або фізичного навантаження.
C. Підвищення тільки систолічного АТ.
D. Артеріальну гіпертензію у осіб похилого віку.
E. Гіпертензію, причину якої неможливо встановити.

Хворий на ревматизм 48 р., тривало лікувався з приводу серцевої недостатності, через 14 діб
з’явився сухий кашель. Побічна дія якого ЛЗ, найімовірніше, могла проявитись у хворого?:
A. Еналаприлу.
B. Бісопрололу.
C. Дигоксину.
D. Гідрохлортіазиду.
E. Фуросеміду.

В аптеку звернувся пацієнт, який скаржиться на першіння у горлі, біль при ковтанні, підвищення
температури тіла до 38˚С. При огляді запалення мигдаликів відсутнє. Яке захворювання Ви можете
запідозрити?
A. Катаральна ангіна.
B. Фарингіт.
C. Тонзиліт.
D. Ларингіт.
E. Лакунарна ангіна.

Хворому на гострий пієлонефрит, викликаний коковою інфекцією, лікар збирається призначити


антибіотик. Антибіотики пеніцилінового ряду викликають у хворого алергічну реакцію. Порадьте л
ікарю, який антибіотик можна рекомендувати для лікування хворого?
A. Оксацилін
B. Цефуроксим
C. Феноксиметилпеніцилін
D. Азитроміцин
E. Цефтазидим

Про який вид фармакотерапії йдеться, коли хворому з ревматизмом лікар призначає пеніцілін?
A. Патогенетичне лікування.
B. Симптоматичне лікування.
C. Етіотропне лікування.
D. Базове лікування.
E. Замісне лікування.

Вкажіть, яке явище буде спостерігатися при застосуванні лікарського засобу до моменту виведення
попередньої дози:
A. резистентність
B. кумуляція
C. відсутність фармакотерапевтичного ефекту
D. антагонізм
E. рецидив

«Синдром відміни»- це Побічна дія, обумовлена


раптовим впливом приймання
лз
АнтибактеріальнимЛЗ вибору для лікування синусів є Амоксицилін
Антикоагулянтна дія гепарину припиняється за допомогою: Протаміну сульфату
Апластична анемія – це захворювання що характеризується втратою здатності кісткового
мозку достатню кількість
еритроцитів

Базисна ( протизапальна) терапія бронхіальної астми передбачає Глюкокортикостероїдів


застосування
Безпосередньою причиною виникнення болю при нирковій кольці є: Перешкода відтоку сечі з
ниркової миски в сечовід
Більшість лікарських засобів елімінують з організму Нирками
Біотрансформація лз в основному відбувається в печінці
В аптеку звернувся пацієнт, який втрату голосу, сухий подразнюючий Ларингіт
кашель і, підвищення температури тіла до 37,4. Яке захворювання ви
можете запідозрити?
В аптеку звернувся пацієнт, який скаржиться на пердіння вгорлі, біль Фарингіт
при ковтанні, підвищення температури тіла до 38 . При огляді
запалення мигдаликів відсутнє. Яке захворювання Ви можете
запідозрити?
В І триместрі вагітності при розвитку пієлонефриту показана лише одна А)Аміноглікозиди В)
група антибактеріальних засобів Вторхінолони Г)Сульфаніламіди
Д) Цефалоспорини Е)
Пеніциліни
В комплексне лікування не ускладненого туберкульозу легень Вітаміни групи В
включають
Вкажіть етапи встановлення діагнозу:
Вкажіть стан та захворювання, при яких зменшується здатність Цироз печінки, опіки, похилий
лікарських речовин зв'язуватись з білками плазми вік
Вкажіть чинник який впливає на процес абсорбції
Вкажіть чинник, який визначає ризик небажаного впливу лікарського Проникність лікарського засобу
засобу на організм матері та плоду під час вагітності через плаценту
Гематурія - це Наявність крові в сечі
Гостре бактеріальне захворювання ниркових канальців, мисочок і мисок Гострий пієлонефрит
- це
Гостре бактеріальне захворювання ниркових мисок,мисочок і канальців Гострий гестаційний
у вагітних жінок і породіль - це пієлонефрит
Діагноз( розпізнання хвороби) це Коротке заключення лікаря про
суть хвороби та стан пацієнта
Діагностичним критерієм при захворюваннях щитоподібної залози Рівень ТТГ та Т3 і Т4
вважають
Для гострої пневмонії НЕ характерні наступні симптом Постійний сухий кашель
Для обструктивного бронхіту НЕ характерно кровяне харкотиння
Для покращення відходження мокроти, дренажної функції бронхів, Ацетилцистеїн
хворій 52 років, з бронхіальною астмою призначений лікарській засіб.
Через деякий час у хворої зявилася сльозотеча, ринорея , лоскіт в горлі.
Який засіб отримувала хвора?
Для фармакотерапії ранньої стадії хвороби Паркінсона застосовують: Усі відповіді вірні
До групи інгібіторів АПФ належить: Периндоприл
До додаткових методівдослідження хворих НЕ належить: Огляд хворого
До завдань ефективної фармакотерапії НЕ належить:
До лікаря звернувся хворий 70 р., з приводу підвищеного артеріального Доксазозин
тиску. З анамнезу відомо про наявність доброякісної гіперплазії
передміхурової залози. Який ЛЗ бажано призначити цьому хворому?
До основних фармакокінетичних процесів належать усі відповіді правильні
До пероральних цукрознижуючих засобів належать всі групи, окрім Препарати інсуліну
До принципів раціональної фармакотерапіїНЕ належить Фармакоекономічна
доцільність
До симптомів цукрового діабету належать всі перелічені, окрім Тремор повік
До тіазидоподібних діуретиків належить: Індапамід
До ускладнень цукрового діабету належать Всі відповіді вірні
До факторів, щовпливають на процес біотрансформації ЛЗ у немовлят Недостатня сформованість
належать ферментативних систем, що
метаболізують ліки
До якої групи гіпотензивних ЛЗ належить моксонідин Антагоністи імідазолінових
рецепторів.
До якої групи ЛЗ належить клопідогрель? Антиагреганти
До якої групи ЛЗ, що діють на систему зсідання крові, належить тромболітики
альтеплаза?
Допоможіть лікарю вибрати інгаляційний глюкокортикоїд, який Будесонід
застосовують у лікуванні бронхіальної астми

Допоможіть молодому колезі обрати симптом не характерний для Прискорення мови


хвороби Паркінсона.
Етіологічними причинами синдрому Іценко-Кушинга є всі перелічені, Надлишкова секреція АКТГ в
окрім аденогіпофізі
Жінка 25 років III триместрі вагітності. Скарги на втомлюваність, нудоту, пероральні ЛЗ заліза в
гемоглобін 105г/л. Якізасоби для лікування будуть найефективнішими розчині
для неї, якщо вона з дитиною не може ковтати таблетки?
З якою сполукою може з’єднуватися і транспортувати її до клітини білок Усі відповіді вірні
еритроцитів гемоглобін
За метаболізм лікарських засобів в основному відповідає: Печінка, нирки
Зазначте чинники з якимипов’язаний ступінь всмоктування лікарських
засобів
Запаленняпорожнини верхньої пазухи носа Гайморит

Запалення слизової оболонки гортанні, яке часто супроводжується Ларингіт


частковою або повною втратою голосу - це
Застосування ЛЗ з метою лікування хворого це- фармакотерапія
Комбіноване застосування В2-агоністів (фенотерол) та холінолітиків Для посилення бронхолітичного
(іпратропію бромід) у хворих на бронхіальну астму доцільне; ефекту та його положення
Летальна доза заліза 180-300 мг/кг маси тіла
ЛЗ першої черги для лікування генералізованої епілепсії з абсансами є: Вальпроєва кислота,
етосуксимід
ЛЗ якої фармакотерапевтичної групи будуть застосовані в якості В2-агоністи короткої дії
«швидкої допомоги» для зняття нападу гострого бронхоспазму?
Лікарськими засобами першого вибору для лікування поза шпитальних амоксициклін або
пневмоній у пацієнтів 1 категорії є; кларитроміцин перорально
Лікарськими засобами першої черги для парціальної епілепсії є: Карбамазепін і івальпроєва
кислота
Лікування залізодефіцитної анемії може передбачувати Включення в раціон
залізовмісних продуктів
Лікування, що призначається хворому з метою впливу на причину етіотропне
захворювання
Метаболіз багатьох лікарських засобів в організмі залежить від Активності системи цитохрому
P450
Молода жінка знепритомніла в тісному приміщенні. Щоб привести її до Рефлекторна дія
тями лікар дав понюхати розчин аміаку. Яка точка прикладання
фармакотерапії цим ЛЗ?
На прийом до лікаря звернулась пацієнтка 45 рю зі скаргами на біль АТ І стадія
голови в потиличній ділянці, періодичне підвищення АТ, зниження
працездатності. Хворіє декілька тижнів після псиоемоційного
навантаження. На момент огляду АТ становить 145/95 мм рт. ст., інших
об’єктивних ознак захворювання не виявлено. Який попередній діагноз
можна поставити у цьому випадку?
Назвіть антибіотик з групи цефалоспоринів 3-го покоління для Цефіксим
пероральної терапії
Назвіть вид фармакотерапії, який використовується при лікуванні Замісна
гіпофізарного нанізму
Назвіть групу лікарських засобів, раптове припинення вживання якої глюкокортикоїди
може спричинити розвиток «синдрому відміни»
Назвіть додаткові методи обстеження хворого Лабораторне та
інструментальне дослідження
Назвіть другий етап фармакотерапії Проведення діагностичного
процесу з метою контролю
клінічних данних
Назвіть критерії неефективності фармакотерапії Прогресування хвороби і
розвиток її ускладнень
Назвіть ЛЗ, який належить до групи інгаляційних В2-адреноміметиків Формотерол
тривалої дії
Назвіть основний критерій компенсації цукрового діабету Рівень глікозильованого
гемоглобіну
Назвіть основну ланку патогенезу пієлонефриту Запально-інфекційне ураження
канальців
Назвіть сучасні методи лікування акромегалії Всі відповіді вірні
Найбільш ефективний метод усунення азотемії при гострій нирковій А) Застосування фуросеміду
недостатності В)Обмеження білків в раціоні Г)
Прийом фіто лізину Д)
Гемодіаліз Е) Вуглеводно-
жирова дієта
Найефективнішим методом діагностики епілепсії є: Електроенцефалографія
Накопичення ЛЗ при його повторних веденнях- це кумуляція
Оберіть антибактеріальний ЛЗ широкого спектру дії для лікування Рифампіцин
туберкульозної інфекції
Оберіть із нижче наведених лікарських засобів, той який є інгібітором Хлодітан
біосинтезу гормонів у корі наднирників
Оберіть коректний шлях введення туберкуліну для проведення проби Внутрішньошкірний
Манту:
Оберіть ЛЗ що можуть викликати Паркінсонізм Усі вірні
Оберіть ЛЗ, що можуть спричинити виникнення паркінсонізму. Усі відповіді вірні
Оберіть невірне твердження стосовно ЛЗ амантадину: Усі відповіді вірні
Оберіть невірне твердження стосовно ЛЗ тригексифенідилу (циклодол) Призначають переважно при
ригідності та гіпокінезії.
Оберіть оптимальний шлях введення ЛЗ немовлятам Внутрішньовенний
Оберіть правильний принцип фармакотерапії протиепілептичними Монотерапія
засобами.
Оберіть серед наведених симптомів НЕ характерний для інтоксикації Гіперетермія
залізом
Оберіть серед перелічених протитуберкульозних ЛЗ першого вибору Ізоніазид
при лікочутливому туберкульозі
Оберіть шлях введення антибактеріальних ЛЗ для лікування поза пероральний
шпитальної пневмонії у пацієнтів 1 категорії
Оберіть, що не характерно для дитячого організму Значна кількість підшкірної
жирової клітковини
Одним із етіологічних чинників, які спричиняють розвиток хвороби Туберкульоз
Аддісона є
Основним антитиреоїдним засобом, який застосовується при Тіамазол
гіперфункції щитоподібної залози є
Основним засобомфармакотерапії цукрового діабету 1 типу є Препарати інсуліну
Основним засобом замісної гормонотерапії при гіпотиреозі є L-тироксин
Пальпація- це метод обстеження хворого, що ґрунтується на Дотику та промацувані
поверхневих тканин і органів,
що знаходяться глибше
Патогенетичною основою розвитку цукрового діабету 2 типу є Інсулінорезистентність,
інсулінодефіцит
Пацієнт поскаржився на відчуття неприємного запаху, після чого Великий напад тоніко-
втратив свідомив, впав. Спостерігались тонічні і клонічні скорочення клонічних судом.
м’язів тулуба, з рота виділялась піна забарвлена кров’ю. Потім
мимовільне сечовипускання і глибокий сон. Після сну пацієнт не може
розповісти що з ним сталось. Який це вид епілептичного нападу.
Пацієнт Т. страждає на хворобу Паркінсона. Приймає леводопу у Обмеження білку і рівномірний
комбінації з карбідопою 5 разів на добу. Надайте конкретну його розподіл у раціоні,
фармацевтичну опіку стовоно дієти пацієнта. збагаченням кальцію та
вітаміном Д.
Пацієнту з закладенням носа для відновлення носового дихання Симптоматичної
призначили деконгестант оксиметазолін, що є прикладом: фармакотерапії
Пацієнту, в якого з’явилися перші клінічні прояви вітряної віспи, етіотропна
призначили противірусний лз ацикловір. Оберіть вид фармакотерапії
що проводиться
Перелічіть ЛЗ другого роду для лікування туберкульозу (при доведеній Канаміцин, стюнамід,
резистентності до протитуберкульозних ЛЗ 1роду) циклосерин, капресоміцин,
ПАСК, ципрофлюксацин
Під час вагітності при лікуванні гострого гестаційного пієлонефриту Стрептоміцин, тетрациклін
абсолютно протипоказані
Позашпитальні пневмонії 2-Ї категорії – це пневмонії У хворих із супутньою
патологією ( цукровий діабет,
хвороби ССС) та у осіб віком 60
років і старших ( не важкий
перебіг)
Поліморбність- це Наявність водночас кількох
захворювань у пацієнта
Поняття комплаєнс означає: Готовність пацієнта виконувати
всі рекомендації лікаря
Порадьте, який агоніст В2- адренорецепторів застосовують для Сальметерол
усунення нападів бронхіальної астми
Порекомендуйте пацієнту з ангіною жарознижуючий ЛЗ, який проявляє Мефенамінову кислоту
імуностимулюючу активність
Поясніть лікарю-інтерну з якою метою до леводопи додаютю Для зменшення периферійного
карбідопу, бенсеразид і … метаболізму леводопи.
Поясніть молодому колезі, що є патологічною ланкою у розвитку Прогресуюча дегенерація
хвороби Паркінсона? допамінергічних нейронів.
Пресистемна елімінація лікарського засобу ефект першого проходження
При відпуску з аптеки вушних крапель пацієнта слід повідомити, що: Краплі слід нагріти до
температури тіла
При лікуванні гострого пієлонефриту не показані наступні з перелічених Глюкокортикоїди
груп ЛЗ
При лікуванні пієлонефриту вагітних в II –III триместрі при Макролідів
непереносимості антибіотиків пеніцилінового ряду можливе
застосування
При якій супутній патології застосування ксилометазоліну та інших Глаукома
симпатоміметиків не рекомендовано?
Причиною зниження ефективності пероральних ЛЗ заліза у пацієнтів ↓всмоктування у кишківнику
похилого та старечого віку є
Причиною хвороби Іценко-Кушинга є Гіперпродукція АКТГв
аденогіпофізі
Про який вид фармакотерапії йдеться, коли хворому з туберкульозом Симптоматичне лікування
легень лікар призначає лібексим?
Раціональна фармакотерапія базується на Усі відповіді правильні
Режим дозування еноксапарину для профілактики тромботичних 40 мг 1 раз в день
післяопераційних ускладнень:
Резистентність- це Стійкість організму до дії
певних лз
Розрахувати дозу лікарського засобу для дітей можна використовуючи Усі відповіді вірні
Розрахувати константу елімінації офлоксацину,якщо відомо,що його 0,116год-1
період напіввиведення становить 6 год
Розрахувати константу елімінації пефлоксацину,якщо відомо,що його 0,087год-1
період напіввиведення становить 8 год
Розрахувати константу елімінації триметазину,якщо відомо,що його 0,154год-1
період напіввиведення становить 4,5 год
Серед факторів, що призводять до виникнення анемій оберіть Надлишковий вміст заліза в
нехарактерні дієті
Симпатоміметики для лікування риніту призначають з обережністю при Артеріальній гіпертензії
Скільки білка повинно бути в сечі здорової людини? Не повинно бути
Схема введення препаратів інсуліну, яка імітує фізіологічну секрецію Базис-болюсна
називається
Термін «вторинна артеріальна гірпертензія»означає: Гіпертензію, причину якої
можна встановити.
Термін «есенціальна гіпертензія» означає? Первинну артеріальну
гіпертензію, причина якої
невідома.
Тривалість антибактеріальної терапії неускладненої поза шпитальної 7 днів
пневмонії складає в середньому :
Тривалість антибіотикотерапії при лікуванні лакунарної ангіни 10 днів
становить:
У жінки 25р яка годує груддю, гемоглобін 108г/л, сиров. Залізо 8.5 ЛЗ заліза для перорального
ммоль/л. Оберіть основний метод лікування. застосування
У жінки, яка тривалий час лікується зприводу інфільтративно- Стрептоміцин
вогнищевого туберкульозу,різко знизився слух. Який з перерахованих
ЛЗ міг спричинити таку побічну дію?
У пацієнта Л. 3р. діагностовано сфеноїдин. Із запропонованих Тетрациклін
антибактерійних ЛЗ оберіть найбезпечніший у цій клінічній ситуації.
У пацієнтів похилого та старечого вікукількість альбуміну плазми крові, Знижена
як правило
У пацієнтів старшого віку спостерігається Усі відповіді вірні (Зростання
ризику виникнення токсичної
дії ліків, Зниження видільної
функції нирок, Зменшення
кількості нефронів, Зниження
ниркового кровопостачання)
У хворого вперше виник приступ бронхіальної астми. З якого ЛЗ Інгаляційного В2-агоністу
найдоцільніше починати лікування хворого?
У хворого К, 53 років, бронхіальна астма та стенокардія напруги. Який Орципреналін
лікарській засіб із перелічених нижче протипоказаний в цьому випадку?
У хворого на гострий пієлонефрит інтенсивний біль у попереку, часті Антибіотики
позиви до сечовипускання, температура тіла 39°С, піурія,Які ЛЗ в першу
чергу слід призначати хворому?
У хворого на позашпитальну пневмонію в анамнезі – алергія на Лінкоміцин
ампіцилін у вигляді набряку Квінке. Який з перелічених
антибактеріальних ЛЗ інших груп не слід призначати?
У хворого скарг на інтенсивний приступоподібний інтенсивний біль Ниркова коліка
внизу живота з іррадіацією в спину, нудоту. Загальний аналіз сечі: білок
0,006 г/л, вполі зору, епітелій перехідний і нирковий по 2-3, лейкоцити
8-10 місцями групами по 20-30, еритроцити не змінені 80-120, критали
сечової кислоти окремо і зростки- багато. Яке захворювання можна
запідозрити?
У хворого хронічний гепатит типу А зі зниженою секреторною функцією Ціанокобаламін
… анемія. Який із ЛЗ показаний хворому?
У хворої виник епілептичний напад, однак при цьому свідомість була Прості парціальні
збереженою і вона могла описати свої відчуття. Вкажіть до якого виду
нападів це характерно?
Упацієнтів похилого віку процеси фармакокінетики лікарських засобів Поліморбідність
залежать від низки факторів, зокрема наявності водночас кількох
патологій, що називається
Упацієнтів похилого та старечого вікупри в/м та п/ш введенні Сповільнене
всмоктування ЛЗ, як правило
Ускладнення гострого гломерулонефриту не є? Ниркові конкременти
Фармакотерапію ранньої стадії хвороби Паркінсона у пацієнтів старше Леводопи
75 років починають із призначення наступного ЛЗ:
Характерними клінічними ознаками акромегалії є Збільшення кистей, стоп,
«грубі» риси обличчя, зміна
голосу, порушення обміну
речовин
Характерними клінічними ознаками хвороби Іценко-Кушинга є Ожиріння тулоба, стрії на шкірі,
артеріальна гіпертензія
Характерною ознакою хвороби Аддісона є Прогресуюче потемніння шкіри
та слизових оболонок
Хвора К. вагітність 1 триместр, встановлено діагноз гострого Амокицилін
пієлонефриту. Який антибіотик є ЛЗ вибору для лікування даної
пацієнтки?
Хвора скаржиться на відчуття печіння язика. При огляді язик Вітамін В-12 дефіцитна анемія
малинового кольору , блискучий, гладкий. Гемоглобін 105г/л,
еритроцитопенія, еритроцити великих розмірів, колірний показник –
1,3. Про яке захворювання слід думати?
Хворий 52 років переніс інфаркт мікарда. Для попередження 100 мг
тромбоутворення призначена ацетилсаліцилова кислота. Визначене
раціональну добову дозу
Хворий звернувся до лікаря зі скаргами на пронос, переймоподібний симптоматична
біль в животі, нудоту. Після обстеження діагностовано амебну
дизентерію. Лікарем призначено тетрациклін. Який вид фармакотерапії
застосовано лікарем у даному випадку?
Хворий на гострий бронхіт скаржиться на кашель з виділенням помірної відмінити ацетилцистеїн
кількості харкотиння. Лікар призначив бромгексин, ацетил цистеїн та
преноксдіазин. Яка корекція фармакотерапії необхідна у цьому
випадку?
Хворий на залізодефіцитну анемію лікар призначив таблетки заліза За 30 хв до їди, не розжовуючи
фумарату. Як правильно застосовувати цей ЛЗ?
Хворий на тахіаритмію тривалий час приймав пропранолол яки раптово Синдром відміни
припинив приймати. Стан хворого погіршився, розвинувся
гіпертензивний криз. До якої категорії належить побічна дія?
Хворому 57 років для усунення приступів стенокардіїлікарем в Розвиток толерантності до лз
комплексіз аспірином призначено нітросорбіт 10 мг двічі на добу. Чим
пояснити зміну ефективності фармакотерапії у цьому випадку
Хворому з гіпертензивним кризом ввели клонідин. Який механізм Стимуляція центральних а2-
лежить в основі антигіпертензивної дії клонідину? адренорецепторів.
Хворому з діагностовано хворобою Аддісона лікар призначив замісна
глюкокортикоїди та мінералокортикоїди. Який вид фармакотерапії
лікар передбачив в даному випадку?
Хворому М., який місяць тому переніс гострий інфаркт міокарда, для 100-300 мг
профілактики тромбоутворення призначено ацетилсаліцилову
кислоту.В якій дозі слід рекомендувати цей ЛЗ у такій клінічній ситуації7
Хворому на гігантизм лікар призначив засіб етіологічної терапії, але він Парлодел
забув його назву. Про який засіб йде мова?
Хворому на гострий бронхіт призначили відхаркувальний засіб, після Терпінгідрат
приймання якого виникло блювання .Який з перерахованих ЛЗ міг
спричинити подібну дію .
Хворому потрібно призначити синтетичний аналог соматостатину. Який Окреотид
з перелічених засобів ви можете запропонувати?
Цукровий діабет 1типу відрізняється від цукрового діабету 2 типу Хворіють переважно діти
Через який проміжок часу після перенесеної інфекції може розвинутися Через 2-3 тижні
готрий глумерулонефрит??
Чоловік 40 р., ріст 130 см, пропорційної тіло будови, інтелект Гіпофізарний нанізм
збережений. Яке захворювання можна припустити?
Що таке епілептичний статус? Епілептичний напаж, який
триває більше 30 хвилин або
повторні епілептичні напади,
між якими у хворого свідомість
не відновлюється.
Як називається ендокринне захворювання, спричинене надлишком Гігантизм
соматотропну, яку виникає у дітей та підлітків?
Як правильно вводити препарати інсуліну за базис-болюсною схемою? Базистий інсулін ввечері, інсулін
ультракороткої дії перед
кожним прийомом їжі
Яка група ЛЗ із перелічених не викликає «синдрому відміни» при Блокатори протоної помпи
раптовому припиненні їх вживанні
Яка група лікарських засобів повинна бути використана для лікування Імунодепресанти
хворих на хронічний гломерулонефрит з високою активністю
запального процесу ?
Яка з наведених анемій є мікроцитарною? Залізодефіцитна
Яка з наведених анемій спричиняє прискорення руйнування Гемолітична анемія
еритроцитів?
Яка побічна реакція не характерна для ЛЗ заліза Поліурія
Яке захворювання можна запідозрити у пацієнта, якщо після зняття Виразково-некротична ангіна
нальоту із мигдаликів з’явились кровоточиві виразки?

Який вид фармакотерапії застосовується для лікування хвороби Замісна


Аддісона?
Який головний етіологічний фактор розвитку пієлонефриту?? Інфекція
Який гормон гіпофізу впливає на білковий, вуглеводний і ліпідний Соматропін
обмін, контролює швидкість скелетного та вісцерального росту?
Який з наведенихЛЗ використовують для гігієниносових ходів? Засоби морської води
Який з перелічених лікарських засобів лід застосовувати в першу чергу Папаверин
призначають для усунення приступу ниркової кольки
Який з перелічених лікарських засобів не покращують нирковий ацетомінофен
кровоплин
Який з перелічених лікарських слід застосовувати в першу чергу при Дротаверин
нирковій кольці??
Який з симптомів найбільш характерний для анемій? Слабкість та блідість шкіри
Який з типів побічних дій ЛЗ зустрічається найчастіше? Побічна дія, обумовлена
специфічними
фармакологічними
властивостями ліків
Який зі способів введення лікарських засобів не належить до Підшкірний
парентерального
Який із наведених блокаторів β-адренорецепторів належить до Бісопролол
кардіоселективних?
Який із перелічених ЛЗ застосовують при інфаркті міокарда для Ацетилсаліцилова кислота
проведення тромболітичної терапії?
Який ЛЗ застосовують при гострому інфаркті міокарда з метою Ацетилсаліцилова кислота
тромболітичної терапії?
Який лікарський засіб є препаратом вибору у хворих на цукровий діабет Метформін
2 типу із ожирінням?
Який рівень артеріального тиску є цільовим при лікування АГ у хворих <140/90
похилого віку?
Який чинник зумовлює розвиток мегалобластних анемій? Недостатність заліза в крові
Які з наведених засобів можуть послаблювати терапевтичну дію ЛЗ Аскорбінова кислота
заліза?
Які з наведених продуктів можуть послабити терапевтичну дію ЛЗ Кава, чай
заліза
Які з перелічених збудників найчастіше є причиною розвитку гострого Стрептокок
глумерулонефриту?
Які з форменних елементів транспортують білок зв’язаний із киснем до Еритроцити
клітин по всьому організму
Які зміни в сечі дозволяють вдрізнити гломерулонефрит від Вираженість протеїнурії
пієлонефриту?
Які ЛЗ є найбільш активними щодо мікобактерій туберкульозу? Ізоніазид, рифампіцин

Ураження шкіри у вигляді вузлуватої та кільцеподібної еритеми, що Гостра ревматична лихоманка


може локалізуватися на обличчі, грудях, животі, спині, кінцівках, є
проявом:
Специфічні шкірні прояви ревматизму це: Еритема кільцеподібна
Які з перелічених побічних реакційне характерні для глюкокортикоїдів? Порушення серцевого ритму
Які з перелічених ЛЗ є селективними інгібіторами ЦОГ-2 мелоксикам
Оберіть найбільш правильне твердження для системного червоного Етіологія невідома, найчастіше
вовчака. виникає в жінок молодого віку,
характерний розвиток серозитів
З яким із перелічених збудників асоційований етіопатогенез Бета-гемолітичний стрептокок
ревматизму? групи А
До клінічних ознак системного червоного вовчака належать всі Остеоліз нігтьових фаланг, рот у
перелічені, окрім наступних: вигляді кисету
Виберіть симптом характерний для ревматоїдного артриту Ранкова скутість суглобів
При ревматизмі для етіотропної фармакотерапії не використовують: тетрациклін
Характерною клінічною ознакою ревматоїдного артриту є? Ранкова скутість понад 3
суглобових зон привалістю
понад 1 год
Хворий К. 50 років скаржиться на біль, припухлість та обмеження рухів в Ревматоїдний артрит
суглобах пальців рук, ніг і колін, ранкову скутість до 12 год. хворіє
близько 5 років. При обстеженні спостерігається деформація кистей рук
(«ласти моржа») та колін. Визначте попередній діагноз
Виберіть основні клінічні симптоми гострої ревматичної гарячки: Кардит, поліартрит, хорея
ВкажітьЛЗ для етіотропної терапії фармакотерапії ревматизму: бензилпеніцилін
Назвіть антифіброзний ЛЗ, що застосовують для лікування пеніциламін
ревматоїдного запалення суглобів
При непереносимості антибіотиків пеніцилінового роду для макроліди
профілактики і лікування ревматизму показані ЛЗ із групи:
Лопус-кардит(перикардит, міокардит та ендокардит із розвитком вад Системного червоного вовчака
серця) є проявом:
У жінки 50 р. біль у суглобах кистей рук, значна ранкова скутість; ШОЕ Ревматоїдний артрит
24 мм/год. який діагноз можна припустити?
У якого із перелічених нестероїдних протизапальних засобів найслабше мелоксикам
проявляється ульцерогенна дія?
Дайте визначення поняттю «первинна профілвктика ревматизму» Введення після перенесеної
гострої ревматичної гарячки
бензатину бензилпеніциліну
щомісячно впродовж
індивідуально визначеного
терміну
У хворого М. що лікується з приводу гострої ревматичної гарячки, макроліди
виявляється гіперчутливість до пеніциліну.Які з перерахованих груп
протимікробних ЛЗ йому показані?
Неврологічна симптоматика характерна для ревматизму: хорея
Неврологічна симптоматика характерна для ревматизму хорея
У хворого, який тривалий час лікується з приводу ревматоїдного тріамцінолон
артриту, виявлено гіперглікемію. Який з ЛЗ міг її спричинити?
Хворій П., 57 р. встановлено діагноз ревматоїдного артриту, в анамнезі Целекоксиб
пептична виразка. який ЛЗ з групи НПЗП можна рекомендувати?
Ревматизм характеризується: Руйнуванням тканинних
структур серця
Найчастіша локалізація ураження суглобів при ревматоїдному артриті: П’ясткофалангові суглоби кистей
рук
Ефективність фармакотерапії при ревматизмі визначають усі з Відсутність проявупобічних
перелічених критеріїв окрім реакцій при застосуванні лз
Яка з перелічених ознак не стосується ревматоїдного поліартриту? Ушкодження сонячного
сплетіння
Назвіть ваду серця, яка найчастіше розвивається при ревматизмі: Недостатність
мітральногоклапана
Пацієнт 33 р. скаржиться на значну м’язову слабкість, ранкову скутість в Аутоімунне, імунне ураження
суглобах, яка триває декілька год. При огляді деформація кистей сполучної тканини
(«плавники моржа»). Які процеси лежать в основі розвитку даного
захворювання ?
Назвіть клінічні ознаки системного червоного вовчака Ексудативна еритема на щоках,
вісцерити, лопус -нефрит, лопус
- кардит
Підшкірні вузлики є діагностичним критерієм: Гострої ревматичної лихоманки
Пацієнтка 32 . звернулась до лікаря зі скаргами на ранкову скутість в Ревматоїдний артрит
суглобах, яка триває кілька год. При огляді – симетрична деформація
кистей у вигляді «плавнів моржа», коліна збільшені деформовані. Який
діагноз можна припустити?
У хворого Г. 54 р. на фоні ревматоїдного артриту розвинулась виразка мелоксикам
12 –ти палої кишки. Який НПЗЗіз перелічених рекомендовано
призначити для лікування основного захворювання?
Дайте визначення «вторинна профілактика ревматизму» Введення після перенесеного
гострого стрептококового
тонзиліту бензатину
бензилпеніциліну, одноразово
До основних діагностичних критеріїв ревматизму належать кардит
Назвіть найбільш характерні симптоми коліту: тенезми,диспепсичний синдром

Назвіть ЛЗ із групи патопротекторів – синтетичних простогландинів для Сукральфат


лікування пептичної виразки
Для гострого панкреатиту характерно Нудота, блювання,
пронос,інтенсивний біль
оперізуючого характеру

До вас в аптеку звернувся хворий на виразкову хворобу шлунка зі Антисекреторні ЛЗ


скаргами на нудоту у якого при обстеженні виявлено підвищену
кислотноутворюючу функцію шлунка. Яка група ЛЗ має бути основою
лікування
При пептичній виразці 12-палої кишки шлункова секреція, як правило Підвищена
Хворий А.42 роки після надмірного вживання алкоголю та жирної їжі Пептична виразка 12-палої
відчув гострий напад болю в епігастральній ділянці живота, який через кишки в стані загострення
деякий час набув оперізуючого характеру, зявилося блювання. Дані
лабораторних досліджень: значний лейкоцитоз їз зсувом
лейкоцитарної формули в ліво, прискорене ШОЕ, різке збільшення
концентрації амілази в сечі та крові. Яке захворювання розвинулось у
хворого
До лікаря звернувся хворий зі скаргами на пронос, болі внизу Дисбактеріоз кишечника
живота,слабість, які не залежать від прийому їжі. Хворий втратив вагу. В
анамнезі – нещодавне тривале самолікування антибіотиками з приводу
ангіни. Про яке захворювання слід думати в першу чергу
До симптоматичного лікування діареї належать всі заходи окрім Застосування ЛЗ для
пероральної регідратації

Хвора 52 років перебуває на стаціонарному лікуванні з діагнозом Морфін


гострого панкреатиту. Який їз перелічених Лз призначений у даному
випадку
Для діагностики контамінації H.pylori окрім гістологічного дослідження Уреазний тест
застосовують
Який із перерахованих ЛЗ може маскувати ознаки кровотечі у верхніх Вісмуту субцитрат
відділах шлунково-кишкового тракту(забарвлювати стілець в чорний
колір)
Хворий з хронічними ентероколітом та холециститом що Панкреатин + хіміцелюлоза +
супроводжується важкістю в шлунку після їжі та здуттям живота порошок жовчі бичачої
звернувся до провізора з проханням порекомендувати йому препарат
для покращення процесів травлення. Який з названих засобів найбільш
ефективний
У хворого на пептичну виразку під час комбінованої фармакотерапії Вісмуту субцитрат
випорожнення набули чорного забарвлення. Назвіть Лз який міг
спричинити таку зміну
Оберіть схему фармакотерапії пептичної виразки 12-палої кишки Омепразол,
першої лінії Амоксициклін,кларитроміцин

У схему раціональної фармакотерапії гастриту асоційованого з Н.pilori в Кларитроміцин


першу доцільно включити
В аптеку звернувся хворий з рецептами на омепразол 20 мг х 2р/д, Хронічного гастриту В
кларитроміцин 0.5 х 2р/д, амоксициклін 1г - 2р/д. Така схема терапії
найімовірніше була призначена з приводу
При диференційній діагностиці між пептичною виразкою шлунка і гастродуоденоскопія
гастритом важливим є
Для ерадикації H.pylori при первинному лікуванні пептичної виразки Кларитроміцин
необхідно включити
Хворому з пептичною виразкою 12-палої кишки для ерадикації Дисбіоз
хелікобактерної інфекції було призначино лансопразол, кларитроміцин,
амоксициклін. Яка побічна реакція може виникнути у хворого
Як слід приймати ЛЗ для перорального літолізу некальцифікованих 1 раз на добу ввечері перед
холестеринових конкрементів сном

В аптеку звернувся хворий якому з приводу виразкової хвороби лікар Пригнічення активності Н+, К-
призначив омепразол Який механізм лежить в основі дії даного ЛЗ АТФ-ази
До анти секреторних ЛЗ належать всі окрім Комбінації алюмінію
гідроксиду+магній гідроксид
При інформуванні лікарів про наявність в аптеці жовчогінних Лз Холагол
зазначте якому з них властива холекінетична дія
Хворому С. із виразковою хворобою дванадцятипалої кишки для Пенетрація
ерадикації хелікобактерної інфекції була призначена комбінована
терапія. Який небажаний стан може виникнути у хворого у першу чергу
Який з перелічених ЛЗ не викликає ульцерогенної дії А. Пропранолоз
Виберіть правильне твердження E. лікування засобами жовчних
кислот триває 1-2 місяці
Атеросклеротична бляшка формується внаслідок: Підвищення концентрації ліпідів
та холестерину в крові.

Блокатори бета-адренорецепторів можуть викликати наступні побічні Тахікардія


реакції, окрім:

В аптеку звернувся хворий 75 років, який скаржиться на напади болю за Нітрогліцерин


грудиною ів ділянці серця, задишку, біль голови. Артеріальний тиск у
хворого 110/60 мм. рт. ст. Має глаукому. Який з лікарських засобів йому
протипоказаний?

Для стенокардії не характерно: Колючий характер болю

До лікарських засобів, що виявляють короткотривалий позитивний Добутамін, мілринон


ізотропний ефект належать:

До лікарських засобів, що застосовуються для фармакотерапії серцевої Усі відповіді вірні


недостатності належать:

До симптомів передозування нітрогліцерином не належить: Артеріальна гіпертензія

Згідно даних хронофармакології підвищити ефект статинів можна, Не має значення


призначаючи їх у наступний період доби:

Лікарським засобом, що НЕ належить до інгібіторів ГМГ-КоА-редуктази Холестирамін


є:

Механізм дії ловвастатину полягає у: Інгібуванні ферменту ГМГ-КОА-


редуктаза

Найчастішим симптомом серцевої недостатності є: Задишка

Оберіть лікарський засіб, що застосовується при ІХС для нормалізації Розувастатин


холестеринемії шляхом інгібування активності ГМГ-КоА-редуктази?
Оберіть тип судин, які можуть уражатися атеросклерозом Артерії

Серцеві глікозиди, отримані з листя конвалії є діючою речовиною Корглікон


лікаського засобу:

Характерною офтальмологічною ознакою дигіталісної інтоксикації є: Забарвлення оточуючих


предметів в жовтий та зелений
колір
Хвора М., 64 роки приймає ізосорбіду динітрат по 10мг 4р/д. Через три Відмінити нітрати і додатково
тижні від початку терапії збільшилась кількість та тривалість нападів призначити сидноніми
стенокардії, що лікар пов’язав із розвитком толерантності до нітратів.
Які рекомендації щодо подальшої фармакотерапії є раціональним у
цьому випадку?
Хворий 52років, з метою купування нападу стенокардії прийняв Валідол
таблетку нітрогліцерину, після чого відчув сильний головний біль. Який
лікарський засіб слід рекомендувати приймати одночасно з нітратами
для зменшення інтенсивності болю голови?
Хворий зі стабільною формою стенокардії протягом 3-х місяців вживає Молсидомін
ізосорбіду динітрат (кардикет) 40 мг на добу, який в останній тиждень
став менш ефективним. Після обстеження лікар зробив висновок про
розвиток толерантності до ЛЗ. Який з перелічених засобів слід
призначити хворому замість кардикету?

Хворого турбують періодичні напади болю за грудиною, з приводу чого Від кількох секунд до 20-30
він звернувся в аптеку з проханням порекомендувати ЛЗ. Після розмови хвилин
з хворим провізор зробив висновок, що у нього стенокардія. Яка
тривалість больового синдрому при стенокардії?
Через декілька місяців після початку лікування у хворого знову Виникнення толерантності до
почастішали напади стенокардії. Які можливі причини погіршення нітратів
стану?

Чоловіка 62-х років після фізичного навантаження почав турбувати біль Нітрогліцерин
за грудиною з іррадіацією в ліву руку. Який лікарський засіб слід
рекомендувати для його усунення?

Який із блокаторів бета-адренорецепторів належить до Бісопролол


кардіоселективних:

Який лікарський засіб застосовують при гострому інфаркті міокарда з Ацетилсаліцилова кислота
метою тромболітичної терапії?

Який лікарський засіб отримують із трави горицвіту весняного, що Адонізид


виявляє позитивний ізотропний ефект?

Антибіотики якого ряду найдоцільніше призначити хворому на сифіліс? Пеніциліни


Безпека фармакотерапії сифілісу у вагітних еритроміцином полягає у ЛЗ не поникає через плаценту
тому, що:
В аптеку звернувся хворий, якому дерматолог з приводу трихофітії Гризеофульвін
волосистої частини голови призначив протигрибковий антибіотик, який
має здатність накопичуватися в клітинах з високим вмістом кератину
(шкіра, нігті,волосся). Невдовзі хворий почав скаржитись наголовний
біль, дезорієнтацію,кропивницю. Який препарат був призначений
хворому?

Виберіть антибактеріальний засіб для лікування хламідійної інфекції: Роваміцин

Виберіть антибіотик – макролід: Спіраміцин

Визначте який з названих лікарських засобів не належить до Ципрофлоксацин


антибіотиків групи цефалоспоринів.
Вкажіть ЛЗ, який не можна застосовати для лікування сифілісуу дітей до Тетрациклін
8 років?
Дитина 10 років страждає на стафілококовий дерматит. Лікування Інактивація бета-лактамази
бензилпеніциліном не дало результатів, але після додаткової терапії стафілококів
клавулановою кислотою швидко настало одужання. Яка причина
позитивної дії цього препарату?

Для фармакотерапії сифілісу лікарськими засобами вибору є: Пеніциліни

До аптеки звернувся хворий, у якого після курсу антибіотикотерапії ЛЗ, що містить лакто- та
розвинувся дисбактеріоз. Який лікарський засіб доцільно йому біфідобактерії
призначити?
До якого класу антибіотиків належить лікарський засіб цефтріаксон? Цефалоспорини 3 покоління

До якого класу антибіотиків характерні такі побічні реакції: диспепсія, Тетрацикліни


дисбактеріоз, глосит,. Ураження нирок і печінки, гіпоплазія дентальної
емалі, схильність до карієсу?

Надайте рекомендації хворому, який приймає офлоксацин у таблетках Уникати прямого сонячного
для лікування гонореї: світла, можлива
фотосенсибілізація
Назвіть антибіотик з групи цефалоспоринів 1 покоління, що вважається Цефазолін
«золотим стандартом» парентеральної терапії:
Назвіть антибіотик з групи цефалоспоринів 3 покоління для пероральної Цефіксим
терапії.
Назвіть антибіотик з групи цефалоспоринів 3 покоління для пероральної Цефіксим
терапії:
Найчастішими ускладненнями фармакотерапії при застосуванні Алергійні реакції
антибактеріальних засобі пеніцилінового ряду є:

Оберіть мікроорганізм, що є збудником сифілісу: Бліда трепонема

Офлоксацин можна застосувати для фармакотерапії після досягнення: 18 років

Після курсу антибактерійної терапії у дитини 2 р. з’явились сіре Хлорамфенікол


забарвлення шкіри, диспептичні явища, гіпотермія. Який ЛЗ міг
викликати такі побічні явища?

При непереносимості пеніцилінів фармакотерапію сифілісу у вагітних Еритроміцин


можна проводити застосовуючи:

При підозрі на гострий апендицит: Усі відповіді вірні

У 6-літньому віці дитина захворіла пневмонією, з приводу якої був Аміноглікозиди


назначений антибіотик. Після лікування дитина втратила слух. Яка група
антибіотиків могла викликати це ускладнення?

У пацієнта К., 58 років, у минулому був набряк Квінке на амоксицилін. Цефазолін


Застосування яких ЛЗ протипоказане через ризик перехресної алергії?

У пацієнта Н., 22 роки на зовнішньому статевому органі з’явилася ерозія Сифіліс


з валикоподібними краями та червоном блискучим центом – твердий
шанкр. Відзначає загальну слабкість, втрату апетиту. Регіонарні
лімфатичні вузли – збільшені. Яке захворювання можна запідозрити?
У хворого на перитоніт висіяна синьо-гнійна паличка. Який лікарський Цефтріаксон
засіб з групи цефалоспоринів ІІІ покоління слід призначити?

У хворої після лікування важкої гінекологічної інфекції розкинувся Кліндаміцин


псевдомембранозний ентероколіт. Який з перелічених засобів у першу
чергу міг викликати ускладнення?

Хворий Л., 56 років ВІЛ-інфікований був госпіталізований з приводу Амфотеицин В


генералізованого кандидомітозу. Який з нижченаведених
протигрибкових препаратів застосовується для лікування системних
мікозів?

Хворому з генералізованою формою сальмонельозу була призначена Хлорамфенікол


антибактеріальна терапія. Через деякий час при проведенні загального
аналізу крові була виявлена ретикулота грунулоцитипенія, відмічалось
деяке зниження кількості еритроцитів. При призначенні якого
антибіотику найбільш вірогідна подібна зміна картини крові?
Хворому з інфекційним захворюванням сечовивідної системи Похідні фторхінолону
призначено лікарський засіб з сильною протимікробною дією. В
механізмі дії провідним є вплив на РНК-гіразу збудників. Препарат якої
групи призначив хворому лікар?

Хворому призначили антибіотик четвертого покоління цефалоспоринів, Цефепім


який добре проникає крізь мембрани грамнегативних бактерій,
забезпечує стійкість відносно лактамаз і має широкий спектр
антибактеріальної дії. Який засіб лікар призначив хворому.

Цефтріаксон несумісний з розчинами, що містять солі кальцію. Вкажіть Розчин Рінгера


такий розчин:

Цефтріаксон несумісний з розчинами, що містять солі кальцію. Вкажіть Розчин Рінгера


такий розчин:
Яка побічна дія антибактерійних засобів з групи фторхінолонів зумовила Виразкування хрящів та
заборону до їх застосування дітьми та підлітками? порушення їх формування

Який з наведених антибіотиків має нефротоксичну дію: Гентаміцин

Який із вказаних лікарських засобів відноситься до засобів етіотропної Ацикловір


фармакотерапії оперізуючого лишаю (захворювання, яке викликає вірус
простого герпесу):

Який із вказаних лікарських засобів відноситься до засобів етіотропної Ацикловір


фармакотерапії оперізуючого лишаю (захворювання, яке викликає вірус
простого герпесу):
- До лікаря звернувся хворий зі скаргами на помірний біль в епігастрії, нудоту,
дратівливість, поганий сон. При обстеженні виявлено пептичну виразку шлунка,
Helicobacter pylori, підвищену кислотоутворюючу функцію шлунка. Яка група
лікарських засобів має бути основою лікування? антибактеріальні
- Назвіть лікарський засіб із групи цитопротекторів – синтетичних простагландинів для
лікування пептичної виразки: мізопростол
- При огляді хворого виявлено збільшений живіт, випинання пупка, наявність на шкірі
живота судинних зірочок та розширених судин у вигляді «голови медузи». Для якого
захворювання характерні такі симптоми цироз печінки
- Хворому 40 років зі скаргами на печію дисфагію біль в епігастральній ділянці натше та
через 3 години післяїжі, проведено фіброскопічне дослідження. Виявлена
дуоденальна виразка, гістологічний аналіз слизової оболонки шлунка – наявність
Helicobacter pylori. Яку групу лікарських засобів Ви порекомендуєте хворому?
протимікробні
- Для ерадикації Helicobacter pylori при первинному лікуванні пептичної виразки
необхідно включити Кларитроміцин
- У хворої після вживання жирної їжі виникли тупі ниючі болі в правому підребер’ї, які
іррадіюють в праву лопатку, нудота, відчуття гіркоти в роті. Яке дослідження слід
провести для верифікації діагнозу захворювання? УЗД печінки та жовчного міхура
- В аптеку звернувся хворий з пептичною виразкою дванадцятипалої кишки в анамнезі
з проханням порадити йому лікарський засіб. Зараз турбують болі у верхній ділянці
живота. Провізор порадив ланзопразол. Укажіть механізм дії рекомендованого
лікарського засобу Блокатор Н+/К+-АТФ-ази
- При обсетеженні хворого з цирозом печінки виявляють наступні симптоми, окрім
наступного: нефроптоз
- У хворого 60 років, після емоційного перенапруження з’явився біль за грудиною з
іррадіацією в ліву руку. Назвіть лікарський засіб, який тамує больовий синдром?
Нітрогліцерин
- Хворий Л. 70 років госпіталізований в кардіологічне відділення з нападом стенокардії,
де йому було призначено лікування. Після прийому хворий відмітив сильний біль
голови та нудоту. Для якого антиангінального засобу характерна така побічна реакція?
Ізосорбіду динітрат.
- Основою лікування пневмонії є: антибактеріальна терапія.
- Збудження наступних рецепторів веде до бронходилатації: бета 2-адренорецепторів.
- Основним методом оцінки важкості перебігу хронічного обструктивного бронхіту є:
спірометрія.
- Який ЛЗ покращує відходження мокротиння амброксол
- У клінічних ситуаціях призначають антибіотики при бронхіальній астмі? За наявності
ознак бактерійної інфекції у бронхо-легеневій системі
- Хворий К.,50 років ,який протягом 5 років страждає на артеріальну
гіпертензію,відзначає погіршення стану. Лікар визначив при опитуванні й огляді ряд
ознак. Яка з наведених нижче належить до об’єктивних? АТ 170/90 мм рт ст
- Найчастішою причиною пневмонії є: пневмокок.
- Основу терапії персистуючої астми складає: протизапальна терапія.
- Основним методом діагностики пневмонії є: рентгенологічне дослідження.
- Назвіть найхарактерніші ускладнення гострого бронхіту: Пневмонія, хронічний
бронхіт
- При призначенні якого ЛЗ бажаний моніторинг його концентрації у плазмі крові?
Теофілін.
- Терапія, спрямована на корекцію окремих проявів хвороби: симптоматична.
- Ключовими напрямками у фармакології 3-ого тисячоліття є всі , за винятком:
зростання ролі фармакокінетики і токсикології
- Старечий вік визначається наступним діапазоном: 75-89 років.
- Хворий почав лікуватися з приводу гіпертонічної хвороби. Лікар у листі лікарських
призначень зробив записи назв призначених засобів. Що повинно бути ще наведено у
листах лікарських призначень Разова доза, кратність приймання та шлях введення
- Ембріотоксичні і тератогенні реакції розцінюються як наступний тип побічної дії ліків:
тип D.
- Про який вид фармакотерапії йдеться, коли хворому з ревматоїдним артритом лікар
призначає один з нестероїдних протизапальних засобів? Патогенетична.
- Хворому з діагностованою хворобою Адісона лікар призначив глюкокортикоїди та
мінералокортикоїди. Який вид фармакотерапії передбачив лікар в даному випадку?
Замісна
- У хворої А. 30 років ,гостра респіраторна вірусна інфекція. Встановлено підвищену
температуру тіла,біль у горлі, у м’язах, закладання носа ,розвинення пневмонії. Що
можна віднести до ускладнення захворювання Розвиток пневмонії
- Частота побічних ефектів при амбулаторному лікуванні коливається в межах? 8-15%
- Ушкодження кровоносних судин з наступною кровотечею пов’язане з дією наступних
етіологічних чинників: механічні, фізичні, біологічні
- Хворому з гіпоацидним гастритом лікар призначив ацедин-пепсин. Який вид
фармакотерапії передбачив лікар в даному випадку? Замісна.
- До завдань ефективної фармакотерапії НЕ належить: проведення радикального
хірургічного лікування.
- До «медикаментозного» анамнезу, що збирають у хворого при проведенні
моніторингу фармакотерапії, не належить з’ясування наступних даних: комплаєнс або
шкідливі звички.
- Інфекційні хвороби виникають внаслідок впливу: виключно біологічних
- Назвіть групу лікарських засобів , раптове припинення вживання якої може
спричинити розвиток ад? Глюкокортикоїди.
- Хворий потрапив до лікарні. В анамнезі життя лікар відобразив ряд показників. Що
треба висвітлити у цьому розділі історії хвороби?Терапію,яка проводилася раніше
- Втрата свідомості при збереженні деяких рефлекторних форм діяльності розцінюється
як: сопор
- Назвіть другий етап раціональної фармакотерапії? Вибір фармакологічної групи ЛЗ.
- До основних методів обстеження хворих не належить: інструментальна діагностика.
- Назвіть цефалоспорин 3 покоління для пероральної терапії: цефіксим.
- Базисна терапія хронічного обструктивного бронхіту на початкових етапах включає:
Бронхолітичну терапію
- Який засіб групи адреноміметичних засобів порадите хворому з бронхіальною
астмою? Кленбутерол(спіропент)
- М-холінолітики широко застосовуються при: хронічному обструктивному
захворюванні легень
- Для тривалого лікування бронхіальної астми перевагу надають: Інгаляційному
способу введення ЛЗ.
- При бронхіальній астмі інтал призначають?Для запобігання нападів ядухи.
- Засобами вибору для зняття нападу задухи при бронхіальній астмі є: бета-2-агоністи.
- До можливих шляхів корекції порушень, спричинених тривалою фармакотерапією
хвороби Паркінсона належать усі наведені крім:
збільшення кратності прийому леводопи чи тривалості дії кожної дози
- У хворого К., 80 р., з’явився тремору в спокої, більш виражений у правій руці, що
зникає під час сну, збіднення рухів та ригідність. Яке нервологічне захворювання
можна припустити?
Хвороба Паркінсона
- Розрахувати константу елімінації цефазоліну, якщо відомо, що його період
напіввиведення становить 2 год: 0.347
- серед вказаних чинників, оберіть ті, які впливають на біодоступність лікарського
засобу
місце введення, значення рН в місці введення (функціональний стан нирок)
- фармакокінетичний параметр, яки характеризує час виведення лікарського засобу з
організму t1/2
- розрахувати константу елімінації лікарського засобу, якщо відомо, що його об'єм
дистибуції становить 63 л, кліренс – 40 мл/хв 0,635
- вкажіть, коли в організмі встановлюється рівноважна концентрація в момент
введення
- метаболізм багатьох лікарських засобів в організмі залежить від усі відповіді
правильні
- вкажіть ідеальну для фармакотерапії (з точки зору фармакокінетики) частоту
застосування лікарських засобів при багаторазовому їх застосуванні 1 раз на 1 період
напіввиведення
- до факторів, які впливають на елімінацію лікарського засобу належать всі, окрім
значення Рн
- визначте період напіввиведення аміодарону, якщо відомом, що його константа
елімінації становить 0,001 год -1 693 год
- Назвіть збудник, що найчастіше викликає розвиток пієлонефриту Кишкова паличка
- Для лікування пієлонефриту перш за все застосовують антибіотики з групи Похідні
хінолінів
- Які характерні симптоми гострого гломерулонефриту дозволяють відрізнити його від
гострого пієлонефриту? Висока протеїнурія, гіперкреатинінемія
- Хвороба аддісона – замісна терапія
- Протикашльовим ефектом не володіє : Цефодокс
- Вагітна жінка 26 років (3 триместр вагітності) скаржиться на підвищення температури до 39 С.
Який жарознижувальний ЛЗ можна запропонувати в цій ситуації? Парацетамол
- До факторів ризику захворюваності на ГРВІ у дітей відносять усі, крім: Малої маси тіла при
народженні
- До побічних ефектів симпатоміметиків для симптоматичного лікування нині у відносять усі,
крім: Зниження АТ
- В аптеку звернулася молода жінка зі скаргами на відчуття сухості, подряпування й болючості в
горлі. Які основні принципи фармакотерапії у цьому випадку?Зменшення подразнення
- Перелічіть ЛЗ другого ряду для лікування туберкульозу (при доведеній резистентності до
протитуберкульозних ЛЗ 1 ряду): Канаміцин, етіонамід, циклосерин, капреміоцин, ПАСК,
ципрофлоксацин
- Препаратом для симптоматичного лікування болю в горлі є всі, крім:Саламолу
- Найчастішими ускладненнями фармакотерапії при застосуванні антибактеріальних засобів
пеніцилінового ряду є:Алергічні реакції
- До правил раціонального харчування при застуді відносять усі, крім:Збільшення надходження
жирів з їжею
- У жінки, яка тривалий час лікується з приводу інфільтративно-вогнищевого туберкульозу,
різко знизився слух. Який з перерахованих лікарських засобів міг спричинити таку побічну
дію? Стрептоміцин
- Причинами сухого кашлю є всі, крім:Бронхоектатична хвороба
- Який із наведених нижче препаратів не можна рекомендувати відвідувачу аптеки для
самостійного лікування болю в горлі? Стрепсілс
- До факторів, що сприяють розвитку застуди усі, крім: Прогулянок
- Який з перелічених протитуберкульозних засобів виявляє ототоксичну дію? Стрептоміцин
- До методів зниження підвищеної температури тіла у дітей відносять усі, крім: Зниження за
допомогою аспірину
- Які ЛЗ використовуються для зменшення набряку слизової оболонки носа при хронічному
риніті? 2% розчин протаргол
- Дитині призначено мікстуру, що містить відхаркувальний засіб. Щоб прискорити її одужання,
мати самостійно підвищила дозу препарату, внаслідок чого у дитини виникло
блювання.Назвіть можливий препарат. Настій трави термопсису
- Який із наведених нижче препаратів не можна рекомендувати відвідувачу аптеки для
самостійного лікування болю в горлі? Біопарокс
- Які антагоністи кальцію належать до групи бензотіазепінів: Дилтіазем
- Назвіть номер дієти для лікування хворого з артеріальною гіпертензією та ішемічною
хворобою серця? 10
- До якої групи ЛЗ, що діють на систему зсідання крові, належить альтеплаза? Тромболітики
- До блокаторів рецепторів ангіотензину II належить: Телмісартан
- Який із наведених блокаторів кальцієвих каналів належить до групи похідних
фенілалкіламіну? Верапаміл
- Термін «есенціальна гіпертензія» означає: Первинну …
- Який ЛЗ – антидот слід застосувати, якщо геморагічні ускладнення виникли на фоні лікування
гепарином? Протаміну сульфат
- На прийом до лікаря звернулась пацієнтка 55 р. зі скаргами на біль голови в потиличній
ділянці, періодичне підвищення АТ, зниження працездатності. Хворіє декілька тижнів після
психоемоційного навантаження. На момент огляду АТ 165/100 мм рт. ст. При обстеженні
виявлено гіпертрофію лівого шлуночка і зміни сітківки очей. Який попередній діагноз можна
поставити у цьому випадку? АГ ІІ стадії
- Який із наведених блокаторів бета-адренорецепторів виявляє властивості альфа-блокаторів?
Лабеталол
- До якої групи гіпотензивних ЛЗ належить моксонідин? Агоністи імідазолінових рецепторів
- В комплекс лікування неускладненого туберкульозу легень включаючи: Вітаміни групи В
- Основними напрями фармакотерапії фарингіту є наступні, крім: Гіпосенсибілізація
- Який препарат не показаний для симптоматичного лікування ГРВІ у дорослого пацієнта 2-3
дні? Аугментин
- Підберіть препарат для симптоматичного лікування болю в горлі: Гівалекс
- Які краплі доцільно використати для місцевого лікування отитів? Розчин борного спирту
- Причинами вологого кашлю є всі, крім: Ларингіт
- До аптеки звернулася мама двомісячної дитини з проханням видати жарознижувальний
препарат. Який препарат Ви порекомендуєте для дитини? Парацетамол
- Порекомендуйте засіб для усунення вологого кашлю водію громадського транспорту?
Амброксол
- Хворий Ю., 30 років, скаржиться на більшу правому вусі ниючого характеру з іррадіацією в
скроню, який підсилюється при ковтанні й кашлі. Останню добу визначає підвищення
температури до 38 С., зниження слуху, закладання й шум у правому вусі. Яке можливе
захворювання у хворого? Гострий середній отит
- У чоловіка, який тривалий час лікується з приводу вогнищевого туберкульозу, різко
погіршився зір, з’явилися крововиливи у сітківку. Який з перерахованих лікарських засобів міг
спричинити таку побічну дію? Етамбутол
- Яких із препаратів не належить до ненаркотичних протикашльових препаратів? Кодеїну
фосфат
- Хворий страждає на стенокардію напруги у поєднанні з артеріальною гіпертензію. Яка з
наведених груп ЛЗ найбільш оптимальна у даному випадку? Блокатори бета-
адренорецепторів
- У хворої В., 55 р., яка тривалий час приймає гідрохлортіазид з приводу лікування артеріальної
гіпертензії, з’явились м’язова слабість, судоми м’язів нижніх кінцівок, відчуття перебоїв у
роботі серця. Яке ускладнення фармакотерапії найбільш ймовірно виникло у даної хворої?
Гіпокаліємія
- 1.Антидотна терапія при отруєнні чадним газом: А.гіпербарична оксигенація
- 2.Обтураційно-аспіраційна форма порушення дихання при отруєннях зумовлена,в
основному: Д.гіперсалівацією і бронхореєю
- 3.Хворий з пораненням нижньої кінцівки перебуває в стані травматичного шоку. Його
АТ 90/60 мм.рт.ст. Необхідно виконати первинну хірургічну обробку рани. Можлива
тривалість такої операції 10-20 хвилин. Який наркозний засіб для внутрішньовенного
введення доцільно ввести хворому? В.тіопентал-натрій
- 4.Пацієнта було доставлено у відділення невідкладної допомоги без свідомості;
сухожилкові рефлекси підвищені; тонус мязів у нормі; зінниці звуженні. Брадикардія,
дихання типу Чейна-Стокса. Попередній діагноз: передозування морфіном. Який
препарат слід використати для специфічної терапії цього гострого
отруєння?С.налоксон
- 5. У хворого, 42 років, спостерігаються біль у животі, закрепи, здуття живота, слабість
впродовж 20-и років. Вкажіть на найчастішу причину: Д.хвороба Гіршпрунга
- 6.Ваша знайома С., 22 років, після тривалого перебування у натовпі звернулась до
аптеки зі скаргами на слабкість, запаморочення. Раптово короткочасно втратила
свідомість, швидко отямилась. Шкіра бліда, кінцівки холодні, діяльність серця
ритмічна, ЧСС 100 уд/хв., пульс слабкого наповнення. Який ЛЗ доцільно ввести
хворій? Д.мезатон
- 7.Чоловік, 27 років, який приймає інсулін з приводу цукрового діабету, знепритомнів.
Шкіра суха,гаряча, дихання шумне. Які ЛЗ доцільно вводити у даному випадку: В.20%
р-н глюкози
- 8.При отруєнні блідою поганкою найефективніша детоксикація досягається за
допомогою: А.гемодіалізу і гемосорбції
- 9.Пацієнту, 35 років, з анкілозуючим спондилоартритом призначили препарат, що має
виражену протизапальну дію та час напіввиведення 40-45 годин. Тому він призначається один
раз на день. Як правило, в-ся для лікування ревматизму, ревматоїдного артриту, остеартриту,
загострення подагри. Який препарат було призначено? Д.піроксикам
- 10.Хвора скаржиться на біль і обмеження рухів у колінних суглобах. Який із нижчеперелічених
ненаркотичних анальгетиків слід застосувати з урахуванням того, що пацієнтка має в анамнезі
хронічний гастродуоденіт? В.целекоксиб
- 11.Вкажіть на кардинальний симптом наявності гострої хірургічної патології органів черевної
порожнини: Е.біль
- 12.Після випадкового застосування рідини у хворого спостерігаються болі в епігастрії,
блювання, розвиток шокового стану. Слизова оболонка порожнини рота має білий колір.
Блювотні маси білого кольору, швидко темніють. Що є причиною отруєння та який засіб
потрібен для промивання шлунка в цій ситуації? ТУПО НЕ ЄБУ
А.серцеві глікозиди, унітіол
В.формальдегід, розчин амонію хлориду
С.сильна кислота, холодна вода
Д.морфін, розчин калію перманганату
Е.сіль срібла, розчин натрію хлориду
- 13.Жінці, 46 років, при фіброзно-кістозній мастопатії, показано:С.пункцію+цитологічне
дослідження
- 14.Жінка випила концентровану кислоту з метою самогубства. Її доставлено в лікарню з
некрозом слизової оболонки порожнині рота й сильним болем у здовж стравоходу та
епігастрії. Біохімічний аналіз крові виявив метаболічний ацидоз. Який препарат слід
застосувати лікарям?А.натрію гідрокарбонат
- 15.У дівчинки, 9 років, без свідомості знижена температура тіла, суха шкіра, сухий червоний
язик, часте поверхневе дихання, тахікардія. Яку кому слід запідозрити?Д.гіперглікемічну
- 16.При якому виді гіпоксії оксигенотерапія найменш ефективна:А.гістотоксичній
- 17.У чоловіка, 56 років, при огляді виявлено пахвинну килу. Якою буде ваша
рекомендація?Е.планове оперативне втручання
- 18.До аптеки звернулась жінка з проханням порекомендувати ЛЗ для зниження температури
тіла у хворої дитини 4-х років. Засобом вибору в цій ситуації є:Д.парацетамол
- 19.Дитина, 11 років, постраждала при пожежі. Опіки рук з розвитком пухирців, плаче від
болю. Які ЛЗ доцільно негайно ввести хворій?С.анальгін
- 20.У пацієнтки С., 39 років, яка страждає на жовчнокам’яну хворобу, виник напад різкого
болю в правому підребер’ї, нудота, блювання. Які ЛЗ треба невідкладно використати в
даному випадку?В.но-шпа
- 21.У пацієнта на гіпертонічну хворобу та ішемічну хворобу серця виник гіпертонічний криз з
АТ 220/120 мм рт.ст., ЧСС 110 уд/хв.. Які ЛЗ можна в-ти для лікування гіпертонічного кризу у
пацієнта?В.метопролол
- 22.За умов неможливості довенного введення ліків при реанімації адреналін найчастіше
вводять:В.в трахею
- 23.Який діуретик в-ся при лікуванні набряку лягенів?С.манітол
- 24.Перед кардіохірургічною операцією хворому з вродженою вадою серця проводять його
катетеризацію…наркозу під час цієї діагностичної маніпуляції в-ли кетамін, який не викликає
кардіодепресії…є порушення кровообігу. Які побічні явище можливі при дії цього наркозного
засобу?А.пригнічення дихання, ацидоз
- 25.Гостре отруєння метанолом розвинулось унаслідок прийому сурогатів алкоголю. Його
ранніми проявами були нудота,блювання, головний біль, запаморочення. Через 12 годин
хворого доставлено в реанімаційне відділення, порушенням зору та судомами. У складі
комбінованої терапії йому розпочато внутрішньовенне, крапельне введення 5% розчину
спирту етилового. Який ефект етанолу в-ся в цьому випадку?Е.антидотна дія
- 26.Анестезіолог проводить наркоз ізофлураном у комбінації із закисом азоту. Він
спостергігаєЮ шо АТ,пульс і дихання стабільні, рефлекси відсутні, а скелетні мязи
перебувають у розслабленому стані. Для якої стадії наркозу х-ні такі ознаки?В.хірургічного
сну
- 27.Пацієнта, 43 років, було доставлено у відділення невідкладної допомоги в стані коми,
відзначається блідість шкіри, зниження АТ, пригнічення дихання, гіпорефлексія, мимовільне
сечовипускання й дефекація. Повітря, що видихається має специфічний запах алкоголю.
Діагноз: гостра алкогольна інтоксикація. Крім інших заходів хворому внутрішньовенно
вводять розчин глюкози з інсуліном та вітамінами. Яка мета такої терапії?Д.прискорення
біотрансформації етанолу
- 28.Хвора перебуває в реанімації. ЇЇ дихання підтримується шляхом штучної вентиляції легень.
З метою седації хворій проводять постійну інфузію з неінгаляційним наркозним засобом, який
у разі болюсного введення діє 30 секунд. Який засіб застосовують реаніматологи?В.кетамін
- 29.Пацієнт, 32 років,хворіє близько 2-х років виразковою хворобою із локалізацією у малій
кривизні шлунку. Вкажіть спосіб лікування даного з-ння:А.стаціонарне консервативне в
гастроентерологічному відділенні
- 30.У комплексній терапії гострого артриту використали диклофенак-натрій як потужний
протизапальний агент. Відзначено зменшення набряку в ділянці суглоба. За даними
рентгенографії,кількість рідини в порожнині суглоба редукувалась. На яку стадію запалення
найбільше впливає цей ненаркотичний анальгетик?А.проліферація
- 31.У хворого, 72 років, з ознаками перенесеного інсульту діагностовано защемлену пахвинну
килу. Виберіть лікувальну тактику:С.спрямувати в хірургічний стаціонар для операції
- 32.При асфіксії внаслідок стороннього тіла при неможливості усунути його на місці випадку
слід:С.ввести повітровід
- 33.Слабким протизапальним ефектом характеризується:С.анальгін та парацетамол
- 34.Хворий на стенокардію випадково прийняв токсичну дозу таблеток нітрогліцерину
пролонгованої дії. Розвинулись головний біль, задишка, порушення зору, гіпотензія й
тахікардія. Шкіра гіперемована, ціаноз губ і нігтів. Викликана швидка допомога доставила
хворого в місцевий токсикологічний центр, де йому було надано невідкладну допомогу. Який
антидот в-ли лікарі? С.хромосмон
-

You might also like